ГДЗ к учебнику Химия 9 класс Ерёмин В этой статье мы затронем тему ГДЗ для очередного учебника по химии за 9 класс. Действительно, не смотря на то, что у школьников условно один предмет химии, учиться они могут по разным учебникам. Так вот, в этой статье речь пойдет о готовых домашних заданиях к учебнику по химии для девятого класса, авторы учебника В. В. Ерёмин и другие, 2019 год издания. Что бы нам хотелось отметить, что весь материал здесь подан в общем. То есть вся органическая химия дана в этом учебнике очень поверхностно. Если собираетесь сдавать ОГЭ по химии, нужно будет готовиться к органике по другим пособиям в том числе!

Готовые домашние задания к учебнику по химии для девятого класса, авторы учебника В. В. Ерёмин и другие, 2019 год издания.

Обратите внимание, что вся органическая химия дана в этом учебнике очень поверхностно. Если собираетесь сдавать ОГЭ по химии, нужно будет готовиться к органике по другим пособиям.

Кликайте по вкладкам с номерами параграфов, чтобы увидеть правильные ответы.

Ответы (ГДЗ) к учебнику по химии 9 класс Еремин:

ГДЗ к учебнику Химия 9 класс Ерёмин В этой статье мы затронем тему ГДЗ для очередного учебника по химии за 9 класс. Действительно, не смотря на то, что у школьников условно один предмет химии, учиться они могут по разным учебникам. Так вот, в этой статье речь пойдет о готовых домашних заданиях к учебнику по химии для девятого класса, авторы учебника В. В. Ерёмин и другие, 2019 год издания. Что бы нам хотелось отметить, что весь материал здесь подан в общем. То есть вся органическая химия дана в этом учебнике очень поверхностно. Если собираетесь сдавать ОГЭ по химии, нужно будет готовиться к органике по другим пособиям в том числе!

Готовые домашние задания к учебнику по химии для девятого класса, авторы учебника В. В. Ерёмин и другие, 2019 год издания.

Обратите внимание, что вся органическая химия дана в этом учебнике очень поверхностно. Если собираетесь сдавать ОГЭ по химии, нужно будет готовиться к органике по другим пособиям.

Кликайте по вкладкам с номерами параграфов, чтобы увидеть правильные ответы.

Ответы (ГДЗ) к учебнику по химии 9 класс Еремин:

§1

Глава 1. Стехиометрия. Количественные отношения в химии.

§1. Моль – единица количества вещества

1. Дайте определение понятия "моль".

Ответ:

Моль – это количество вещества, содержащее 6,02∙1023 структурных единиц данного вещества.

2. В стакане воды содержится 11 моль этого вещества. Рассчитайте число молекул воды в стакане.

Ответ:

Дано:

`n(H_2O) = 11" моль"`

`N(H_2O) = ?`

Решение

`N(H_2O) = n(H_2O)*N_A = 11*6.02*10^23 = 6.62*10^24`

Ответ: `N(H_2O) = 6.62*10^24`.

3. Плёнка золота содержит 3,01∙1019 атомов золота. Рассчитайте количество вещества золота (в молях).

Ответ:

Дано:

`N(Au) = 3.01*10^19`

`n(Au) = ?`

Решение

`n(Au) = (N(Au))/N_A = (3.01*10^19)/(6.02*10^23) = 0.5*10^(-4)" моль"`

Ответ: `n(Au) = 0.5*10^(-4)" моль"`.

4. Сколько молей атомов кислорода содержится в углекислом газе, количество вещества которого: а) 1 моль; б) 4 моль; в) 0,37 моль?

Ответ:

Дано:

`n_а(CO_2) = 1" моль"`

`n_б(CO_2) = 4" моль"`

`n_в(CO_2) = 0.37" моль"`

`n_а(O) = ?`

`n_б(O) = ?`

`n_в(O) = ?`

Решение

`n(O) = 2*n(CO_2)`

`n_а(O) = 2*1 = 2" моль"`

`n_б(O) = 2*4 = 8" моль"`

`n_в(O) = 2*0.37 = 0.74" моль"`

Ответ: `n_а(O) = 2" моль"`, `n_б(O) = 8" моль"`, `n_в(O) = 0.74" моль"`.

5. Рассчитайте количество вещества оксида серы (VI) SO3, если известно, что количество атомов кислорода в нём составляет: а) 1 моль; б) 3 моль; в) 0,18 моль.

Ответ:

Дано:

`n_а(O) = 1" моль"`

`n_б(O) = 3" моль"`

`n_в(O) = 0.18" моль"`

`n_а(SO_3) = ?`

`n_б(SO_3) = ?`

`n_в(SO_3) = ?`

Решение

`n(SO_3) = (n(O))/3`

`n_а(SO_3) = 1/3 = 0.33" моль"`

`n_б(SO_3) = 3/3 = 1" моль"`

`n_в(SO_3) = 0.18/3 = 0.06" моль"`

Ответ: `n_а(SO_3) = 0.33" моль"`, `n_б(SO_3) = 1" моль"`, `n_в(SO_3) = 0.06" моль"`.

6. Сколько молей электронов содержится в одном моле: а) золота; б) хлорида натрия?

Ответ:

Дано:

`n_а(Au) = 1" моль"`

`n_б(NaCl) = 1" моль"`

`n_а(ē) = ?`

`n_б(ē) = ?`

Решение

а) В 1 атоме золота `""_79Au` содержится 79ē, поэтому:

`n_а(ē) = 79*n_а(Au) = 79*1 = 79" моль"`

б) В 1 молекуле хлорида натрия `""_11Na" "_17Cl` содержится 11ē+17ē=28ē, поэтому:

`n_б(ē) = 28*n_б(NaCl) = 28*1 = 28" моль"`

Ответ: `n_а(ē) = 79" моль"`, `n_б(ē) = 28" моль"`.

7. Общее число атомов на Земле примерно равно 6∙1049. Сколько это молей?

Ответ:

Дано:

`N = 6*10^49`

`n = ?`

Решение

`n = N/N_A = (6*10^49)/(6*10^23) = 1*10^26" моль"`

Ответ: `n(Au) = 1*10^26" моль"`.

8. Если взять один моль букв и разместить их в строчку, то чему будет равна её длина в световых годах? Считайте, что каждая буква занимает 1 см строки, скорость света равна 300 тыс. км/с, в одном году около 30 млн. секунд.

Ответ:

Дано:

`n = 1" моль"`

`l = 1" см" = 1*10^(-5)" км"`

`ν = 300000" км/с"`

`"св. год" = 30*10^6 с`

`ly = ?" св. год"`

Решение

`S = l*n*N_a = 1*10^(-5)*1*6*10^23 = 6*10^18" км"`

`t = S/ν = (6*10^18)/300000 = 2*10^13 с`

`ly = (t)/("св. год") = (2*10^13)/(30*10^6) = 666666" св. год"`

Ответ: `ly = 666666" св. год"`.

§2

§2. Молярная масса

1. Дайте определение молярной массы. В каких единицах она выражается?

Ответ:

Молярная масса – это физическая величина, равная отношению массы вещества к его количеству.

Молярная масса выражается в граммах на моль (г/моль).

2. Что общего и в чём различие между молярной массой и относительной молекулярной массой?

Ответ:

Сходство состоит в том, что молярная масса, выраженная в г/моль, численно равна относительной молекулярной массе вещества.

Есть два различия. Во-первых, молярная масса характеризует один моль вещества, тогда как относительная молекулярная масса – одну молекулу. Во-вторых, молярная масса не является безразмерной величиной и, в отличие от относительной молекулярной массы, выражается в г/моль.

3. Какое вещество имеет наименьшую молярную массу?

Ответ:

Наименьшую молярную массу имеет водород.
M(H2) = 2 г/моль

4. Рассчитайте молярные массы следующих веществ по их формулам: a) H2S, NH3, F2, CO2, CaCO3; б) SO2, NaOH, K2SO4, N2, Fe(NO3)3; в) KOH, HNO3, H3PO4, BaSO4, CuSO4∙5H2O.

Ответ:

а)

M(H2S) = 2∙M(H) + M(S) = 2∙1 + 32 = 34 г/моль

M(NH3) = M(N) + 3∙M(H) = 14 + 3∙1 = 17 г/моль

M(F2) = = 2∙M(F) = 2∙19 = 38 г/моль

M(CO2) = M(C) + 2∙M(O) = 12 + 2∙16 = 44 г/моль

M(CaCO3) = M(Ca) + M(C) + 3∙M(O) = 40 + 12 + 3∙16 = 100 г/моль

б)

M(SO2) = M(S) + 2∙M(O) = 32 + 2∙16 = 64 г/моль

M(NaOH) = M(Na) + M(O) + M(H) = 23 + 16 + 1 = 40 г/моль

M(K2SO4) = 2∙M(K) + M(S) + 4∙M(O) = 2∙39 + 32 + 4∙16 = 174 г/моль

M(N2) = 2∙M(N) = 2∙14 = 28 г/моль

M(Fe(NO3)3) = M(Fe) + 3∙M(N) + 9∙M(O) = 56 + 3∙14 + 9∙16 = 242 г/моль

в)

M(KOH) = M(K) + M(O) + M(H) = 39 + 16 + 1 = 56 г/моль

M(HNO3) = M(H) + M(N) + 3∙M(O) = 1 + 14 + 3∙16 = 63 г/моль

M(H3PO4) = 3∙M(H) + M(P) + 4∙M(O) = 3∙1 + 31 + 4∙16 = 98 г/моль

M(BaSO4) = M(Ba) + M(S) + 4∙M(O) = 137 + 32 + 4∙16 = 233 г/моль

M(CuSO4∙5H2O) = M(Cu) + M(S) + 9∙M(O) + 10∙M(H) = 64 + 32 + 9∙16 + 10∙1 = 250 г/моль

5. Молярная масса соединения углерода с водородом равна 16 г/моль. Определите формулу этого соединения.

Ответ:

Дано:

`M(C_xH_y) = 16" г/моль"`

`C_xH_y" – "?`

Решение

`12*x + y = 16`

x и y могут принимать только положительные целочисленные значения. Из уравнения видно, что x не может быть больше 1, тогда:

`12*1 + y = 16`

`y = 4`

Формула соединения `CH_4`.

Ответ: `CH_4`.

6. Рассчитайте количество вещества в: а) 16 г серы; б) 24 г кислорода; в) 49 г серной кислоты; г) 500 г карбоната кальция; д) 10 кг воды.

Ответ:

 Дано:

`"а) "m(S) = 16 г`

`"б) "m(O_2) = 24 г`

`"в) "m(H_2SO_4) = 49 г`

`"г) "m(CaCO_3) = 500 г`

`"д) "m(H_2O) = 10" кг" = 10^4 г`

`"а) "n(S) = ?`

`"б) "n(O_2) = ?`

`"в) "n(H_2SO_4) = ?`

`"г) "n(CaCO_3) = ?`

`"д) "n(H_2O) = ?`

Решение

а)

`M(S) = 32" г/моль"`

`n(S) = (m(S))/(M(S)) = 16/32 = 0.5" моль"`

б)

`M(O_2) = 2*M(O) = 2*16 = 32" г/моль"`

`n(O_2) = (m(O_2))/(M(O_2)) = 24/32 = 0.75" моль"`

в)

`M(H_2SO_4) = 2*M(H) + M(S) + 4*M(O) = 2*1 + 32 + 4*16 = 98" г/моль"`

`n(H_2SO_4) = (m(H_2SO_4))/(M(H_2SO_4)) = 49/98 = 0.5" моль"`

г)

`M(CaCO_3) = M(Ca) + M(C) + 3*M(O) = 40 + 12 + 3*16 = 100" г/моль"`

`n(CaCO_3) = (m(CaCO_3))/(M(CaCO_3)) = 500/100 = 5" моль"`

д)

`M(H_2O) = 2*M(H) + M(O) = 2*1 + 16 = 18" г/моль"`

`n(H_2O) = (m(H_2O))/(M(H_2O)) = (10^4)/18 = 555.6" моль"`

Ответ: `"а) "n(S) = 0.5" моль"`, `"б) "n(O_2) = 0.75" моль"`, `"в) "n(H_2SO_4) = 0.5" моль"`, `"г) "n(CaCO_3) = 5" моль"`, `"д) "n(H_2O) = 555.6" моль"`.

7. Где содержится больше молекул – в 100 г кислорода O2 или в 100 г азота N2? Дайте ответ, не прибегая к расчётам.

Ответ:

Молярная масса кислорода больше, чем у азота, поэтому при одинаковой массе число молей азота будет больше, чем число молей кислорода, а значит и число молекул азота будет больше, чем число молекул кислорода.

8. Где содержится больше атомов – в 100 г кислорода O2 или в 100 г озона O3? Дайте ответ, не прибегая к расчётам.

Ответ:

Молярная масса озона больше, чем у кислорода, поэтому при одинаковой массе число молей кислорода будет больше, чем число молей азота, а значит и число молекул кислорода будет больше, чем число молекул озона.

9. Сколько молекул и атомов содержится в 160 г жидкого кислорода?

Ответ:

Дано:

`m(O_2) = 160 г`

`N(O_2) = ?`

`N(O) = ?`

Решение

`M(O_2) = 2*M(O) = 2*16 = 32" г/моль"`

`n(O_2) = (m(O_2))/(M(O_2)) = 160/32 = 5" моль"`

`n(O) = 2*n(O_2) = 2*5 = 10" моль"`

`N(O_2) = n(O_2)*N_A = 5*6.02*10^23 = 3.01*10^24`

`N(O) = n(O)*N_A = 10*6.02*10^23 = 6.02*10^24`

Ответ: `N(O_2) = 3.01*10^24`, `N(O) = 6.02*10^24`.

10. Образец серной кислоты имеет массу 196 г. Рассчитайте количество вещества: а) серной кислоты; б) атомов серы; в) атомов водорода; г) атомов кислорода в этом образце.

Ответ:

Дано:

`m(H_2SO_4) = 196 г`

`"а) "n(H_2SO_4) = ?`

`"б) "n(S) = ?`

`"в) "n(H) = ?`

`"г) "n(O) = ?`

Решение

`M(H_2SO_4) = 2*M(H) + M(S) + 4*M(O) = 2*1 + 32 + 4*16 = 98" г/моль"`

а)

`n(H_2SO_4) = (m(H_2SO_4))/(M(H_2SO_4)) = 196/98 = 2" моль"`

б)

`n(S) = n(H_2SO_4) = 2" моль"`

в)

`n(H) = 2*n(H_2SO_4) = 2*2 = 4" моль"`

г)

`n(S) = 4*n(H_2SO_4) = 4*2 = 8" моль"`

Ответ: `"а) "n(H_2SO_4) = 2" моль"`, `"б) "n(S) = 2" моль"`, `"в) "n(H) = 4" моль"`, `"г) "n(O) = 8" моль"`.

11. В образце массой 2,8 г содержится 0,05 моль вещества. Найдите молярную массу.

Ответ:

Дано:

`m("в-ва") = 2.8 г`

`n("в-ва") = 0.05" моль"`

`M("в-ва") = ?`

Решение

`M("в-ва") = (m("в-ва"))/(n("в-ва")) = 2.8/0.05 = 56" г/моль"`

Ответ: `M("в-ва") = 56" г/моль"`.

12. В образце оксида азота массой 69 г содержится 1,5 моль вещества. Установите формулу оксида.

Ответ:

Дано:

`m(N_xO_y) = 69 г`

`n(N_xO_y) = 1.5" моль"`

`N_xO_y" – "?`

Решение

`M(N_xO_y) = (m(N_xO_y))/(n(N_xO_y)) = 69/1.5 = 46" г/моль"`

`14*x + 16*y = 46`

Допустим молекула вещества состоит из 1 атома N и 1 атома O, тогда молярная масса этого вещества равна 30 г/моль. Из полученной и необходимой молярной массы видно, что нахватает 16 г/моль, она как раз советует кислороду, поэтому:

`x = 1`, `y = 2`.

Формула вещества `NO_2`.

Ответ: `NO_2`.

13. Используя определения молярной массы, относительной молекулярной массы и атомной единицы массы, докажите, что молярная масса численно равна относительной молекулярной массе.

Ответ:

`M = m_m*N_A`

`m_m = M_r*1" а. е. м."`

`M = M_r*1" а. е. м."*N_A = M_r*1" г/моль"`.

§3

§3. Вывод простейшей формулы вещества

1. Определите формулу оксида серы, который содержит 40% серы по массе.

Ответ:

Дано:

`S_xO_y`

`ω(S) = 40%`

`S_xO_y" – "?`

Решение

`ω(O) = 100 - ω(S) = 100 - 40 = 60%`

`x" : "y = n(S)" : "n(O) = 40/32" : "60/16`

`x" : "y = 1" : "3`

Формула оксида серы: `SO_3`.

Ответ: `SO_3`.

2. Рассчитайте массовые доли водорода в соединениях по их формулам: CH4, NH3, H2O, HF.

Ответ:

Дано:

`CH_4`

`NH_3`

`H_2O`

`HF`

`ω(H) = ?`

Решение

`ω_(CH_4)(H) = (100*z*M(H))/(M(CH_4)) = (100*4*1)/16 = 25%`

`ω_(NH_3)(H) = (100*z*M(H))/(M(NH_3)) = (100*3*1)/17 = 17.6%`

`ω_(H_2O)(H) = (100*z*M(H))/(M(H_2O)) = (100*2*1)/18 = 11.1%`

`ω_(HF)(H) = (100*z*M(H))/(M(HF)) = (100*1*1)/20 = 5%`

Ответ: `ω_(CH_4)(H) = 25%`, `ω_(NH_3)(H) = 17.6%`, `ω_(H_2O)(H) = 11.1%`, `ω_(HF)(H) = 5%`.

3. Установите простейшую формулу оксида серы, в котором масса серы равна массе кислорода.

Ответ:

Дано:

`S_xO_y`

`m(S) = m(O)`

`S_xO_y" – "?`

Решение

Так как `m(S) = m(O)`, то `ω(S) = ω(O) = 50%`.

`x" : "y = n(S)" : "n(O) = 50/32" : "50/16`

`x" : "y = 1" : "2`

Формула оксида серы: `SO_2`.

Ответ: `SO_2`.

4. Установите простейшую формулу соединения водорода с кислородом, содержащего 5,88% водорода по массе. Попробуйте определить молекулярную формулу этого вещества, основываясь на валентностях водорода и кислорода.

Ответ:

Дано:

`H_xO_y`

`ω(H) = 5.88%`

`H_xO_y" – "?`

Решение

`ω(O) = 100 - ω(H) = 100 - 5.88 = 94.12%`

`x" : "y = n(H)" : "n(O) = 5.88/1" : "94.12/16`

`x" : "y = 1" : "1`

Так как соединения с формулой `HO` не существует, то значит это пероксид водорода `H_2O_2`.

Ответ: `H_2O_2`.

5. Найдите простейшие формулы соединений, имеющих следующий массовый состав: а) 80% Cu, 20% O; б) 40% Cu, 40% O, 20% S; в) 35% N, 5% H, 60% O.

Ответ:

Дано:

а)

`ω(Cu) = 80%`

`ω(O) = 20%`

б)

`ω(Cu) = 40%`

`ω(S) = 20%`

`ω(O) = 40%`

в)

`ω(H) = 5%`

`ω(N) = 35%`

`ω(O) = 60%`

`"а) "Cu_xO_y" – "?`

`"б) "Cu_xS_yO_z" – "?`

`"в) "H_xN_yO_z" – "?`

Решение

а)

`x" : "y = n(Cu)" : "n(O) = 80/64" : "20/16`

`x" : "y = 1" : "1`

Формула вещества `CuO`.

б)

`x" : "y" : "z = n(Cu)" : "n(S)" : "n(O) = 40/64" : "20/32" : "40/16`

`x" : "y" : "z = n(Cu)" : "n(S)" : "n(O) = 10/16" : "10/16" : "40/16`

`x" : "y" : "z = 1" : "1" : "4`

Формула вещества `CuSO_4`.

в)

`x" : "y" : "z = n(H)" : "n(N)" : "n(O) = 5/1" : "35/14" : "60/16`

`x" : "y" : "z = n(H)" : "n(N)" : "n(O) = 5" : "2.5" : "3.75`

`x" : "y" : "z = 2" : "1" : "1.5`

`x" : "y" : "z = 4" : "2" : "3`

Формула вещества `H_4N_2O_3` или `NH_4NO_3`.

Ответ: `"а) "CuO`, `"б) "CuSO_4`, `"в) "NH_4NO_3`.

6. Оксид трёхвалентного элемента содержит 47,1% кислорода по массе. Установите формулу оксида.

Ответ:

Дано:

`"Э"_2O_3`

`ω(O) = 47.1%`

`"Э"_2O_3" – "?`

Решение

`ω("Э") = 100 - ω(O) = 100 - 47.1 = 52.9%`

`x" : "y = n("Э")" : "n(O) = 52.9/(M("Э"))" : "47.1/16`

`2" : "3 = 52.9/(M("Э"))" : "47.1/16`

`2/3 = (16*52.9)/(47.1*M("Э"))`

`2/3*47.1*M("Э") = 846.4`

`M("Э") = 27" г/моль"`

Молярная масса элемента соответствует алюминию, значит формула оксида `Al_2O_3`.

Ответ: `Al_2O_3`.

7. Приведите примеры двух веществ, простейшие формулы которых не совпадают с молекулярными.

Ответ:

Название вещества Молекулярная формула Простейшая формула
Перекись водорода H2O2 HO
Глюкоза C6H12O6 CH2O
Бензол С6H6 CH

§4

§4. Расчёты по уравнениям реакций

1. Проанализируйте уравнение реакции 2H2S + 3O2 = 2H2O + 2SO2 подобно тому, как это сделано на с. 19-20 для реакции горения метана.

Ответ:

2H2S + 3O2 = 2H2O + 2SO2

Два моль сероводорода реагирует с тремя молями кислорода с образованием двух моль воды и двух моль оксида серы (IV). Если в реакцию вступает x моль H2S, то при этом расходуется 1.5x моль O2 и образуется x моль H2O и x моль SO2:
n(H2S) : n(O2) : n(H2O) : n(SO2) = x : 1.5x : x : x = 1 : 1.5 : 1 : 1.

2. В каком массовом соотношении необходимо смешать железо с серой для получения сульфида железа FeS?

Ответ:

 Дано:

Fe

S                         

`m(Fe)" : "m(S) = ?`

Решение

`Fe + S = FeS`

`m(Fe)" : "m(S) = M(Fe)" : "M(S) = 56" : "32`

`m(Fe)" : "m(S) = 56" : "32`

`m(Fe)" : "m(S) = 7" : "4`

Ответ: `m(Fe)" : "m(S) = 7" : "4`.

3. Сколько молей оксида фосфора (V) P2O5 образуется при сгорании 2 моль фосфора? Сколько молей кислорода расходуется при этом?

Ответ:

4P + 5O2 ⟶ 2P2O5

При сгорании 2 моль фосфора образуется 1 моль P2O5, при этом расходуется 2,5 моль O2.

4. Определите количество вещества хлорида натрия, который образуется при взаимодействии соляной кислоты с 0,5 моль: а) гидроксида натрия; б) оксида натрия.

Ответ:

а) NaOH + HCl ⟶ NaCl + H2O

При взаимодействии соляной кислоты с 0,5 моль гидроксида натрия образуется 0,5 моль хлорида натрия.

б) Na2O + 2HCl ⟶ 2NaCl + H2O

При взаимодействии соляной кислоты с 0,5 моль оксида натрия образуется 1 моль хлорида натрия.

5. Составьте уравнение реакции сгорания аммиака NH3 в кислороде с образованием азота и воды. Сколько молей кислорода необходимо для сжигания 24 моль аммиака? Сколько молей каждого из продуктов реакции при этом образуется?

Ответ:

Дано:

`n(NH_3) = 24" моль"`
-------------------------------

`n(O_2) = ?`

`n(N_2) = ?`

`n(H_2O) = ?`

Решение

`4NH_3 + 3O_2 ⟶ 2N_2 + 6H_2O`

`n(O_2) = (3*n(NH_3))/4 = (3*24)/4 = 18" моль"`

`n(N_2) = (2*n(NH_3))/4 = (2*24)/4 = 12" моль"`

`n(H_2O) = (6*n(NH_3))/4 = (6*24)/4 = 36" моль"`

Ответ: `n(O_2) = 18" моль"`, `n(N_2) = 12" моль"`, `n(H_2O) = 36" моль"`.

6. Сколько граммов угля сгорело, если при этом образовалось 22 г углекислого газа?

Ответ:

Дано:

`m(CO_2) = 22 г`

`m(C) = ?`

Решение

`C + O_2 ⟶ CO_2`

`n(CO_2) = (m(CO_2))/(M(CO_2)) = 22/44 = 0.5" моль"`

`n(C) = n(CO_2) = 0.5" моль"`

`m(C) = n(C)*M(C) = 0.5*12 = 6 г`

Ответ: `m(C) = 6 г`.

7. Сколько граммов серной кислоты и хлорида бария требуется для получения 4,66 г сульфата бария?

Ответ:

Дано:

`m(BaSO_4) = 4.66 г`

--------------------------

`m(H_2SO_4) = ?`

`m(BaCl_2) = ?`

Решение

`H_2SO_4 + BaCl_2 ⟶ 2HCl + BaSO_4"↓"`

`n(BaSO_4) = (m(BaSO_4))/(M(BaSO_4)) = 4.66/233 = 0.02" моль"`

`n(H_2SO_4) = n(BaCl_2) = n(BaSO_4) = 0.02" моль"`

`m(H_2SO_4) = n(H_2SO_4)*M(H_2SO_4) = 0.02*98 = 1.96 г`

`m(BaCl_2) = n(BaCl_2)*M(BaCl_2) = 0.02*208 = 4.16 г`

Ответ: `m(H_2SO_4) = 1.96 г`, `m(BaCl_2) = 4.16 г`.

8. Сколько граммов оксида кальция необходимо взять для получения 18,5 г гашёной извести (гидроксида кальция)?

Ответ:

Дано:

`m(Ca(OH)_2) = 18.5 г`

--------------------------------

`m(CaO) = ?`

Решение

`CaO + H_2O ⟶ Ca(OH)_2`

`n(Ca(OH)_2) = (m(Ca(OH)_2))/(M(Ca(OH)_2)) = 18.5/74 = 0.25" моль"`

`n(CaO) = n(Ca(OH)_2) = 0.25" моль"`

`m(CaO) = n(CaO)*M(CaO) = 0.25*56 = 14 г`

Ответ: `m(CaO) = 14 г`.

9. Определите массу соды (карбоната натрия), необходимую для реакции с 4,9 г серной кислоты.

Ответ:

Дано:

`m(H_2SO_4) = 4.9 г`

---------------------------

`m(Na_2CO_3) = ?`

Решение

`Na_2CO_3 + H_2SO_4 ⟶ Na_2SO_4 + H_2O + CO_2"↑"`

`n(H_2SO_4) = (m(H_2SO_4))/(M(H_2SO_4)) = 4.9/98 = 0.05" моль"`

`n(Na_2CO_3) = n(H_2SO_4) = 0.05" моль"`

`m(Na_2CO_3) = n(Na_2CO_3)*M(Na_2CO_3) = 0.05*106 = 5.3 г`

Ответ: `m(Na_2CO_3) = 5.3 г`.

10. Образцы магния массой по 36 г вступили в следующие реакции:
а) 2Mg + O2 = 2MgO;
б) Mg + 2HCl = MgCl2 + H2↑;
в) Mg + H2SO4 = MgSO4 + H2↑.

Рассчитайте массы образовавшихся соединений магния.

Ответ:

Дано:

`m(Mg) = 36 г`

--------------------

`"а) "m(MgO) = ?`

`"б) "m(MgCl_2) = ?`

`"в) "m(MgSO_4) = ?`

Решение

`n(Mg) = (m(Mg))/(M(Mg)) = 36/24 = 1.5" моль"`

а)

`2Mg + O_2 = 2MgO`

`n(MgO) = n(Mg) = 1.5" моль"`

`m(MgO) = n(MgO)*M(MgO) = 1.5*40 = 60 г`

б)

`Mg + 2HCl = MgCl_2 + H_2"↑"`

`n(MgCl_2) = n(Mg) = 1.5" моль"`

`m(MgCl_2) = n(MgCl_2)*M(MgCl_2) = 1.5*95 = 142.5 г`

в)

`Mg + H_2SO_4 = MgSO_4 + H_2"↑"`

`n(MgSO_4) = n(Mg) = 1.5" моль"`

`m(MgSO_4) = n(MgSO_4)*M(MgSO_4) = 1.5*120 = 180 г`

Ответ: `"а) "m(MgO) = 16 г`, `"б) "m(MgCl_2) = 142.5 г`, `"в) "m(MgSO_4) = 180 г`.

11. Составьте уравнение реакции магния с серой. В каком соотношении по массе надо взять эти вещества, чтобы они полностью прореагировали друг с другом?

Ответ:

Дано:

Mg

S                               

`m(Mg)" : "m(S) = ?`

Решение

`Mg + S = MgS`

`m(Mg)" : "m(S) = M(Mg)" : "M(S) = 24" : "32`

`m(Mg)" : "m(S) = 24" : "32`

`m(Mg)" : "m(S) = 3" : "4`

Ответ: `m(Mg)" : "m(S) = 3" : "4`.

12. Рассчитайте массу сульфата цинка, образующегося при растворении оксида цинка в 200 г 24,5%-й серной кислоты.

Ответ:

Дано:

`m("р-ра") = 200 г`

`ω(H_2SO_4) = 24.5%`

-----------------------

`m(ZnSO_4) = ?`

Решение

`ZnO + H_2SO_4 ⟶ ZnSO_4 + H_2O`

`m(H_2SO_4) = (ω(H_2SO_4)*m("р-ра"))/100 = (24.5*200)/100 = 49 г`

`n(H_2SO_4) = (m(H_2SO_4))/(M(H_2SO_4)) = 49/98 = 0.5" моль"`

`n(ZnSO_4) = n(H_2SO_4) = 0.5" моль"`

`m(ZnSO_4) = n(ZnSO_4)*M(ZnSO_4) = 0.5*161 = 80.5 г`

Ответ: `m(ZnSO_4) = 80.5 г`.

13. Сколько граммов карбоната натрия может быть получено при пропускании углекислого газа через 200 г 10%-го раствора гидроксида натрия?

Ответ:

Дано:

`m("р-ра") = 200 г`

`ω(NaOH) = 10%`

-------------------------

`m(Na_2CO_3) = ?`

Решение

`2NaOH + CO_2 ⟶ Na_2CO_3 + H_2O`

`m(NaOH) = (ω(NaOH)*m("р-ра"))/100 = (10*200)/100 = 20 г`

`n(NaOH) = (m(NaOH))/(M(NaOH)) = 20/40 = 0.5" моль"`

`n(Na_2CO_3) = (n(NaOH))/2 = 0.5/2 = 0.25" моль"`

`m(Na_2CO_3) = n(Na_2CO_3)*M(Na_2CO_3) = 0.25*106 = 26.5 г`

Ответ: `m(Na_2CO_3) = 26.5 г`.

§5

§5. Закон Авогадро. Молярный объём газов

1. Сформулируйте закон Авогадро.

Ответ:

Закон Авогадро: в равных объёмах любых газов, находящихся при одних и тех же условиях (температура и давление), содержится одинаковое число молекул.

2. Какие условия называют нормальными? Чему равен молярный объём газа: а) при нормальных условиях; б) при 25°C и 1 атм?

Ответ:

Атмосферное давление равное 760 мм. рт. ст. (или 101,3 кПа) и температура равная 0°C – нормальные условия.

а) При нормальных условиях молярный объём любого газа равен 22,4 л/моль.

б) При 25°C и 1 атм молярный объём любого газа равен 24,4 л/моль.

3. Какой объём занимают при н. у.: а) 2 моль азота; б) 0,5 моль кислорода; в) 0,25 моль фтора?

Ответ:

Дано:

`"а) "n(N_2) = 2" моль"`

`"б) "n(O_2) = 0.5" моль"`

`"в) "n(F_2) = 0.25" моль"`

------------------------------

`"а) "V(N_2) = ?`

`"б) "V(O_2) = ?`

`"в) "V(F_2) = ?`

Решение

а)

`V(N_2) = n(N_2)*V_m = 2*22.4 = 44.8 л`

б)

`V(O_2) = n(O_2)*V_m = 0.5*22.4 = 11.2 л`

в)

`V(F_2) = n(F_2)*V_m = 0.25*22.4 = 5.6 л`

Ответ: `"а) "V(N_2) = 44.8 л`, `"б) "V(O_2) = 11.2 л`, `"в) "V(F_2) = 5.6 л`.

4. Сколько молекул находится внутри резинового шара объёмом 5,6 л (н. у.), заполненного водородом? Изменится ли ответ, если в условии задачи водород заменить на: а) азот; б) неизвестный газ; в) водопроводную воду?

Ответ:

Дано:

`V(H_2) = 5.6 л`

----------------------

`N(H_2) = ?`

Решение

`n(H_2) = (V(H_2))/V_m = 5.6/22.4 = 0.25" моль"`

`N(H_2) = n(H_2)*N_A = 0.25*6*10^23 = 1.5*10^23`

Ответ: `N(H_2) = 1.5*10^23`.

а и б) При нормальных условиях молярный объём любого газа равен 22,4 л/моль, поэтому ответ не изменится.

в) Водопроводная вода при н. у. является жидкостью, поэтому ответ изменился бы.

5. Объясните, как с помощью закона Авогадро можно установить, что формула воды H2O, а не HO.

Ответ:

Необходимо заполнить пробирку методом вытеснения воды смесью водорода и кислорода с объёмным соотношением 2:1. Просунуть в пробирку электроподжиг, и поджечь смесь газов, после реакции вода полностью заполнит пробирку, что подтверждает формулу воды H2O.

Если смесь водорода и кислорода будет иметь объёмное соотношение 1:1, то после реакции вода не полностью заполнит пробирку, что опровергает формулу воды HO.

6. В каком объёме аммиака содержится в 3 раза больше молекул, чем в 100 л метана? Газы находятся при одинаковых температуре и давлении.

Ответ:

В 300 л аммиака содержится в 3 раза больше молекул, чем в 100 л метана, так как в равных объёмах любых газов, находящихся при одних и тех же условиях, содержится одинаковое число молекул.

7. Имеются два газа, взятые при одинаковых условиях: 10 л метана CH4 и 20 л хлора Cl2. В каком из них содержится больше молекул, а в каком – больше атомов и во сколько раз?

Ответ:

В 20 л Cl2 содержится в 2 раза больше молекул, чем в 10 л CH4.

1 молекула метана CH4 состоит из 5 атомов, а 1 молекула хлора Cl2 из 2 атомов. Значит в 10 л CH4 содержится в 1,25 раза больше атомов, чем в 20 л Cl2.

8. Где содержится больше молекул аммиака – в 100 г или в 100 л (н. у.)?

Ответ:

Дано:

`m_1(NH_3) = 100 г`

`V_2(NH_3) = 100 л`

------------------------------

`N_1(NH_3)" "?" "N_2(NH_3)`

Решение

`n_1(NH_3) = (m(NH_3))/M((NH_3)) = 100/17 = 5.88" моль"`

`n_2(NH_3) = (V(NH_3))/V_m = 100/22.4 = 4.46" моль"`

Так как `n_1(NH_3)" ">" "n_2(NH_3)`, то:

`N_1(NH_3)" ">" "N_2(NH_3)`

Ответ: больше молекул аммиака содержится в 100 г аммиака.

9. Сколько молекул кислорода находится при нормальных условиях: а) в одном кубометре; б) в одном килограмме этого вещества?

Ответ:

Дано:

`"а) "V(O_2) = 1 м^3 = 1000 л`

`"б) "m(O_2) = 1" кг" = 1000 г`

---------------------------------------

`"а) "N(O_2) = ?`

`"б) "N(O_2) = ?`

Решение

а)

`n(O_2) = (V(NH_3))/V_m = 1000/22.4 = 44.64" моль"`

`N(O_2) = n(O_2)*N_A = 44.64*6.02*10^23 = 2.69*10^25`

б)

`n(O_2) = (m(O_2))/(M(O_2)) = 1000/32 = 31.25" моль"`

`N(O_2) = n(O_2)*N_A = 31.25*6.02*10^23 = 1.88*10^25`

Ответ: `"а) "N(O_2) = 2.69*10^25`, `"б) "N(O_2) = 1.88*10^25`.

10. Какой объём занимают при нормальных условиях: а) 10 г водорода; б) 10 моль водорода?

Ответ:

Дано:

`"а) "m(H_2) = 10 г`

`"б) "n(H_2) = 10" моль"`

--------------------------------

`"а) "V(H_2) = ?`

`"б) "V(H_2) = ?`

Решение

а)

`n(H_2) = (m(H_2))/(M(H_2)) = 10/2 = 5" моль"`

`V(H_2) = n(H_2)*V_m = 5*22.4 = 112 л`

б)

`V(H_2) = n(H_2)*V_m = 10*22.4 = 224 л`

Ответ: `"а) "V(H_2) = 112 л`, `"б) "V(H_2) = 224 л`.

11. При растворении 300 л хлороводорода (н. у.) в 1 л воды образуется раствор объёмом 1,28 л. Рассчитайте массовую долю хлороводорода в полученной соляной кислоте и плотность раствора.

Ответ:

Дано:

`V(HCl) = 300 л`

`V(H_2O) = 1 л = 1000" мл"`

`V("р-ра") = 1.28 л = 1280" мл"`

--------------------------------

`ω(HCl) = ?`

`ρ("р-ра") = ?`

Решение

`n(HCl) = (V(HCl))/V_m = 300/22.4 = 13.4" моль"`

`m(HCl) = n(HCl)*M(HCl) = 13.4*36.5 = 489 г`

`m(H_2O) = ρ(H_2O)*V(H_2O) = 1*1000 = 1000 г`

`m("р-ра") = m(H_2O) + m(HCl) = 1000 + 489 = 1489 г`

`ω(HCl) = (100*m(HCl))/(m("р-ра")) = (100*489)/1489 = 32.8%`

`ρ("р-ра") = (m("р-ра"))(V("р-ра")) = 1489/1280 = 1.16" г/мл"`

Ответ: `ω(HCl) = 32.8%`, `ρ("р-ра") = 1.16" г/мл"`.

§6

§6. Относительная плотность газов

1. Во сколько раз кислород тяжелее: а) водорода; б) воздуха? Газы находятся при одинаковых температуре и давлении.

Ответ:

Дано:

`O_2`

----------------

`"а) "D_(H_2)(O_2) = ?`

`"б) "D_("возд.")(O_2) = ?`

Решение

а)

`D_(H_2)(O_2) = (M(O_2))/(M(H_2)) = 32/2 = 16`

б)

`D_("возд.")(O_2) = (M(O_2))/(M("возд.")) = 32/29 = 1.1`

Ответ: а) кислород тяжелее водорода в 16 раз; б) кислород тяжелее воздуха в 1.1 раза.

2. Найдите относительную плотность гелия и неона: а) по водороду; б) по воздуху.

Ответ:

Дано:

`He`

`Ne`

------------

а)

`D_(H_2)(He) = ?`

`D_(H_2)(Ne) = ?`

б)

`D_("возд.")(He) = ?`

`D_("возд.")(Ne) = ?`

Решение

а)

`D_(H_2)(He) = (M(He))/(M(H_2)) = 4/2 = 2`

`D_(H_2)(Ne) = (M(Ne))/(M(H_2)) = 20/2 = 10`

б)

`D_("возд.")(He) = (M(He))/(M("возд.")) = 4/29 = 0.14`

`D_("возд.")(Ne) = (M(Ne))/(M("возд.")) = 20/29 = 0.69`

Ответ: а) `D_(H_2)(He) = 2`, `D_(H_2)(Ne) = 10`; б) `D_("возд.")(He) = 0.14`, `D_("возд.")(Ne) = 0.69`.

3. Объясните, почему относительные плотности всех газов по водороду больше 1.

Ответ:

Водород является самым лёгким газом, поэтому относительные плотности всех газов по водороду больше 1.

4. Найдите относительную плотность по гелию следующих газов: H2, CH4, N2, O2, SO2.

Ответ:

Дано:

`H_2`

`CH_4`

`N_2`

`O_2`

`SO_2`

`D_(He)(H_2) = ?`

`D_(He)(CH_4) = ?`

`D_(He)(N_2) = ?`

`D_(He)(O_2) = ?`

`D_(He)(SO_2) = ?`

Решение

`D_(He)(H_2) = (M(H_2))/(M(He)) = 2/4 = 0.5`

`D_(He)(CH_4) = (M(CH_4))/(M(He)) = 16/4 = 4`

`D_(He)(N_2) = (M(N_2))/(M(He)) = 28/4 = 7`

`D_(He)(O_2) = (M(O_2))/(M(He)) = 32/4 = 8`

`D_(He)(SO_2) = (M(SO_2))/(M(He)) = 64/4 = 16`

Ответ: `D_(He)(H_2) = 0.5`, `D_(He)(CH_4) = 4`, `D_(He)(N_2) = 7`, `D_(He)(O_2) = 8`, `D_(He)(SO_2) = 16`.

5. Из таблицы 2 выпишите газы, которые: а) легче воздуха; б) тяжелее воздуха. Какой газ из приведённых в ней самый лёгкий; самый тяжёлый?

Ответ:

а) H2, N2, CH4, NH3.

б) O2, F2, Cl2, Br2 (пары), CO2.

Из приведённых газов самый лёгкий – H2, самый тяжёлый – Br2 (пары).

6. Плотность некоторого газа по кислороду равна 2. Чему равна плотность этого газа по водороду?

Ответ:

Дано:

`D_(O_2)(X) = 2`

`D_(H_2)(X) = ?`

Решение

`M(X) = M(O_2)*D_(O_2)(X) = 32*2 = 64`

`D_(H_2)(X) = (M(X))/(M(H_2)) = 64/2 = 32`

Ответ: `D_(H_2)(X) = 32`.

7. Какой газ тяжелее азота, но легче кислорода?

Ответ:

Этан C2H6 тяжелее азота, но легче кислорода.

Воздух (но это смесь газов) тяжелее азота, но легче кислорода.

8. Приведите формулы пяти газов, которые легче воздуха.

Ответ:

H2, He, Ne, N2, CH4, NH3.

9. Оксид углерода и оксид азота имеют одинаковую плотность при одних и тех же условиях. Определите формулы оксидов.

Ответ:

Если оксид углерода и оксид азота имеют одинаковую плотность, то значит они имеют одинаковые молярные массы.

Существуют два оксида углерода CO и CO2, их молярные массы соответственно 28 г/моль и 44 г/моль.

Существуют пять оксидов азота N2O (44 г/моль), NO (30 г/моль), N2O3 (76 г/моль), NO2 (46 г/моль) и N2O5 (108 г/моль).

Одинаковую плотность имеют оксид углерода (IV) CO2 и оксид азота (I) N2O.

10. Углерод образует два газообразных оксида. Чему равна плотность более тяжёлого оксида по более лёгкому?

Ответ:

`D_(CO)(CO_2) = (M(CO_2))/(M(CO)) = 44/28 = 1.57`

11. Самый тяжёлый газ при комнатной температуре – гексафторид теллура TeF6. Чему равна его относительная плотность: а) по воздуху; б) по водороду?

Ответ:

`"а) "D_("возд.")(TeF_6) = (M(TeF_6))/(M("возд.")) = 242/29 = 8.35`

`"б) "D_(H_2)(TeF_6) = (M(TeF_6))/(M(H_2)) = 242/2 = 121`

12. Трёхвалентный элемент образует газообразное соединение с водородом, которое в 2 раза тяжелее аммиака. Какова формула соединения?

Ответ:

Молярная масса аммиака 17 г/моль, значит молярная масса соединения равна 31 г/моль. Если в газообразном соединении с водородом элемент является трёхвалентным, то он находится в 3 группе. Посчитаем молярную массу водородного соединения элемента находящегося под азотом – PH3 – его молярная масса равна 34 г/моль, что ровно в 2 раза тяжелее аммиака.

Формула соединения PH3.

13. Хлор образует газообразные оксиды, в которых его валентность равна I и IV. Какой из оксидов легче хлора, а какой тяжелее?

Ответ:

`D_(Cl_2)(Cl_2O) = (M(Cl_2O))/(M(Cl_2)) = 87/71 = 1.23`

`D_(Cl_2)(ClO_2) = (M(ClO_2))/(M(Cl_2)) = 67.5/71 = 0.95`

Оксид хлора (IV) легче хлора, а оксид хлора (I) – тяжелее.

§7

§7. Расчёты по уравнениям химических реакций с участием газов

1. При разложении воды электрическим током получили 30 л водорода. Какой объём кислорода образовался при этом?

Ответ:

Дано:

`V(H_2) = 30 л`

`V(O_2) = ?`

Решение

`2H_2O overset("электролиз")(⟶) 2H_2"↑" + O_2"↑"`

`V(O_2) = (V(H_2))/2 = 30/2 = 15 л`

Ответ: `V(O_2) = 15 л`.

2. При реакции аммиака с кислородом в присутствии катализатора образуются оксид азота (II) и вода. Рассчитайте, какой объём кислорода необходим для окисления 100 м3 аммиака. Какой объём оксида азота (II) при этом образуется?

Ответ:

Дано:

`V(NH_3) = 100 м^3`

`V(O_2) = ?`

`V(NO_2) = ?`

Решение

`4NH_3 + 5O_2 ⟶ 4NO_2 + 6H_2O`

`V(O_2) = (5*V(NH_3))/4 = (5*100)/4 = 125 м^3`

`V(NO_2) = V(NH_3) = 100 м^3`

 Ответ: `V(O_2) = 125 м^3`, `V(NO_2) = 100 м^3`.

3. Сгорело 6 кг углерода. Рассчитайте массу и объём (при н. у.) образовавшегося углекислого газа.

Ответ:

Дано:

`m(C) = 6" кг"`

`m(CO_2) = ?`

`V(CO_2) = ?`

Решение

`C + O_2 ⟶ CO_2`

`n(C) = (m(C))/(M(C)) = 6/12 = 0.5" кмоль"`

`n(CO_2) = n(C) = 0.5" кмоль"`

`m(CO_2) = n(CO_2)*M(CO_2) = 0.5*44 = 22" кг"`

`V(CO_2) = n(CO_2)*V_m = 0.5*22.4 = 11.2 м^3`

Ответ: `m(CO_2) = 22" кг"`, `V(CO_2) = 11.2 м^3`.

4. Сколько граммов угля сгорело, если при этом образовалось 67,2 л (н. у.) углекислого газа?

Ответ:

Дано:

`V(CO_2) = 67.2 л`

`m(C) = ?`

Решение

`C + O_2 ⟶ CO_2`

`n(CO_2) = (V(CO_2))/V_m = 67.2/22.4 = 3" моль"`

`n(C) = n(CO_2) = 3" моль"`

`m(C) = n(C)*M(C) = 3*12 = 36 г`

Ответ: `m(C) = 36 г`.

5. Определите массу оксида кальция и объём оксида углерода (IV) (н. у.), которые образуются при прокаливании 1000 г карбоната кальция.

Ответ:

Дано:

`m(CaCO_3) = 1000 г`

`m(CaO) = ?`

`V(CO_2) = ?`

Решение

`CaCO_3 overset(t)(⟶) CaO + CO_2"↑"`

`n(CaCO_3) = (m(CaCO_3))/(M(CaCO_3)) = 1000/100 = 10" моль"`

`n(CaO) = n(CO_2) = n(CaCO_3) = 10" моль"`

`m(CaO) = n(CaO)*M(CaO) = 10*56 = 560 г`

`V(CO_2) = n(CO_2)*V_m = 10*22.4 = 224 л`

Ответ: `m(CaO) = 560 г`, `V(CO_2) = 224 л`.

6. Определите массу хлора, прореагировавшего с 10 г водорода. Какой объём хлороводорода (н. у.) при этом образовался?

Ответ:

Дано:

`m(H_2) = 10 г`

`m(Cl_2) = ?`

`V(HCl) = ?`

Решение

`H_2 + Cl_2 ⟶ 2HCl`

`n(H_2) = (m(H_2))/(M(H_2)) = 10/2 = 5" моль"`

`n(Cl_2) = n(H_2) = 5" моль"`

`m(Cl_2) = n(Cl_2)*M(Cl_2) = 5*71 = 355 г`

`n(HCl) = 2*n(H_2) = 2*5 = 10" моль"`

`V(HCl) = n(HCl)*V_m = 10*22.4 = 224 л`

Ответ: `m(Cl_2) = 355 г`, `V(HCl) = 224 л`.

7. Какой объём углекислого газа (н. у.) требуется для получения 10,6 г карбоната натрия?

Ответ:

Дано:

`m(Na_2CO_3) = 10.6 г`

`V(CO_2) = ?`

Решение

`2NaOH + CO_2 ⟶ Na_2CO_3 + H_2O`

`n(Na_2CO_3) = (m(Na_2CO_3))/(M(Na_2CO_3)) = 10.6/106 = 0.1" моль"`

`n(CO_2) = n(Na_2CO_3) = 0.1" моль"`

`V(CO_2) = n(CO_2)*V_m = 0.1*22.4 = 2.24 л`

Ответ: `V(CO_2) = 2.24 л`.

8. При взаимодействии натрия с водой образовалось 560 мл (н. у.) водорода. Рассчитайте массу натрия.

Ответ:

Дано:

`V(H_2) = 560" мл" = 0.56 л`

`m(Na) = ?`

Решение

`2Na + 2H_2O ⟶ 2NaOH + H_2"↑"`

`n(H_2) = (V(H_2))/V_m = 0.56/22.4 = 0.025" моль"`

`n(Na) = 2*n(H_2) = 2*0.025 = 0.05" моль"`

`m(Na) = n(Na)*M(Na) = 0.05*23 = 1.15 г`

Ответ: `m(Na) = 1.15 г`.

9. Какой объём кислорода (н. у.) образуется при полном разложении 120 г пероксида водорода?

Ответ:

Дано:

`m(H_2O_2) = 120 г`

`V(O_2) = ?`

Решение

`2H_2O_2 overset(MnO_2)(⟶) 2H_2O + O_2"↑"`

`n(H_2O_2) = (m(H_2O_2))/(M(H_2O_2)) = 120/34 = 3.53" моль"`

`n(O_2) = (n(H_2O_2))/2 = 3.53/2 = 1.77" моль"`

`V(CO_2) = n(CO_2)*V_m = 1.77*22.4 = 39.65 л`

Ответ: `V(CO_2) = 39.65 л`.

10. Водород получают действием металлов на кислоты. Определите массы магния и цинка, которые надо взять, чтобы при взаимодействии с соляной кислотой получить 5,6 л водорода (н. у.).

Ответ:

Дано:

`V(H_2) = 5.6 л`

`m(Mg) = ?`

`m(Zn) = ?`

Решение

`Mg + 2HCl ⟶ MgCl_2 + H_2"↑"`

`Zn + 2HCl ⟶ ZnCl_2 + H_2"↑"`

`n(H_2) = (V(H_2))/V_m = 5.6/22.4 = 0.25" моль"`

`n(Mg) = n(Zn) = n(H_2) = 0.25" моль"`

`m(Mg) = n(Mg)*M(Mg) = 0.25*24 = 6 г`

`m(Zn) = n(Zn)*M(Zn) = 0.25*65 = 16.25 г`

Ответ: `m(Mg) = 6 г`, `m(Zn) = 16.25 г`.

11. Кислород можно получить разными способами. Определите массы бертолетовой соли, пероксида водорода и перманганата калия, необходимые для получения 6,72 л (н. у.) кислорода.

Ответ:

Дано:

`V(O_2) = 6.72 л`

`m(KClO_3) = ?`

`m(H_2O_2) = ?`

`m(KMnO_4) = ?`

Решение

`n(O_2) = (V(O_2))/V_m = 6.72/22.4 = 0.3" моль"`

`2KClO_3 overset(t)(⟶) 2KCl + 3O_2"↑"`
`n(KClO_3) = (2*n(O_2))/3 = (2*0.3)/3 = 0.2" моль"`
`m(KClO_3) = n(KClO_3)*M(KClO_3) = 0.2*122.5 = 24.5 г`

`2H_2O_2 overset(MnO_2)(⟶) 2H_2O + O_2"↑"`
`n(H_2O_2) = 2*n(O_2) = 2*0.3 = 0.6" моль"`
`m(H_2O_2) = n(H_2O_2)*M(H_2O_2) = 0.6*34 = 20.4 г`

`2KMnO_4 overset(t)(⟶) K_2MnO_4 + MnO_2 + O_2"↑"`
`n(KMnO_4) = 2*n(O_2) = 2*0.3 = 0.6" моль"`
`m(KMnO_4) = n(KMnO_4)*M(KMnO_4) = 0.6*158 = 94.8 г`

Ответ: `m(KClO_3) = 24.5 г`, `m(H_2O_2) = 20.4 г`, `m(KMnO_4) = 94.8 г`.

12. При полном разложении газообразного оксида азота образовалось 20 л азота и 10 л кислорода. Установите формулу оксида.

Ответ:

Формула оксида: N2O.

Так как объёмы газов прямо пропорциональны их количествам.

13. Имеется смесь CO и O2 в соотношении 2 : 1. Смесь подожгли. Во сколько раз уменьшится объём газа (при постоянных температуре и давлении) после реакции?

Ответ:

Дано:

`V(CO)" : "V(O_2) = 2" : "1`

`(V_"исх.")/(V(CO_2)) = ?`

Решение

`2CO + O_2 ⟶ 2CO_2`

Реагирующие вещества прореагируют полностью (исходя из соотношения).

Допустим `V(O_2) = x`, тогда:

`V_"исх." = V(CO) + V(O_2) = 2x + 1x = 3x`

`V(CO_2) = 2x`

`(V_"исх.")/(V(CO_2)) = (3x)/(2x) = 1.5`

Ответ: объём газа уменьшится в 1.5 раза.

14. Напишите уравнения реакций, в которых два газообразных при обычных условиях вещества реагируют друг с другом в объёмном соотношении: а) 2 : 1; б) 2 : 3; в) 4 : 3; г) 2 : 5; д) 1 : 3.

Ответ:

а) 2CO + O2 ⟶ 2CO2

б) 2H2S + 3O2 ⟶ 2SO2 + 2H2O

в) 4NH3 + 3O2 ⟶ 2N2 + 6H2O

г) 2C2H2 + 5O2 ⟶ 4CO2 + 2H2O

д) N2 + 3H2   t, кат. ⟶ 2NH3

§8

§8. Более сложные расчёты по уравнениям реакций

1. Определите массу воды, которая образуется при взрыве смеси, состоящей из 2,8 л водорода и 1,5 л кислорода (н. у.).

Ответ:

Дано:

`V(H_2) = 2.8 л`

`V(O_2) = 1.5 л`

`m(H_2O) = ?`

Решение

`2H_2 + O_2 ⟶ 2H_2O`

Кислород в избытке, значит расчёт ведём по водороду.

`n(H_2) = (V(H_2))/V_m = 2.8/22.4 = 0.125" моль"`

`n(H_2O) = n(H_2) = 0.125" моль"`

`m(H_2O) = n(H_2O)*M(H_2O) = 0.125*18 = 2.25 г`

Ответ: `m(H_2O) = 2.25 г`.

2. Определите массу сульфида железа, образующегося при сплавлении 6,4 г серы и 11,8 г порошка железа.

Ответ:

Дано:

`m(S) = 6.4 г`

`m(Fe) = 11.8 г`

`m(FeS) = ?`

Решение

`Fe + S ⟶ FeS`

`n(Fe) = (m(Fe))/(M(Fe)) = 11.8/56 = 0.21" моль"`

`n(S) = (m(S))/(M(S)) = 6.4/32 = 0.2" моль"`

Железо в избытке, значит расчёт ведём по сере.

`n(FeS) = n(S) = 0.2" моль"`

`m(FeS) = n(FeS)*M(FeS) = 0.2*88 = 17.6 г`

Ответ: `m(FeS) = 17.6 г`.

3. К раствору, содержащему 8,32 г хлорида бария, прибавили раствор, в котором содержалось 4,9 г серной кислоты. Рассчитайте массу полученного осадка.

Ответ:

Дано:

`m(BaCl_2) = 8.32 г`

`m(H_2SO_4) = 4.9 г`

`m(BaSO_4) = ?`

Решение

`BaCl_2 + H_2SO_4 ⟶ 2HCl + BaSO_4"↓"`

`n(BaCl_2) = (m(BaCl_2))/(M(BaCl_2)) = 8.32/208 = 0.04" моль"`

`n(H_2SO_4) = (m(H_2SO_4))/(M(H_2SO_4)) = 4.9/98 = 0.05" моль"`

Серная кислота в избытке, значит расчёт ведём по хлориду бария.

`n(BaSO_4) = n(BaCl_2) = 0.04" моль"`

`m(BaSO_4) = n(BaSO_4)*M(BaSO_4) = 0.04*233 = 9.32 г`

Ответ: `m(BaSO_4) = 9.32 г`.

4. К раствору, содержащему 22,4 г гидроксида калия, прибавили раствор, содержащий 21,9 г хлороводорода. Сколько граммов соли образовалось?

Ответ:

Дано:

`m(KOH) = 22.4 г`

`m(HCl) = 21.9 г`

`m(KCl) = ?`

Решение

`KOH + HCl ⟶ KCl + H_2O`

`n(KOH) = (m(KOH))/(M(KOH)) = 22.4/56 = 0.4" моль"`

`n(HCl) = (m(HCl))/(M(HCl)) = 21.9/36.5 = 0.6" моль"`

Хлороводород в избытке, значит расчёт ведём по гидроксиду калия.

`n(KCl) = n(KOH) = 0.4" моль"`

`m(KCl) = n(KCl)*M(KCl) = 0.4*74.5 = 29.8 г`

Ответ: `m(KCl) = 29.8 г`.

5. Кусочек цинка массой 3,9 г растворили в 75 г 7,3%-й соляной кислоты. Найдите массу образовавшегося хлорида цинка и объём выделившегося водорода (н. у.).

Ответ:

Дано:

`m(Zn) = 3.9 г`

`m("р-ра") = 75 г`

`ω(HCl) = 7.3%`

`m(ZnCl_2) = ?`

`V(H_2) = ?`

Решение

`Zn + 2HCl ⟶ ZnCl_2 + H_2"↑"`

`n(Zn) = (m(Zn))/(M(Zn)) = 3.9/65 = 0.06" моль"`

`m(HCl) = (ω(HCl)*m("р-ра"))/100 = (7.3*75)/100 = 5.5 г`

`n(HCl) = (m(HCl))/(M(HCl)) = 5.5/36.5 = 0.15" моль"`

Хлороводород в избытке, значит расчёт ведём по цинку.

`n(ZnCl_2) = n(H_2) = n(Zn) = 0.06" моль"`

`m(ZnCl_2) = n(ZnCl_2)*M(ZnCl_2) = 0.06*136 = 8.16 г`

`V(H_2) = n(H_2)*V_m = 0.06*22.4 = 1.344 л`

Ответ: `m(ZnCl_2) = 8.16 г`, `V(H_2) = 1.344 л`.

6. Какой объём газа (н. у.) выделится при смешении 20 г 10,6%-го раствора карбоната натрия и 80 г 4,9%-го раствора серной кислоты?

Ответ:

Дано:

`m("р-ра") = 20 г`

`ω(Na_2CO_3) = 10.6%`

`m("р-ра") = 80 г`

`ω(H_2SO_4) = 4.9%`

`V(CO_2) = ?`

Решение

`Na_2CO_3 + H_2SO_4 ⟶ Na_2SO_4 + H_2O + CO_2"↑"`

`m(Na_2CO_3) = (ω(Na_2CO_3)*m("р-ра"))/100 = (10.6*20)/100 = 2.12 г`

`n(Na_2CO_3) = (m(Na_2CO_3))/(M(Na_2CO_3)) = 2.12/106 = 0.02" моль"`

`m(H_2SO_4) = (ω(H_2SO_4)*m("р-ра"))/100 = (4.9*80)/100 = 3.92 г`

`n(H_2SO_4) = (m(H_2SO_4))/(M(H_2SO_4)) = 3.92/98 = 0.04" моль"`

Серная кислота в избытке, значит расчёт ведём по карбонату натрия.

`n(CO_2) = n(Na_2CO_3) = 0.02" моль"`

`V(CO_2) = n(CO_2)*V_m = 0.02*22.4 = 0.448 л = 448" мл"`

Ответ: `V(CO_2) = 448" мл"`.

7. Сколько граммов 10%-й соляной кислоты можно получить из 5,6 л хлора и 11,2 л водорода (н. у.)?

Ответ:

Дано:

`V(Cl_2) = 5.6 л`

`V(H_2) = 11.2 л`

`ω(HCl) = 10%`

`m("р-ра") = ?`

Решение

`H_2 + Cl_2 ⟶ 2HCl`

Водород в избытке, значит расчёт ведём по хлору.

`n(Cl_2) = (V(Cl_2))/V_m = 5.6/22.4 = 0.25" моль"`

`n(HCl) = 2*n(Cl_2) = 2*0.25 = 0.5" моль"`

`m(HCl) = n(HCl)*M(HCl) = 0.5*36.5 = 18.25 г`

`m("р-ра") = (100*m(HCl))/(ω(HCl)) = (100*18.25)/10 = 182.5 г`

Ответ: `m("р-ра") = 182.5 г`.

8. При прокаливании 29 г гидроксида магния получили 7,2 г воды. Какая часть гидроксида разложилась?

Ответ:

Дано:

`m_"исх."(Mg(OH)_2) = 29 г`

`m(H_2O) = 7.2 г`

`ω_"разлож."(Mg(OH)_2) = ?`

Решение

`Mg(OH)_2 overset(t)(⟶) MgO + H_2O`

`n(H_2O) = (m(H_2O))/(M(H_2O)) = 7.2/18 = 0.4" моль"`

`n_"разлож."(Mg(OH)_2) = n(H_2O) = 0.4" моль"`

`m_"разлож."(Mg(OH)_2) = n_"разлож."(Mg(OH)_2)*M(Mg(OH)_2) = 0.4*58 = 23.2 г`

`ω_"разлож."(Mg(OH)_2) = (100*m_"разлож."(Mg(OH)_2))/(m_"исх."(Mg(OH)_2)) = (100*23.2)/29 = 80%`

Ответ: `ω_"разлож."(Mg(OH)_2) = 80%`.

9. При нагревании 80 г брома с избытком водорода образовалось 56,7 г бромоводорода. Найдите выход продукта этой реакции.

Ответ:

Дано:

`m(Br_2) = 80 г`

`m_"пр."(HBr) = 56.7 г`

`η(HBr) = ?`

Решение

`H_2 + Br_2 ⟶ 2HBr`

`n(Br_2) = (m(Br_2))/(M(Br_2)) = 80/160 = 0.5" моль"`

`n_"теор."(HBr) = 2*n(Br_2) = 2*0.5 = 1" моль"`

`m_"теор."(HBr) = n_"теор."(HBr)*M(HBr) = 1*81 = 81 г`

`η(HBr) = (100*m_"пр."(HBr))/(m_"теор."(HBr)) = (100*56.7)/81 = 70%`

Ответ: `η(HBr) = 70%`.

10. При разложении 4,9 г бертолетовой соли KClO3 было получено 1,28 л кислорода (н. у.). Определите выход продукта.

Ответ:

Дано:

`m(KClO_3) = 4.9 г`

`V_"пр."(O_2) = 1.28 л`

`η(HBr) = ?`

Решение

`2KClO_3 ⟶ 2KCl + 3O_2"↑"`

`n(KClO_3) = (m(KClO_3))/(M(KClO_3)) = 4.9/122.5 = 0.04" моль"`

`n_"теор."(O_2) = (3*n(KClO_3))/2 = (3*0.04)/2 = 0.06" моль"`

`V_"теор."(O_2) = n_"теор."(O_2)*V_m = 0.06*22.4 = 1.344 л`

`η(O_2) = (100*V_"пр."(O_2))/(V_"теор."(O_2)) = (100*1.28)/1.344 = 95.2%`

Ответ: `η(O_2) = 95.2%`.

11. При сжигании 100 кг серы образовалось 160 кг оксида серы (IV). Рассчитайте выход продукта.

Ответ:

Дано:

`m(S) = 100" кг"`

`m_"теор."(SO_2) = 160" кг"`

`η(SO_2) = ?`

Решение

`S + O_2 ⟶ SO_2`

`n(S) = (m(S))/(M(S)) = 100/32 = 3.125" кмоль"`

`n_"теор."(SO_2) = n(S) = 3.125" кмоль"`

`m_"теор."(SO_2) = n_"теор."(SO_2)*M(SO_2) = 3.125*64 = 200" кг"`

`η(SO_2) = (100*m_"пр."(SO_2))/(m_"теор."(SO_2)) = (100*160)/200 = 80%`

Ответ: `η(SO_2) = 80%`.

12. Определите массу железа, которое можно получить восстановлением 480 г оксида железа (III), если выход продукта равен 75%.

Ответ:

Дано:

`m(Fe_2O_3) = 480 г`

`η(Fe) = 75%`

`m_"пр."(Fe) = ?`

Решение

`Fe_2O_3 + 3H_2 ⟶ 2Fe + 3H_2O`

`n(Fe_2O_3) = (m(Fe_2O_3))/(M(Fe_2O_3)) = 480/160 = 3" моль"`

`n_"теор."(Fe) = 2*n(Fe_2O_3) = 2*3 = 6" моль"`

`m_"теор."(Fe) = n_"теор."(Fe)*M(Fe) = 6*56 = 336 г`

`m_"пр."(Fe) = (η(Fe)*m_"теор."(Fe))/100 = (75*336)/100 = 252 г`

Ответ: `m_"пр."(Fe) = 252 г`.

13. Определите массу карбоната бария, который вступил в реакцию с избытком соляной кислоты, если при этом выделилось 20 л углекислого газа (н. у.), а выход продукта составил 90%.

Ответ:

Дано:

`V_"пр."(CO_2) = 20 л`

`η(CO_2) = 90%`

`m(BaCO_3) = ?`

Решение

`BaCO_3 + 2HCl ⟶ BaCl_2 + H_2O + CO_2"↑"`

`V_"теор."(CO_2) = (100*V_"пр."(CO_2))/(η(CO_2)) = (100*20)/90 = 22.2 л`

`n_"теор."(CO_2) = (V_"теор."(CO_2))/V_m = 22.2/22.4 = 0.99" моль"`

`n(BaCO_3) = n_"теор."(CO_2) = 0.99" моль"`

`m(BaCO_3) = n(BaCO_3)*M(BaCO_3) = 0.99*197 = 195 г`

Ответ: `m(BaCO_3) = 195 г`.

14. При взаимодействии раствора сульфата меди (II) с железными опилками выделилось 7,68 г меди, что составляет 60% от теоретически возможной массы. Сколько граммов 10%-го раствора сульфата меди (II) было использовано?

Ответ:

Дано:

`m_"пр."(Cu) = 7.68 г`

`η(Cu) = 60%`

`ω(CuSO_4) = 10%`

`m("р-ра") = ?`

Решение

`CuSO_4 + Fe ⟶ FeSO_4 + Cu`

`m_"теор."(Cu) = (100*m_"пр."(Cu))/(η(Cu)) = (100*7.68)/60 = 12.8 г`

`n_"теор."(Cu) = (m_"теор."(Cu))/(M(Cu)) = 12.8/64 = 0.2" моль"`

`n(CuSO_4) = n_"теор."(Cu) = 0.2" моль"`

`m(CuSO_4) = n(CuSO_4)*M(CuSO_4) = 0.2*160 = 32 г`

`m("р-ра") = (100*m(CuSO_4))/(ω(CuSO_4)) = (100*32)/10 = 320 г`

Ответ: `m("р-ра") = 320 г`.

15. Сколько литров аммиака можно получить из 600 л водорода, если объёмная доля выхода аммиака равна 40%? Объёмы газов измерены при одинаковых условиях.

Ответ:

Дано:

`V(H_2) = 600 л`

`η(NH_3) = 40%`

`V_"пр."(NH_3) = ?`

Решение

`N_2 + 3H_2 ⟶ 2NH_3`

`n(H_2) = (V(H_2))/V_m = 600/22.4 = 26.8" моль"`

`n_"теор."(NH_3) = (2*n(H_2))/3 = (2*26.8)/3 = 17.9" моль"`

`V_"теор."(NH_3) = n_"теор."(NH_3)*V_m = 17.9*22.4 = 401 л`

`V_"пр."(NH_3) = (η(NH_3)*V_"теор."(NH_3))/100 = (40*401)/100 = 160 л`

Ответ: `V_"пр."(NH_3) = 160 л`.

§9

Глава 2. Химическая реакция

§9. Электролиты и неэлектролиты. Электролитическая диссоциация

1. Какие вещества называют электролитами; неэлектролитами? Приведите примеры.

Ответ:

Вещества, растворы или расплавы которых проводят электрический ток, называют электролитами, а вещества, не проводящие ток ни в растворе, ни в расплаве, – неэлектролитами.

Электролитом является, например, хлорид натрия, его водный раствор и расплав проводят электрический ток.

Неэлектролитом является, например, сахар, его водный раствор и расплав не проводит электрический ток.

2. Что такое электролитическая диссоциация? Могут ли при растворении электролита в воде образоваться только катионы или только анионы? Почему?

Ответ:

Процесс распада вещества на ионы при растворении в воде или плавлении называют электролитической диссоциацией.

Не могут, потому что молекулы электролита в целом электронейтральны, а значит они состоят одновременно из катионов и анионов, т. е. при растворении электролита в воде обязательно образуются как катионы, так и анионы.

3. Выпишите из текста параграфа основные положения теории электролитической диссоциации: 1) о распаде электролитов на ионы; 2) о движении ионов в электрическом поле.

Ответ:

Основные положения теории электролитической диссоциации:

1) В водном растворе электролиты распадаются (диссоциируют) на положительные и отрицательные ионы.

2) Под действием электрического поля положительно заряженные ионы (катионы) движутся к отрицательному электроду (катоду), а отрицательно заряженные ионы (анионы) – к положительному электроду (аноду).

4. Из приведённых формул выберите формулы веществ, которые, на ваш взгляд, являются: а) электролитами; б) неэлектролитами.
NaI, HBr, O2, CH4, CaCl2, KOH, C12H22O11 (сахар).

Ответ:

а) NaI, HBr, CaCl2, KOH.

б) O2, CH4, C12H22O11.

5. Сколько молей частиц образуется при растворении в воде одного моля: а) хлороводорода; б) хлорида калия; в) хлорида кальция; г) нитрата кальция? Напишите уравнения электролитической диссоциации этих веществ.

Ответ:

а) При растворении в воде одного моля хлороводорода образуется 2 моля частиц:
HCl ⟶ H+ + Cl-

б) При растворении в воде одного моля хлорида калия образуется 2 моля частиц:
KCl ⟶ K+ + Cl-

в) При растворении в воде одного моля хлорида кальция образуется 3 моля частиц:
CaCl2 ⟶ Ca2+ + 2Cl-

г) При растворении в воде одного моля нитрата кальция образуется 3 моля частиц:
Ca(NO3)2 ⟶ Ca2+ + 2NO3-

6. Опишите механизм электролитической диссоциации хлорида натрия. Какую роль играет вода в этом процессе?

Ответ:

При попадании в воду кристалла хлорида натрия, его окружают со всех сторон молекулы воды. Они притягиваются отрицательными концами к ионам натрия, а положительными – к ионам хлора. Такие ионы постепенно теряют связь с другими ионами кристаллической решётки и переходят в раствор, окружённые молекулами воды, т. е. становятся гидратированными.

Вода играет роль разрушителя ионной кристаллической решетки (растворителя).

7. Какие ионы называют гидратированными?

Ответ:

Ионы окружённые молекулами воды называют гидратированными.

8. Приведите по одному примеру веществ, при растворении которых в воде наблюдается: а) сильное разогревание раствора; б) сильное охлаждение раствора. Объясните эти явления.

Ответ:

а) Гидроксида калия.

б) Нитрат аммония.

Для того чтобы разрушить кристаллическую решётку до отдельных катионов и анионов, требуется большое количество энергии (её называют энергией кристаллической решётки). Эти энергетические затраты полностью или отчасти компенсируются энергией, выделяющейся при химическом взаимодействии ионов с молекулами воды (энергией гидратации). Если энергия гидратации больше энергии решётки, то при растворении вещества теплота выделяется, в противном случае – поглощается.

Лабораторный опыт 1. Изучение электропроводности растворов и движения ионов в электрическом поле.

1. В гнезда выданной вам керамической пластинки поместите по одной ложечке хлорида натрия, нитрата калия и сахара. Прибор для испытания электропроводности включите в сеть и прикоснитесь на несколько секунд электродами поочерёдно к каждому веществу. Что вы наблюдаете?

2. Аккуратно прилейте в каждое гнездо дистиллированную воду и вновь исследуйте электропроводность. Запишите наблюдения.

3. Положите на стеклянную пластинку полоску фильтровальной бумаги и смочите её раствором сульфата меди (II). Голубая окраска раствора обусловлена ионами меди Cu2+. Прикоснитесь к бумаге электродами и включите ток. Наблюдайте, в сторону какого электрода смещается цветное пятно. Запишите наблюдения. Выключите прибор из сети, промойте электроды и оботрите их тряпкой или фильтровальной бумагой.

Ответ:

1. При прикосновении электродами к сухим веществам, протекание тока в цепи не наблюдается.

Вывод: сухие кристаллические вещества не проводят электрический ток.

2. При прикосновении электродами к растворам хлорида натрия и нитрата калия, наблюдается появление тока в цепи.

При прикосновении электродами к раствору сахара, протекание тока в цепи не наблюдается.

Вывод: существуют вещества, водные растворы которых проводят электрический ток, и существуют вещества, водные растворы которых не проводят электрический ток.

3. При прикосновении электродами к бумаге смоченной в растворе сульфата меди (II) наблюдается смещение цветного пятна в сторону катода.

Вывод: катионы меди Cu2+ под действием электрического поля движутся в сторону положительно заряженного электрода (катода).

§10

§10. Диссоциация кислот, оснований и солей

1. Дайте определения кислот, оснований и солей с позиций теории электролитической диссоциации.

Ответ:

Кислота – это электролит, при диссоциации которого образуются катионы водорода и анионы кислотного остатка.

2. Вспомните понятие электроотрицательности (8 класс, §48). Объясните, почему ион водорода, образующийся при диссоциации кислот, имеет положительный заряд.

Ответ:

Электроотрицательность – способность атома притягивать валентные электроны других атомов.

Ион водорода, образующийся при диссоциации кислот, имеет положительный заряд, потому что атом водорода в кислоте отдаёт свой валентный электрон более электроотрицательному кислотному остатку.

3. При диссоциации молекулы кислоты образовался ион, который имеет заряд -3. Сколько ионов водорода при этом образовалось?

Ответ:

Заряд одного иона водорода равен +1, значит образовалось 3 иона, так как молекула в целом электронейтральна.

4. Напишите уравнения электролитической диссоциации кислот: а) азотной, серной; б) бромоводородной HBr, азотистой HNO2. Назовите образующиеся анионы.

Ответ:

а)

`HNO_3 ⟶ H^(+) + underset("нитрат-анион")(NO_3^(-))`

`H_2SO_4 ⟶ 2H^(+) + underset("сульфат-анион")(SO_4^(2-))`

б)

`HBr ⟶ H^(+) + underset("бромид-анион")(Br^(-))`

`HNO_2 ⟶ H^(+) + underset("нитрит-анион")(NO_2^(-))`

5. Составьте формулы следующих солей: нитрата бария, хлорида алюминия, карбоната натрия, фосфата калия. Напишите уравнения диссоциации этих солей.

Ответ:

Ba(NO3)2 ⟶ Ba2+ + 2NO3-

AlCl3 ⟶ Al3+ + 3Cl-

Na2CO3 ⟶ 2Na+ + CO32-

K3PO4 ⟶ 3K+ + PO43-

6. Приведите по одному примеру солей, при диссоциации одного моля которых образуются: а) два; б) три; в) четыре; г) пять молей ионов. Напишите уравнения диссоциации.

Ответ:

а) NaCl ⟶ Na+ + Cl-

б) Ca(NO3)2 ⟶ Ca2+ + 2NO3-

в) AlCl3 ⟶ Al3+ + 3Cl-

г) Fe2(SO4)3 ⟶ 2Fe3+ + 3SO42-

7. Расставьте заряды ионов в формулах следующих веществ: а) Na2S, Na2SO4, MgSO4, Na3PO4, AlPO4; б) NaHSO4, Mg(HSO4)2, CaHPO4, NaOH, Ba(OH)2. Назовите эти вещества.

Ответ:

а)

`underset("сульфид натрия")(color(blue)(overset(1+)(Na))_2color(red)(overset(2-)(S)))`

`underset("сульфат натрия")(color(blue)(overset(1+)(Na))_2color(red)(overset(2-)(SO_4)))`

`underset("сульфат магния")(color(blue)(overset(2+)(Mg))color(red)(overset(2-)(SO_4)))`

`underset("фосфат натрия")(color(blue)(overset(1+)(Na))_3color(red)(overset(3-)(PO_4)))`

`underset("фосфат алюминия")(color(blue)(overset(3+)(Al))color(red)(overset(3-)(PO_4)))`

б)

`underset("гидросульфат натрия")(color(blue)(overset(1+)(Na))color(red)(overset(1-)(HSO_4)))`

`underset("гидросульфат магния")(color(blue)(overset(2+)(Mg))"("color(red)(overset(1-)(HSO_4))")"_2)`

`underset("гидрофосфат кальция")(color(blue)(overset(2+)(Ca))color(red)(overset(2-)(HPO_4)))`

`underset("гидроксид натрия")(color(blue)(overset(1+)(Na))color(red)(overset(1-)(OH)))`

`underset("гидроксид бария")(color(blue)(overset(2+)(Ba))"("color(red)(overset(1-)(OH))")"_2)`

8. При диссоциации нитрата металла образовался 1 моль катионов, которые имеют заряд +2. Сколько молей нитрат-ионов при этом образовалось?

Ответ:

Нитрат-ионы имеют заряд -1, значит образовалось 2 моль нитрат-ионов.

9. Составьте формулы и напишите уравнения диссоциации сульфата железа (II) и сульфата железа (III). Чем отличаются эти соли?

Ответ:

FeSO4 ⟶ Fe2+ + SO42-

Fe2(SO4)3 ⟶ 2Fe3+ + 3SO42-

Эти соли отличаются зарядом иона железа.

10. Пусть M обозначает металл, X – кислотный остаток. Ниже приведены четыре типа уравнений диссоциации солей:
а) MX ⟶ M2+ + X2-;
б) MX3 ⟶ M3+ + 3X-;
в) M3X ⟶ 3M+ + X3-;
г) M2X3 ⟶ 2M3+ + 3X2-.

Приведите по одному примеру соли каждого типа.

Ответ:

а) MgSO4 ⟶ Mg2+ + SO42-

б) AlCl3 ⟶ Al3+ + 3Cl-

в) Na3PO4 ⟶ 3Na+ + PO43-

г) Al2(SO4)3 ⟶ 2Al3+ + 3SO42-

11. Напишите уравнения электролитической диссоциации следующих веществ: гидроксида натрия, серной кислоты, нитрата калия, хлорида магния, сульфата алюминия.

Ответ:

NaOH ⟶ Na+ + OH-

H2SO4 ⟶ 2H+ + SO42-

KNO3 ⟶ K+ + NO3-

MgCl2 ⟶ Mg2+ + 2Cl-

Al2(SO4)3 ⟶ 2Al3+ + 3SO42-

12. Напишите формулу вещества, при диссоциации которого образуются ионы кальция и гидроксид-ионы.

Ответ:

Это Ca(OH)2

Ca(OH)2 ⟶ Ca2+ + 2OH-

13. Определите валентность и степень окисления кислорода в ионе гидроксония H3O+.

Ответ:

В ионе гидроксония валентность кислорода равна 3, а его степень окисления равна -2.

14. Напишите уравнения электролитической диссоциации, в результате которой образуются следующие пары ионов:
а) H+ и NO3-;
б) Al3+ и SO42-;
в) Ca2+ и H2PO4-.

Ответ:

а) HNO3 ⟶ H+ + NO3-

б) Al2(SO4)3 ⟶ 2Al3+ + 3SO42-

в) Ca(H2PO4)2 ⟶ Ca2+ + 2H2PO4-

15. Даны правые части уравнений диссоциации:
а) … ⟶ H+ + SO42-;
б) … ⟶ Na+ + SO32-;
в) … ⟶ Ca2+ + NO3-;
г) … ⟶ Cu2+ + Cl-;
д) … ⟶ Fe3+ + NO3-;
е) … ⟶ Ba2+ + OH-.

Расставьте коэффициенты и завершите эти уравнения. Назовите исходные вещества.

Ответ:

а) `underset("серная кислота")(H_2SO_4) ⟶ 2H^(+) + SO_4^(2-)`

б) `underset("сульфит натрия")(Na_2SO_3) ⟶ 2Na^(+) + SO_3^(2-)`

в) `underset("нитрат кальция")(Ca(NO_3)_2) ⟶ Ca^(2+) + 2NO_3^(-)`

г) `underset("хлорид меди (II)")(CuCl_2) ⟶ Cu^(2+) + 2Cl^(-)`

д) `underset("нитрат железа (III)")(Fe(NO_3)_3) ⟶ Fe^(3+) + 3NO_3^(-)`

е) `underset("гидроксид бария")(Ba(OH)_2) ⟶ Ba^(2+) + 2OH^(-)`

16. В растворе присутствуют ионы K+, Mg2+, NO3-, SO42-. Какие вещества были растворены? Предложите два варианта ответа.

Ответ:

1) KNO3 и MgSO4.

2) Mg(NO3)2 и K2SO4.

17. Водный раствор содержит 0,8 моль ионов K+, 0,5 моль ионов Mg2+ и ионы NO3-. Чему равно количество ионов NO3-?

Ответ:

1) KNO3 ⟶ K+ + NO3-

2) Mg(NO3)2 ⟶ Mg2+ + 2NO3-

[NO3-]1 = [K+] = 0,8 моль

[NO3-]2 = 2∙[Mg2+] = 2∙0,5 = 1 моль

[NO3-]общ. = [NO3-]1 + [NO3-]2 = 0,8 + 1 = 1,8 моль

§11

§11. Сильные и слабые электролиты

1. Какие электролиты называют сильными; слабыми? Приведите примеры.

Ответ:

Вещества, которые практически полностью распадаются в водном растворе на ионы, называют сильными электролитами.
К сильным электролитам относятся, например, хлорид натрия, серная кислота, гидроксид натрия.

Вещества, которые при растворении в воде распадаются на ионы лишь частично, называют слабыми электролитами.
К слабым электролитам относятся, например, уксусная, фторводородная, сероводородная кислоты.

2. Из приведённого ниже списка выпишите отдельно формулы: а) сильных электролитов; б) слабых электролитов.

NaCl, HCl, NaOH, NaNO3, HNO3, HNO2, H2SO4, Ba(OH)2, H2S, K2S, Pb(NO3)2.

Ответ:

а) NaCl, HCl, NaOH, NaNO3, HNO3, H2SO4, Ba(OH)2, K2S, Pb(NO3)2.

б) HNO2, H2S.

3. Напишите уравнения диссоциации сульфата натрия, соляной кислоты, карбоната калия, гидроксида калия, сероводородной кислоты. В каких случаях диссоциация протекает обратимо?

Ответ:

Na2SO4 ⟶ 2Na+ + SO42-

HCl ⟶ H+ + Cl-

K2CO3 ⟶ 2K+ + CO32-

KOH ⟶ K+ + OH-

H2S ⇄ H+ + HS-

4. Какую величину используют для количественной характеристики силы электролита? От каких факторов она зависит?

Ответ:

Силу электролитов количественно характеризует степень электролитической диссоциации (α), она показывает долю вещества, распавшегося в растворе на ионы.

Степень электролитической диссоциации зависит от природы электролита, от его концентрации в растворе и от температуры раствора. При разбавлении раствора водой, а также при повышении температуры степень диссоциации увеличивается.

5. В растворе синильной кислоты концентрацией 1,5 моль/л из каждого миллиона молекул HCN только 20 диссоциируют на ионы. Рассчитайте степень диссоциации синильной кислоты в этом растворе. Какая это кислота – сильная или слабая? Чему равна концентрация ионов водорода в растворе?

Ответ:

Дано:

`c(HCN) = 1.5" моль/л"`

`n(HCN) = 20/10^6*n_0(HCN)`

`α = ?`

`c(H^+) = ?`

Решение

`HCN ⟶ H^(+) + CN^(-)`

`α = (100*n(HCN))/(n_0(HCN)) = (100*20)/10^6 = 0.002%`

Степень диссоциации меньше 10%, поэтому синильная кислота – слабый электролит.

`c(H^+) = (α*c(HCN))/100 = (0.002*1.5)/100 = 3*10^(-5)`

Ответ: `α = 0.002%`, слабый электролит.

6. В водном растворе сульфата натрия содержится 0,3 моль ионов. Сколько граммов соли использовали для приготовления раствора?

Ответ:

Дано:

`n("ионов") = 0.3" моль"`

`m(Na_2SO_4) = ?`

Решение

`Na_2SO_4 ⟶ 2Na^(+) + SO_4^(2-)`

`n(Na_2SO_4) = (n("ионов"))/3 = 0.3/3 = 0.1" моль"`

`m(Na_2SO_4) = n(Na_2SO_4)*M(Na_2SO_4) = 0.1*142 = 14.2 г`

Ответ: `m(Na_2SO_4) = 14.2 г`.

7. В одном литре воды растворили 10 моль бромоводорода. Полученный раствор содержит 8,7 моль ионов водорода. Определите степень диссоциации бромоводорода (в процентах). Какой это электролит – сильный или слабый?

Ответ:

Дано:

`n_0(HBr) = 10" моль"`

`n(H^+) = 8.7" моль"`

`α = ?`

Решение

`HBr ⟶ H^(+) + Br^(-)`

`n(HBr) = n(H^+) = 8.7" моль"`

`α = (100*n(HBr))/(n_0(HBr)) = (100*8.7)/10 = 87%`

Степень диссоциации больше 50%, поэтому бромоводород – сильный электролит.

Ответ: `α = 87%`, сильный электролит.

8. В одном литре воды растворили 7 моль фтороводорода HF. Полученный раствор содержит 0,07 моль ионов водорода. Определите степень диссоциации фтороводорода (в процентах). Какой это электролит – сильный или слабый? Сравните полученный ответ с ответом задачи из текста параграфа. Подумайте, почему при увеличении концентрации электролита степень диссоциации уменьшается

Ответ:

Дано:

`n_0(HF) = 7" моль"`

`n(H^+) = 0.07" моль"`

`α = ?`

Решение

`HF ⟶ H^(+) + F^(-)`

`n(HF) = n(H^+) = 0.07" моль"`

`α = (100*n(HF))/(n_0(HF)) = (100*0.07)/7 = 1%`

Степень диссоциации меньше 10%, поэтому фтороводород – слабый электролит.

Ответ: `α = 1%`, слабый электролит.

При увеличении концентрации электролита увеличивается число столкновений катионов и анионов электролита, в результате чего может образоваться недиссоциированная молекула электролита, поэтому степень диссоциации уменьшается.

§12

§12. Кислотность среды. Водородный показатель

1. Водный раствор имеет щелочную среду. О присутствии каких ионов это свидетельствует?

Ответ:

Щелочная среда свидетельствует о присутствии гидроксид-ионов OH-.

2. Из приведённого ниже списка выпишите формулы веществ, растворы которых имеют среду: а) кислотную; б) щелочную.

NaCl, HCl, NaOH, HNO3, H3PO4, H2SO4, Ba(OH)2, H2S, KNO3.

Ответ:

а) HCl, HNO3, H3PO4, H2SO4, H2S.

б) NaOH, Ba(OH)2.

3. Дождевая вода имеет pH = 5,6. Что это значит? Какое вещество, содержащееся в воздухе, при растворении в воде создаёт эту среду?

Ответ:

pH = 5,6 означает то, что дождевая вода имеет слабокислую среду, которая является результатом растворения углекислого газа в воде.
CO2 + H2O ⇄ H2CO3
H2CO3 ⇄ H+ + HCO3-

4. В состав каменного угля, используемого на теплоэлектростанциях, входят соединения азота и серы. Выброс в атмосферу продуктов его сжигания приводит к выпадению кислотных дождей, содержащих небольшие количества азотной или серной кислот. Какие значения pH имеет кислотная дождевая вода – больше или меньше 7,0?

Ответ:

Кислотная дождевая вода имеет pH меньше 7,0.

5. Зависит ли pH раствора сильной кислоты от её концентрации? Ответ обоснуйте.

Ответ:

Растворы сильных кислот одинаковой концентрации имеют одинаковое значение pH. Это связано с тем, что все сильные кислоты почти полностью диссоциированы в водном растворе.

6. Какую окраску имеет лакмус в растворах: а) NaOH; б) HCl; в) NaCl?

Ответ:

а) Синюю.

б) Красную.

в) Фиолетовую.

7. Напишите уравнение диссоциации гидроксида бария. Какой цвет имеют лакмус, фенолфталеин и метилоранж в растворе этого вещества?

Ответ:

Ba(OH)2 ⟶ Ba2+ + 2OH-

В растворе гидроксида бария лакмус имеет синий цвет, фенолфталеин – малиновый, метилоранж – желтый.

8. В двух пробирках находятся растворы азотной кислоты и нитрата калия. Какой индикатор нужно использовать, чтобы определить, где находится раствор соли?

Ответ:

Для того чтобы определить где находится раствор нитрата калия, можно использовать индикатор лакмус. В растворе азотной кислоты он изменит цвет на красный, а в нитрате калия он не изменит окраску, т. е. останется фиолетовым.

9. В трёх пробирках находятся растворы гидроксида бария, азотной кислоты и нитрата кальция. Как с помощью одного реактива можно распознать эти растворы?

Ответ:

Растворы гидроксида бария, азотной кислоты и нитрата кальция можно распознать с помощью индикатора лакмуса. В растворе гидроксида бария он изменит свою окраску на синий цвет, в растворе азотной кислоты – на красный цвет, а в растворе нитрата кальция останется фиолетовым.

10. К раствору, содержащему 1 моль гидроксида калия, прилили фенолфталеин. Изменится ли окраска раствора, если к нему затем добавить соляную кислоту, содержащую: а) 0,5 моль; б) 1 моль; в) 1,5 моль HCl?

Ответ:

В растворе гидроксида калия фенолфталеин имеет малиновый цвет.

KOH + HCl ⟶ KCl + H2O

а) Если прилить раствору, содержащему 1 моль гидроксида калия, 0,5 моль соляной кислоты, то в растворе ещё останется 0,5 моль непрореагировавшего гидроксида калия, поэтому окраска раствора не изменится.

б) Если прилить раствору, содержащему 1 моль гидроксида калия, 1 моль соляной кислоты, то исходные вещества полностью прореагируют между собой, и среда раствора станет нейтральной. В нейтральной среде фенолфталеин не имеет цвета, поэтому раствор обесцветится.

в) Если прилить раствору, содержащему 1 моль гидроксида калия, 1,5 моль соляной кислоты, то в растворе останется 0,5 моль HCl, и среда раствора станет кислой. В кислой среде фенолфталеин не имеет цвета, поэтому раствор обесцветится.

11. Имеются три раствора одинаковой концентрации: KOH, HNO3, H2S. Расположите их в порядке увеличения pH раствора. Ответ подтвердите уравнениями электролитической диссоциации.

Ответ:

В порядке увеличения pH раствора: HNO3, H2S, KOH.

HNO3 ⟶ H+ + NO3-

H2S ⇄ H+ + HS-

KOH ⟶ K+ + OH-

Лабораторный опыт 2. Сравнение окраски индикаторов в разных средах. Определение кислотности среды.

1. В три пробирки налейте примерно по 1 мл раствора лакмуса. В первую пробирку добавьте каплю соляной кислоты, вторую оставьте для сравнения, а в третью добавьте каплю раствора гидроксида натрия. Отметьте изменение окраски раствора. Аналогичный опыт проделайте с метилоранжем и фенолфталеином. Результаты наблюдений занесите в таблицу 7.

Таблица 7. Изменение окраски индикаторов в различных средах.
Индикатор Среда
кислотная нейтральная щелочная
Лакмус      
Метилоранж      
Фенолфталеин      

2. Запишите в тетрадь состав выданных вам растворов и пронумеруйте их. В первый раствор опустите стеклянную палочку, затем прикоснитесь ею к полоске индикаторной бумаги и сравните полученную окраску со шкалой. Значение pH запишите в тетрадь. Промойте палочку водой и проведите исследование остальных растворов. Какие из них являются нейтральными, кислотными, щелочными?

Ответ:

1.

Изменение окраски индикаторов в различных средах.

Индикатор

Среда

кислотная

нейтральная

щелочная

Лакмус

красный

фиолетовый

синий

Метилоранж

красный

оранжевый

желтый

Фенолфталеин

бесцветный

бесцветный

малиновый

2. Значения pH будут индивидуальные.
HCl - кислая среда, NaOH  -  щелочная.

§13

§13. Реакции ионного обмена и условия их протекания

1. При каких условиях протекают реакции ионного обмена в растворе? Приведите по одному примеру для каждого случая.

Ответ:

Реакции ионного обмена в растворах протекают, если один из продуктов реакции – вода, осадок или газ.

NaOH + HCl ⟶ NaCl + H2O

Ca(NO3)2 + K2CO3 ⟶ 2KNO3 + CaCO3

Na2CO3 + 2HCl ⟶ 2NaCl + H2O + CO2

2. Допишите уравнения возможных реакций:
а) Ba(OH)2 + HCl ⟶ …;
б) K2S + H2SO4 ⟶ …;
в) BaCl2 + MgSO4 ⟶ …;
г) NaNO3 + MgCl2 ⟶ …;
д) AgNO3 + AlCl3 ⟶ …;
е) CaCO3 + HCl ⟶ …;
ж) Cu(OH)2 + H2SO4 ⟶ …;
з) FeCl2 + NaOH ⟶ ….

Напишите полные и сокращённые ионные уравнения этих реакций. В каждом случае назовите признак реакции, по которому можно судить о её протекании.

Ответ:

а) Ba(OH)2 + 2HCl ⟶ BaCl2 + 2H2O
Ba2+ + 2OH- + 2H+ + 2Cl- ⟶ Ba2+ + 2Cl- + 2H2O
H+ + OH- ⟶ H2O
Признак реакции: нагревание раствора.

б) K2S + H2SO4 ⟶ K2SO4 + H2S↑
2K+ + S2- + 2H+ + SO42- ⟶ 2K+ + SO42- + H2S↑
2H+ + S2- ⟶ H2S↑
Признак реакции: образование газа.

в) BaCl2 + MgSO4 ⟶ MgCl2 + BaSO4
Ba2+ + 2Cl- + Mg2+ + SO42- ⟶ Mg2+ + 2Cl- + BaSO4
Ba2+ + SO42- ⟶ BaSO4
Признак реакции: выпадение осадка.

г) NaNO3 + MgCl2 ⇸

д) 3AgNO3 + AlCl3 ⟶ Al(NO3)3 + 3AgCl↓
3Ag+ + 3NO3- + Al3+ + 3Cl- ⟶ Al3+ + 3NO3- + 3AgCl↓
Ag+ + Cl- ⟶ AgCl↓
Признак реакции: выпадение осадка.

е) CaCO3 + 2HCl ⟶ CaCl2 + H2O + CO2
CaCO3 + 2H+ + 2Cl- ⟶ Ca2+ + 2Cl- + H2O + CO2
CaCO3 + 2H+ ⟶ Ca2+ + H2O + CO2
Признаки реакции: растворение осадка и образование газа.

ж) Cu(OH)2 + H2SO4 ⟶ CuSO4 + 2H2O
Cu(OH)2 + 2H+ + SO42- ⟶ Cu2+ + SO42- + 2H2O
Cu(OH)2 + 2H+ ⟶ Cu2+ + 2H2O
Признак реакции: растворение осадка.

з) FeCl2 + 2NaOH ⟶ 2NaCl + Fe(OH)2
Fe2+ + 2Cl- + 2Na+ + 2OH- ⟶ 2Na+ + 2Cl- + Fe(OH)2
Fe2+ + 2OH- ⟶ Fe(OH)2
Признак реакции: выпадение осадка.

3. Приведите по два уравнения реакций, соответствующих следующим сокращённым ионным уравнениям:
а) H+ + OH- = H2O;
б) 2H+ + CO32- = CO2↑ + H2O;
в) CO32- + Cа2+ = CаCО3↓;
г) Fe3+ + 3OH- = Fe(OH)3↓;
д) Pb2+ + S2- = PbS↓;
е) Ba2+ + SO42- = BaSO4↓.

Ответ:

а)

NaOH + HCl ⟶ NaCl + H2O

Ba(OH)2 + 2HNO3 ⟶ Ba(NO3)2 + 2H2O

б)

2HCl + Na2CO3 ⟶ 2NaCl + H2O + CO2

H2SO4 + K2CO3 ⟶ K2SO4 + H2O + CO2

в)

Na2CO3 + CaCl2 ⟶ 2NaCl + CaCO3

K2CO3 + Ca(OH)2 ⟶ 2KOH + CaCO3

г)

FeCl3 + 3NaOH ⟶ 3NaCl + Fe(OH)3

Fe2(SO4)3 + 6KOH ⟶ 3K2SO4 + 2Fe(OH)3

д)

Pb(NO3)2 + Na2S ⟶ 2NaNO3 + PbS↓

Pb(NO3)2 + K2S ⟶ 2KNO3 + PbS↓

е)

Ba(OH)2 + Na2SO4 ⟶ 2NaOH + BaSO4

BaCl2 + H2SO4 ⟶ 2HCl + BaSO4

4. Приведите по одному уравнению, соответствующему каждой схеме реакции:
а) соль + соль = соль↓ + соль;
б) сильная кислота + сильное основание = соль + вода;
в) слабая кислота + сильное основание = соль + вода;
г) сильная кислота + слабое основание = соль + вода;
д) сильная кислота + сильное основание = соль↓ + вода;
е) соль + сильная кислота = соль + слабая кислота.

Напишите сокращённое ионное уравнение каждой реакции.

Ответ:

а) BaCl2 + CuSO4 ⟶ BaSO4↓ + CuCl2
Ba2+ + 2Cl- + Cu2+ + SO42- ⟶ BaSO4↓ + Cu2+ + 2Cl-
Ba2+ + SO42- ⟶ BaSO4

б) Ba(OH)2 + 2HCl ⟶ BaCl2 + 2H2O
Ba2+ + 2OH- + 2H+ + 2Cl- ⟶ Ba2+ + 2Cl- + 2H2O
H+ + OH- ⟶ H2O

в) HF + NaOH ⟶ NaF + H2O
HF + Na+ + OH- ⟶ Na+ + F- + H2O
HF + OH- ⟶ F- + H2O

г) H2SO4 + Cu(OH)2 ⟶ CuSO4 + 2H2O
2H+ + SO42- + Cu(OH)2 ⟶ Cu2+ + SO42- + 2H2O
Cu(OH)2 + 2H+ ⟶ Cu2+ + 2H2O

д) H2SO4 + Ba(OH)2 ⟶ BaSO4↓ + 2H2O
2H+ + SO42- + Ba2+ + 2OH- ⟶ BaSO4↓ + 2H2O

е) K2S + H2SO4 ⟶ K2SO4 + H2S
2K+ + S2- + 2H+ + SO42- ⟶ 2K+ + SO42- + H2S
2H+ + S2- ⟶ H2S

5. Допишите уравнения возможных реакций:
а) AgNO3 + HCl ⟶ …;
б) BaSO4 + NaCl ⟶ …;
в) FeCl2 + Cu(NO3)2 ⟶ …;
г) Al(NO3)3 + HCl ⟶ …;
д) Mg(NO3)2 + KOH ⟶ …;
е) Al(OH)3 + KCl ⟶ …;
ж) K2CO3 + HNO3 ⟶ …;
з) NaCl + H2S ⟶ ….

Объясните, почему остальные реакции не протекают.

Ответ:

а) AgNO3 + HCl ⟶ HNO3 + AgCl

б) BaSO4 + NaCl ⇸ реакция не идёт, т. к. не образуется осадка, газа или воды.

в) FeCl2 + Cu(NO3)2 ⇸ реакция не идёт, т. к. не образуется осадка, газа или воды.

г) Al(NO3)3 + HCl ⇸ реакция не идёт, т. к. не образуется осадка, газа или воды.

д) Mg(NO3)2 + 2KOH ⟶ 2KNO3 + Mg(OH)2

е) Al(OH)3 + KCl ⇸ реакция не идёт, т. к. соли не реагирую с нерастворимыми основаниями.

ж) K2CO3 + 2HNO3 ⟶ 2KNO3 + H2O + CO2

з) NaCl + H2S ⇸ реакция не идёт, т. к. более слабая кислота не может вытеснить более сильную кислоту из её соли.

6. Приведите пример растворимой в воде соли, при обработке которой как сульфатом калия, так и нитратом серебра образуются осадки. Напишите уравнения этих реакций в молекулярном, полном и сокращённом ионном виде.

Ответ:

BaCl2 + K2SO4 ⟶ 2KCl + BaSO4
Ba2+ + 2Cl- + 2K+ + SO42- ⟶ 2K+ + 2Cl- + BaSO4
Ba2+ + SO42- ⟶ BaSO4

BaCl2 + 2AgNO3 ⟶ Ba(NO3)2 + 2AgCl↓
Ba2+ + 2Cl- + 2Ag+ + 2NO3- ⟶ Ba2+ + 2NO3- + 2AgCl↓
Ag+ + Cl- ⟶ AgCl↓

7. Назовите две растворимые в воде соли разных кислот, при обработке которых сильной кислотой выделяются газообразные продукты.

Ответ:

Na2CO3 + 2HCl ⟶ 2NaCl + H2O + CO2

Na2SO3 + 2HCl ⟶ 2NaCl + H2O + SO2

8. Допишите уравнения реакций:
а) MgSO4 + … = …↓ + K2SO4;
б) … + НCl = …↓ + HNO3;
в) FeS + … = FeCl2 + …↑;
г) … + … = Fe(OH)2↓ + BaSO4↓.

Напишите уравнения этих реакций в полном и сокращённом ионном виде.

Ответ:

а) MgSO4 + 2KOH ⟶ Mg(OH)2↓ + K2SO4
Mg2+ + SO42- + 2K+ + 2OH- ⟶ Mg(OH)2↓ + 2K+ + SO42-
Mg2+ + 2OH- ⟶ Mg(OH)2

б) AgNO3 + НCl ⟶ AgCl↓ + HNO3
Ag+ + NO3- + Н+ + Cl- ⟶ AgCl↓ + H+ + NO3-
Ag+ + Cl- ⟶ AgCl↓

в) FeS + 2H+ + 2Cl- ⟶ Fe2+ + 2Cl- + H2S↑
FeS + 2H+ + 2Cl- ⟶ Fe2+ + 2Cl- + H2S↑
FeS + 2H+ ⟶ Fe2+ + H2S↑

г) FeSO4 + Ba(OH)2 ⟶ Fe(OH)2↓ + BaSO4
Fe2+ + SO42- + Ba2+ + 2OH- ⟶ Fe(OH)2↓ + BaSO4

9. Выпишите формулы веществ, с которыми реагирует гидроксид калия в водном растворе: HCl, NaNO3, Ca(OH)2, MgCl2. Напишите уравнения реакций в молекулярном, полном и сокращённом ионном виде.

Ответ:

Гидроксид калия в водном растворе реагирует с: HCl, MgCl2.

KOH + HCl ⟶ KCl + H2O
K+ + OH- + H+ + Cl- ⟶ K+ + Cl- + H2O
H+ + OH- ⟶ H2O

2KOH + MgCl2 ⟶ 2KCl + Mg(OH)2
2K+ + 2OH- + Mg2+ + 2Cl- ⟶ 2K+ + 2Cl- + Mg(OH)2
Mg2+ + 2OH- ⟶ Mg(OH)2

10. Выпишите формулы солей, которые реагируют с соляной кислотой: KBr, AgNO3, CaCO3, MgSO4. Напишите уравнения реакций в молекулярном, полном и сокращённом ионном виде.

Ответ:

С соляной кислотой реагируют: AgNO3, CaCO3, MgSO4.

AgNO3 + HCl ⟶ HNO3 + AgCl↓
Ag+ + NO3- + H+ + Cl- ⟶ H+ + NO3- + AgCl↓
Ag+ + Cl- ⟶ AgCl↓

CaCO3 + 2HCl ⟶ CaCl2 + H2O + CO2
CaCO3 + 2H+ + 2Cl- ⟶ Ca2+ + 2Cl- + H2O + CO2
CaCO3 + 2H+ ⟶ Ca2+ + H2O + CO2

11. Используя таблицу растворимости, предложите реактивы для обнаружения следующих ионов: а) SO42-; б) Cl-; в) S2-; г) Ag+.

Ответ:

а) BaCl2

б) AgNO3

в) Cu(NO3)2

г) NaCl

12. Как отличить друг от друга следующие вещества:
a) NaCl и BaCl2;
б) AgNO3 и KNO3;
в) MgCl2 и MgSO4;
г) H2SO4 и CuSO4;
д) Na2CO3 и CaCO3;
е) KNO3 и K2CO3?

Опишите экспериментальные процедуры, которые вы придумали.

Ответ:

Взять образцы веществ, поместить их в пробирки, и растворить их в воде.

а) Прилить к растворам веществ раствор сульфата натрия, в пробирке с хлоридом бария выпадет осадок, а в пробирке с хлоридом натрия изменений наблюдаться не будет.

б) Прилить к растворам веществ раствор хлорида калия, в пробирке с нитратом серебра (I) выпадет осадок, а в пробирке с нитратом калия изменений наблюдаться не будет.

в) Прилить к растворам веществ раствор хлорида бария, в пробирке с сульфатом магния выпадет осадок, а в пробирке с хлоридом магния изменений наблюдаться не будет.

г) Прилить к растворам веществ раствор гидроксида натрия, в пробирке с сульфатом меди (II) выпадет осадок, а пробирка с серной кислотой нагреется.

д) При добавлении воды карбонат натрия растворится, а карбонат кальция – нет.

е) Прилить к растворам веществ раствор хлорида кальция, в пробирке с карбонатом калия выпадет осадок, а в пробирке с нитратом калия изменений наблюдаться не будет.

Лабораторный опыт 3. Реакции обмена в растворах электролитов.

1. Налейте в пробирку 1 мл раствора хлорида кальция и 1 мл раствора карбоната натрия. Что наблюдаете? Составьте уравнение реакции, напишите полное и сокращённое ионные уравнения.

2. Налейте в пробирку 1 мл раствора карбоната натрия и аккуратно прилейте к нему несколько капель соляной кислоты. Что происходит? Составьте уравнение реакции, напишите полное и сокращённое ионные уравнения.

3. Налейте в пробирку 1 мл раствора гидроксида натрия, добавьте несколько капель фенолфталеина Какова окраска раствора? Добавляйте соляную кислоту до полного исчезновения окраски. Составьте уравнение реакции, напишите полное и сокращенное ионные уравнения.

4. К раствору хлорида натрия прилейте раствор нитрата калия. Наблюдаются ли какие-либо изменения? Составьте полное ионное уравнение Почему в данном случае реакция невозможна? Какие ионы находятся в растворе?

Ответ:

1

При добавлении раствора карбоната натрия к раствору хлорида кальция наблюдается выпадение белого осадка:
CaCl2 + Na2CO3 ⟶ 2NaCl + CaCO3
Ca2+ + 2Cl- + 2Na+ + CO32- ⟶ 2Na+ + 2Cl- + CaCO3
Ca2+ + CO32- ⟶ CaCO3

2

При добавлении соляной кислоты к раствору карбоната натрия наблюдается выделение бесцветного газа:
Na2CO3 + 2HCl ⟶ 2NaCl + H2O + CO2
2Na+ + CO32- + 2H+ + 2Cl- ⟶ 2Na+ + 2Cl- + H2O + CO2
2H+ + CO32- ⟶ H2O + CO2

3

При добавлении фенолфталеина к раствору гидроксида натрия наблюдается окрашивание раствора в малиновый цвет.
NaOH + HCl ⟶ NaCl + H2O
Na+ + OH- + H+ + Cl- ⟶ Na+ + Cl- + H2O
H+ + OH- ⟶ H2O

4

При добавлении раствора нитрата калия к раствору хлорида натрия изменений не наблюдается.
NaCl + KNO3 ⇸ KCl + NaNO3
Na+ + Cl- + K+ + NO3- ⇸ K+ + Cl- + Na+ + NO3-
Реакция невозможна, потому что не образуется осадка, газа или воды.

§14

§14. Гидролиз солей

1. Что называют гидролизом солей?

Ответ:

Взаимодействие солей с водой называют гидролизом солей.

2. Приведите по два примера солей, растворы которых имеют среду: а) нейтральную; б) кислотную; в) щелочную. В какой цвет окрашен лакмус в этих растворах?

Ответ:

а) Хлорид натрия, нитрат калия. Лакмус окрашен в фиолетовый цвет.

б) Сульфат меди, нитрат серебра. Лакмус окрашен в красный цвет.

в) Карбонат натрия, сульфид калия. Лакмус окрашен в синий цвет.

3. Назовите соль калия, раствор которой имеет щелочную среду, и соль азотной кислоты, раствор которой имеет кислотную среду. Напишите уравнения реакций, которые происходят в растворах этих солей. Какая среда в растворе нитрата калия?

Ответ:

Растворы солей калия, которые имеют щелочную среду: карбонат калия K2CO3, метасиликат калия K2SiO3, сульфид калия K2S, сульфит калия K2SO3.

Растворы солей азотной кислоты, которые имеют кислотную среду: нитрат алюминия Al(NO3)3, нитрат аммония NH4NO3, нитрат магния Mg(NO3)2, нитрат меди (II) Cu(NO3)2, нитрат свинца (II) Pb(NO3)2, нитрат серебра (I) AgNO3, нитрат цинка Zn(NO3)2.

Выбирайте любые соли. Если гидролиз соли протекает в несколько стадий, то достаточно написать уравнения реакций гидролиза соли по первой стадии.
В растворе нитрата калия нейтральная среда, так как эта соль образована сильным основанием и сильной кислотой.

4. Почему раствор стиральной соды Na2CO3 имеет сильнощелочную среду, а раствор питьевой соды NaHCO3 – слабощелочную?

Ответ:

Гидролиз карбоната натрия протекает в две стадии, в результате которых образуются гидроксид-ионы:

Na2CO3 + HOH ⇄ NaHCO3 + NaOH
2Na+ + CO32- + HOH ⇄ Na+ + HCO3- + Na+ + OH-
CO32- + HOH ⇄ HCO3- + OH-

NaHCO3 + HOH ⇄ H2CO3 + NaOH
Na+ + HCO3- + HOH ⇄ H2CO3 + Na+ + OH-
HCO3- + HOH ⇄ H2CO3 + OH-

А гидролиз гидрокарбоната натрия протекает в одну стадии, в результате которой образуются гидроксид-ионы:

NaHCO3 + HOH ⇄ H2CO3 + NaOH
Na+ + HCO3- + HOH ⇄ H2CO3 + Na+ + OH-
HCO3- + HOH ⇄ H2CO3 + OH-

Поэтому раствор стиральной соды Na2CO3 имеет сильнощелочную среду, а раствор питьевой соды NaHCO3 – слабощелочную среду.

5. Напишите уравнения гидролиза фосфата калия и хлорида меди (II).

Ответ:

Фосфат калия представляет собой среднюю соль, образованную слабой кислотой – фосфорной (H3PO4) и сильным основанием – гидроксидом калия (KOH):

K3PO4 ↔ 3K+ + PO43-.

Подвергается гидролизу по аниону. Характер среды водного раствора фосфата калия – щелочной. Уравнение гидролиза будет выглядеть следующим образом:

Молекулярное уравнение
K3PO4 + H2O ↔ K2HPO4 + KOH
Полное ионное уравнение
3K+ + PO43-+ H2O ↔ HPO42- + 3K+ + OH- 
Сокращенное ионное уравнение
PO43-+ H2O ↔ HPO42- + OH- 

CuCl2 — соль образованная слабым основанием и сильной кислотой, поэтому реакция гидролиза протекает по катиону.

Молекулярное уравнение
CuCl2 + HOH ⇄ CuOHCl + HCl
Полное ионное уравнение
Cu2+ + 2Cl- + HOH ⇄ CuOH+ + Cl- + H+ + Cl-
Сокращенное ионное уравнение
Cu2+ + HOH ⇄ CuOH+ + H+

6. В трёх пробирках находятся растворы сульфата алюминия, фосфата калия и хлорида натрия. Можно ли их различить при помощи лакмуса? Назовите окраску индикатора в каждом из растворов и объясните причину этого.

Ответ:

Данные растворы можно разлить с помощью лакмуса.

Раствор сульфата алюминия лакмус окрасит в красный цвет, потому что эта соль образована слабым основанием и сильной кислотой. В результате гидролиза такой соли образуются ионы водорода.

Раствор фосфата калия лакмус окрасит в синий цвет, потому что эта соль образована сильным основанием и слабой кислотой. В результате гидролиза такой соли образуются гидроксид-ионы.

Раствор хлорида натрия не изменит окраску лакмуса, потому что эта соль образована сильным основанием и сильной кислотой.

7. Какую среду имеют водные растворы солей: а) сульфата калия, карбоната калия, сульфида калия; б) хлорида меди (II), нитрата меди (II), сульфата алюминия? Напишите уравнения гидролиза.

Ответ:

а) Сульфат калия K2SO4 образован сильным основанием и сильной кислотой, эта соль не подвержена гидролизу, среда раствора остается нейтральна и лакмус не меняет цвет.

Карбонат калия (K2CO3) диссоциирует в растворе K2CO3 ⇄ 2K+ + CO32-, где
CO32- ― анион слабой кислоты и гидролизирует по схеме (вторая стадия практически не происходит):
CO32- + HOH ⇄ HCO3- + OH-      гидролиз по аниону, среда щелочная, лакмус синий.
CO32- + 2K+ + H2O ⇄ HCO3- + 2K+ + OH- 
K2CO3 + H2O ⇄ KHCO3 + KOH

Сульфид калия K2S диссоциирует в растворе K2S ⇄ 2K+ + S2-, где
S2- ― анион слабой кислоты и гидролизирует по схеме (вторая стадия практически не происходит):
S2- + HOH ⇄ HS- + OH-      гидролиз по аниону, среда щелочная, лакмус синий.
S2- + 2K+ + H2O ⇄ HS- + 2K+ + OH- 
K2S + H2O ⇄ KHS + KOH

б) Хлорид меди (II) CuCl2 диссоциирует в растворе CuCl2 ⇄ Cu2+ + 2Cl-, где
Cu2+ ― катион слабого основания и гидролизирует по схеме (вторая стадия практически не происходит):
Cu2+ + HOH ⇄ CuOH+ + H+    гидролиз по катиону, среда кислотная, лакмус красный.
Cu2+ + 2Cl- + H2O ⇄ CuOH+ + 2Cl- + H+  
CuCl2 + H2O ⇄ Cu(OH)Cl + HCl

Нитрат меди (II) Cu(NO3)2  диссоциирует в растворе Cu(NO3)2 ⇄ Cu2+ + 2NO3-, где
Cu2+ ― катион слабого основания и гидролизирует по схеме (вторая стадия практически не происходит):
Cu2+ + HOH ⇄ CuOH+ + H+     Гидролиз по катиону, среда кислотная, лакмус красный.
Cu2+ + 2NO3- + H2O ⇄ CuOH+ + 2NO3- + H+  
Cu(NO3)2 + H2O ⇄ Cu(OH)NO3 + HNO3

Сульфат алюминия Al2(SO4)3 диссоциирует в растворе Al2(SO4)3 ⇄ 2Al3+ + 3SO42-, где
Al3+ ― катион слабого основания и гидролизирует по схеме (вторая стадия практически не происходит):
Al3+ + HOH ⇄ AlOH2+ + H+    гидролиз по катиону, среда кислотная, лакмус красный.
3Al3+ + 3SO42- + 2H2O ⇄ AlOH2+ + 3SO42- + H+  
2Al(SO4)3 + 2H2O ⇄ 2AlOHSO4 + H2SO4

8. Имеются три раствора солей одинаковой концентрации: Na2CO3, NaCl, FeCl3. Расположите их в порядке увеличения pH раствора. Ответ подтвердите ионными уравнениями.

Ответ:

Растворы в порядке увеличения pH: FeCl3, NaCl, Na2CO3.

FeCl3 + HOH ⇄ FeOHCl2 + HCl
Fe3+ + 3Cl- + HOH ⇄ FeOH2+ + 2Cl- + H+ + Cl-
Fe3+ + HOH ⇄ FeOH2+ + H+

NaCl + HOH ⇄ NaOH + HCl
Na+ + Cl- + HOH ⇄ Na+ + OH- + H+ + Cl-
HOH ⇄ H+ + OH-

Na2CO3 + HOH ⇄ NaHCO3 + NaOH
2Na+ + CO32- + HOH ⇄ Na+ + HCO3- + Na+ + OH-
CO32- + HOH ⇄ HCO3- + OH-

Лабораторный опыт 4. Гидролиз солей.

1. К 1 мл раствора нитрата калия прилейте фиолетовый раствор лакмуса. Изменяется ли окраска? Испытайте раствор нитрата калия другими выданными вам индикаторами и запишите в тетради их окраску.

2. Проделайте аналогичный опыт с растворами карбоната калия и хлорида цинка. Какую среду имеют растворы этих солей? Напишите уравнения реакций.

3. К раствору хлорида алюминия прилейте раствор карбоната натрия. Что наблюдается? Какое вещество выделяется в виде газа, а какое выпадает в осадок? Объясните наблюдаемые явления.

Ответ:

1. При добавлении раствора лакмуса к раствору нитрата калия фиолетовая окраска индикатора не изменяется. Окраска других индикаторов также не изменяется.

2. При добавлении раствора лакмуса к раствору карбоната калия фиолетовая окраска индикатора изменяется на синюю, значит раствор имеет имеет щелочную среду.
K2CO3 + H2O ⇄ KOH + KHCO3
2K+ + CO32- + H2O ⇄ K+ + OH- + K+ + HCO3-
CO32- + H2O ⇄ OH- + HCO3-

При добавлении раствора лакмуса к раствору хлорида цинка фиолетовая окраска индикатора изменяется на красную, значит раствор имеет имеет кислую среду.
ZnCl2 + H2O ⇄ ZnOHCl + HCl
Zn2+ + 2Cl- + H2O ⇄ ZnOH+ + Cl- + H+ + Cl-
Zn2+ + H2O ⇄ ZnOH+ + H+

3. При добавлении раствора карбоната натрия к раствору хлорида алюминия наблюдается выпадение осадка и выделение газа.

В результате реакции между карбонатом натрия и хлоридом алюминия должен образоваться осадок карбоната алюминия, однако его образование не происходит, так как карбоната алюминия взаимодействует с водой.
2AlCl3 + 3Na2CO3 + 3H2O ⟶ 6NaCl + 2Al(OH)3↓ + 3CO2
2Al3+ + 6Cl- + 6Na+ + 3CO32- + 3H2O ⟶ 6Na+ + 6Cl- + 2Al(OH)3↓ + 3CO2
2Al3+ + 3CO32- + 3H2O ⟶ 2Al(OH)3↓ + 3CO2

В осадок выпадает гидроксид алюминия, а в виде газа выделяется оксид углерода (IV).

§15

§15. Окисление и восстановление

1. Какие реакции называют окислительно-восстановительными? Дайте определения понятий "окислитель", "восстановитель", "окисление", "восстановление" с позиций электронной теории.

Ответ:

Реакции, в которых изменяются степени окисления некоторых элементов, называют окислительно-восстановительными.

Окислитель – элемент, который принимает электроны.

Восстановитель – элемент, который теряет электроны.

Окисление – процесс потери электронов, сопровождающийся увеличением степени окисления.

Восстановление – процесс присоединения электронов, сопровождающийся уменьшением степени окисления.

2. Какую величину называют степенью окисления? Основываясь на положении элементов в Периодической системе, обоснуйте расстановку степеней окисления в формулах веществ, приведённых ниже:

`overset(0)(H)_2`, `overset(+1)(H)_2overset(-2)(O)`, `overset(0)(O)_2`, `overset(+2)(Ca)overset(-2)(O)`, `overset(+1)(Na)_2overset(-2)(O)`, `overset(0)(Cl)_2`, `overset(+1)(H)overset(-1)(Cl)`, `overset(+1)(Na)overset(-1)(Cl)`, `overset(+3)(Al)overset(-1)(Cl)_3`, `overset(+1)(Cl)_2overset(-2)(O)`, `overset(+1)(H)_2overset(-2)(S)`, `overset(+2)(Mg)overset(-2)(S)`, `overset(0)(Al)`, `overset(+3)(Al)overset(-3)(P)`.

Ответ:

Степень окисления — это условный заряд атомов химического элемента в соединении, вычисленный из предположения, что все ковалентные полярные связи превратились в ионные.

`overset(0)(H)_2` – водород – простое вещество, а значит не содержит ковалентных полярных связей, поэтому он имеет нулевую степень окисления.

`overset(+1)(H)_2overset(-2)(O)` – водород находится в I группе, главной подгруппе, на внешнем уровне имеет 1ē, кислород находится в VI группе, главной подгруппе, на внешнем уровне имеет 6ē. Для получения завершенного внешнего уровня водороду необходимо принять или отдать 1ē. Для получения завершенного внешнего уровня кислороду необходимо принять 2ē или отдать 6ē. Кислороду проще принять 2ē, чем отдать 6ē, поэтому кислород принимает по 1ē от двух атомов водорода.

`overset(0)(O)_2` – кислород – простое вещество, а значит не содержит ковалентных полярных связей, поэтому он имеет нулевую степень окисления.

`overset(+2)(Ca)overset(-2)(O)` – кальций находится в II группе, главной подгруппе, на внешнем уровне имеет 2ē, кислород находится в VI группе, главной подгруппе, на внешнем уровне имеет 6ē. Для получения завершенного внешнего уровня кальцию необходимо принять 6ē или отдать 2ē. Для получения завершенного внешнего уровня кислороду необходимо принять 2ē или отдать 6ē. Кальцию проще отдать 2ē, чем принять 6ē, а кислороду проще принять 2ē, чем отдать 6ē, поэтому магний отдаёт 2ē кислороду.

`overset(+1)(Na)_2overset(-2)(O)` – натрий находится в I группе, главной подгруппе, на внешнем уровне имеет 1ē, кислород находится в VI группе, главной подгруппе, на внешнем уровне имеет 6ē. Для получения завершенного внешнего уровня натрию необходимо принять 7ē или отдать 1ē. Для получения завершенного внешнего уровня кислороду необходимо принять 2ē или отдать 6ē. Натрию проще отдать 1ē, чем принять 7ē, а кислороду проще принять 2ē, чем отдать 6ē, поэтому два атома натрия отдают по 1ē кислороду.

`overset(0)(Cl)_2` – хлор – простое вещество, а значит не содержит ковалентных полярных связей, поэтому он имеет нулевую степень окисления.

`overset(+1)(H)overset(-1)(Cl)` – водород находится в I группе, главной подгруппе, на внешнем уровне имеет 1ē, хлор находится в VII группе, главной подгруппе, на внешнем уровне имеет 7ē. Для получения завершенного внешнего уровня водороду необходимо принять или отдать 1ē. Для получения завершенного внешнего уровня хлору необходимо принять 1ē или отдать 7ē. Хлору проще принять 1ē, чем отдать 7ē, поэтому хлор принимает 1ē от водорода.

`overset(+1)(Na)overset(-1)(Cl)` – натрий находится в I группе, главной подгруппе, на внешнем уровне имеет 1ē, хлор находится в VII группе, главной подгруппе, на внешнем уровне имеет 7ē. Для получения завершенного внешнего уровня натрию необходимо принять 7ē или отдать 1ē. Для получения завершенного внешнего уровня хлору необходимо принять 1ē или отдать 7ē. Натрию проще отдать 1ē, чем принять 7ē, а кислороду проще принять 1ē, чем отдать 7ē, поэтому натрий отдаёт 1ē хлору.

`overset(+3)(Al)overset(-1)(Cl)_3` – алюминий находится в III группе, главной подгруппе, на внешнем уровне имеет 3ē, хлор находится в VII группе, главной подгруппе, на внешнем уровне имеет 7ē. Для получения завершенного внешнего уровня алюминию необходимо принять 5ē или отдать 3ē. Для получения завершенного внешнего уровня хлору необходимо принять 1ē или отдать 7ē. Алюминию проще отдать 3ē, чем принять 5ē, а хлору проще принять 1ē, чем отдать 7ē, поэтому алюминий отдаёт 3ē трём атомам хлора.

`overset(+1)(Cl)_2overset(-2)(O)` – хлор находится в VII группе, главной подгруппе, на внешнем уровне имеет 7ē, кислород находится в VI группе, главной подгруппе, на внешнем уровне имеет 6ē. Для получения завершенного внешнего уровня хлору необходимо принять 1ē или отдать 7ē. Для получения завершенного внешнего уровня кислороду необходимо принять 2ē или отдать 6ē. Кислород является более электроотрицательным элементом, чем хлор, поэтому два атома хлора отдают по 1ē атому кислорода.

`overset(+1)(H)_2overset(-2)(S)` – водород находится в I группе, главной подгруппе, на внешнем уровне имеет 1ē, сера находится в VI группе, главной подгруппе, на внешнем уровне имеет 6ē. Для получения завершенного внешнего уровня водороду необходимо принять или отдать 1ē. Для получения завершенного внешнего уровня сере необходимо принять 2ē или отдать 6ē. Сере проще принять 2ē, чем отдать 6ē, поэтому кислород принимает по 1ē от двух атомов водорода.

`overset(+2)(Mg)overset(-2)(S)` – магний находится в II группе, главной подгруппе, на внешнем уровне имеет 2ē, сера находится в VI группе, главной подгруппе, на внешнем уровне имеет 6ē. Для получения завершенного внешнего уровня магнию необходимо принять 6ē или отдать 2ē. Для получения завершенного внешнего уровня сере необходимо принять 2ē или отдать 6ē. Магнию проще отдать 2ē, чем принять 6ē, а сере проще принять 2ē, чем отдать 6ē, поэтому магний отдаёт 2ē сере.

`overset(0)(Al)` – алюминий – простое вещество, а значит не содержит ковалентных полярных связей, поэтому он имеет нулевую степень окисления.

`overset(+3)(Al)overset(-3)(P)` – алюминий находится в III группе, главной подгруппе, на внешнем уровне имеет 3ē, фосфор находится в V группе, главной подгруппе, на внешнем уровне имеет 5ē. Для получения завершенного внешнего уровня алюминию необходимо принять 5ē или отдать 3ē. Для получения завершенного внешнего уровня фосфору необходимо принять 3ē или отдать 5ē. Алюминию проще отдать 3ē, чем принять 5ē, а фосфору проще принять 3ē, чем отдать 5ē, поэтому алюминий отдаёт 3ē фосфору.

3. По формулам веществ определите степени окисления: а) водорода; б) кислорода; в) хлора; г) азота; д) серы; е) железа; ж) марганца.

а) H2, H2O, HCl, NaOH, NaH;

б) H2O, NO2, H2O2, O3;

в) HCl, Cl2, CuCl2, KClO3;

г) N2, NH3, NO2, HNO3;

д) SO2, S, H2SO4, H2S;

е) FeO, Fe2O3, Fe(OH)2, Fe2(SO4)3;

ж) KMnO4, K2MnO4, MnO2, MnSO4.

Ответ:

а) `overset(0)(H)_2`, `overset(+1)(H)_2O`, `overset(+1)(H)Cl`, `NaOoverset(+1)(H)`, `Naoverset(-1)(H)`;

б) `H_2overset(-2)(O)`, `Noverset(-2)(O)_2`, `H_2overset(-1)(O)_2`, `overset(0)(O)_3`;

в) `Hoverset(-1)(Cl)`, `overset(0)(Cl)_2`, `Cuoverset(-1)(Cl)_2`, `Koverset(+5)(Cl)O_3`;

г) `overset(0)(N)_2`, `overset(-3)(N)H_3`, `overset(+4)(N)O_2`, `Hoverset(+5)(N)O_3`;

д) `overset(+4)(S)O_2`, `overset(0)(S)`, `H_2overset(+6)(S)O_4`, `H_2overset(-2)(S)`;

е) `overset(+2)(Fe)O`, `overset(+3)(Fe)_2O_3`, `overset(+2)(Fe)(OH)_2`, `overset(+3)(Fe)_2(SO_4)_3`;

ж) `Koverset(+7)(Mn)O_4`, `K_2overset(+6)(Mn)O_4`, `overset(+4)(Mn)O_2`, `overset(+2)(Mn)SO_4`.

4. Определите степени окисления всех элементов по формулам веществ: HBr, K2O, Na2S, FeCl3, P2O5, Ca3P2, F2, CO2, SO3, SF6.

Ответ:

`overset(+1)(H)overset(-1)(Br)`, `overset(+1)(K)_2overset(-2)(O)`, `overset(+1)(Na)_2overset(-2)(S)`, `overset(+3)(Fe)overset(-1)(Cl)_3`, `overset(+5)(P)_2overset(-2)(O)_5`, `overset(+2)(Ca)_3overset(-3)(P)_2`, `overset(0)(F)_2`, `overset(+4)(C)overset(-2)(O)_2`, `overset(+6)(S)overset(-2)(O)_3`, `overset(+6)(S)overset(-1)(F)_6`.

5. Ниже приведены уравнения реакций с участием соляной кислоты:
a) NaOH + HCl = NaCl + H2O;
б) Fe + 2HCl = FeCl2 + H2↑;
в) CuO + 2HCl = CuCl2 + H2O;
г) MnO2 + 4HCl = MnCl2 + Cl2↑ + 2H2O.

Какие из этих реакций относятся к окислительно-восстановительным?

Ответ:

б) Fe + 2HCl = FeCl2 + H2

Fe0 - 2ē ⟶ Fe+2 |2| 2 |1    восстановитель (окисление)
2H+ + 2ē ⟶ H20 |2| |1    окислитель (восстановление)

​г) MnO2 + 4HCl = MnCl2 + Cl2↑ + 2H2O

2Cl- - 2ē ⟶ Cl20 |2| 2 |1    восстановитель (окисление)
Mn+4 + 2ē ⟶ Mn+2 |2| |1    окислитель (восстановление)

6. Из приведённого ниже списка выпишите полуреакции: а) окисления; б) восстановления. Дополните схемы этих полуреакций, указав число электронов:

а) `overset(+2)(Fe) ⟶ overset(+3)(Fe)`
б) `overset(+2)(Fe) ⟶ overset(0)(Fe)`
в) `overset(+5)(N) ⟶ overset(+2)(N)`
г) `overset(0)(N_2) ⟶ overset(+2)(2N)`
д) `overset(0)(O_2) ⟶ overset(-2)(2O)`
е) `overset(+7)(Mn) ⟶ overset(+2)(Mn)`
ж) `overset(+7)(Mn) ⟶ overset(+4)(Mn)`
з) `overset(-3)(2N) ⟶ overset(0)(N_2)`
и) `overset(-3)(N) ⟶ overset(+2)(N)`

Ответ:

а) полуреакции окисления

а) `overset(+2)(Fe) - 1ē ⟶ overset(+3)(Fe)`

г) `overset(0)(N_2) - 4ē ⟶ overset(+2)(2N)`

з) `overset(-3)(2N) - 6ē ⟶ overset(0)(N_2)`

и) `overset(-3)(N) - 5ē ⟶ overset(+2)(N)`

б) полуреакции восстановления

б) `overset(+2)(Fe) + 2ē ⟶ overset(0)(Fe)`

в) `overset(+5)(N) + 3ē ⟶ overset(+2)(N)`

д) `overset(0)(O_2) + 4ē ⟶ overset(-2)(2O)`

е) `overset(+7)(Mn) + 5ē ⟶ overset(+2)(Mn)`

ж) `overset(+7)(Mn) + 3ē ⟶ overset(+4)(Mn)`

7. Определите окислители и восстановители в следующих реакциях:
а) N2 + O2 = 2NO;
б) 3Mg + N2 = Mg3N2;
в) Cl2 + 2KBr = 2KCl + Br2;
г) Fe + CuSO4 = FeSO4 + Cu;
д) 2Na + 2H2O = 2NaOH + H2↑;
е) 2Pb(NO3)2 = 2PbO + 4NO2 + O2;
ж) Fe + H2SO4 = FeSO4 + H2↑;
з) 5Cl2 + 2P = 2PCl5.

В каких реакциях: а) все элементы изменяют степень окисления; б) есть элементы, которые не изменяют степень окисления?

Ответ:

а) N2 + O2 = 2NO
N20 - 4ē ⟶ 2N+2 |4| 4 |1    восстановитель (окисление)
O20 + 4ē ⟶ 2O-2 |4| |1    окислитель (восстановление)
б) 3Mg + N2 = Mg3N2
Mg0 - 2ē ⟶ Mg+2 |2| 6 |3    восстановитель (окисление)
N20 + 6ē ⟶ 2N-3 |6| |1    окислитель (восстановление)
в) Cl2 + 2KBr = 2KCl + Br2
2Br0 - 2ē ⟶ Br20 |2| 2 |1    восстановитель (окисление)
Cl20 + 2ē ⟶ 2Cl- |2| |1    окислитель (восстановление)
г) Fe + CuSO4 = FeSO4 + Cu
Fe0 - 2ē ⟶ Fe+2 |2| 2 |1    восстановитель (окисление)
Cu+2 + 2ē ⟶ Cu0 |2| |1    окислитель (восстановление)
д) 2Na + 2H2O = 2NaOH + H2
Na0 - 1ē ⟶ Na+ |1| 2 |2    восстановитель (окисление)
2H+ + 2ē ⟶ H20 |2| |1    окислитель (восстановление)
е) 2Pb(NO3)2 = 2PbO + 4NO2 + O2
2O-2 - 4ē ⟶ O20 |4| 4 |1    восстановитель (окисление)
N+5 + 1ē ⟶ N+4 |1| |4    окислитель (восстановление)
ж) Fe + H2SO4 = FeSO4 + H2
Fe0 - 2ē ⟶ Fe+2 |2| 2 |1    восстановитель (окисление)
2H+ + 2ē ⟶ H20 |2| |1    окислитель (восстановление)
з) 5Cl2 + 2P = 2PCl5
P0 - 5ē ⟶ P+5 |5| 10 |2    восстановитель (окисление)
Cl2+4 + 2ē ⟶ 2Cl- |2| |5    окислитель (восстановление)

а) а, б, з.
б) в, г, д, е, ж.

8. Определите окислитель и восстановитель в реакции восстановления оксида меди (II) водородом. Напишите уравнения полуреакций окисления и восстановления.

Ответ:

CuO + H2 ⟶ Cu + H2O

H20 - 2ē ⟶ 2H+ |2| 2 |1    восстановитель (окисление)
Cu+2 + 2ē ⟶ Cu0 |2| |1    окислитель (восстановление)

9. Определите окислитель и восстановитель в реакции магния с кислородом. Напишите уравнения полуреакций окисления и восстановления.

Ответ:

2Mg + O2 ⟶ 2MgO

Mg0 - 2ē ⟶ C+2 |2| 4 |2    восстановитель (окисление)
O20 + 4ē ⟶ 2O-2 |4| |1    окислитель (восстановление)

10. В каких соединениях в ряду: H2S, FeS, SO2, K2SO3, H2SO4, Fe2(SO4)3 – сера может быть: а) только окислителем; б) только восстановителем; в) окислителем в одних реакциях, восстановителем в других?

Ответ:

а) H2SO4, Fe2(SO4)3 – сера в соединениях может являться только окислителем, потому что она имеет максимальную степень окисления +6, т. е. сера больше не может отдавать электроны, а может только принимать их.

б) H2S, FeS – сера в соединениях может являться только восстановителем, потому что она имеет минимальную степень окисления -2, т. е. сера больше не может принимать электроны, а может только отдавать их.

в) SO2, K2SO3 – сера в соединениях может являться и окислителем, и восстановителем в разных ОВР, потому что она имеет промежуточную степень окисления +4, т. е. сера может как принимать электроны, так и отдавать их.

11. 

Самая распространённая окислительно-восстановительная реакция на Земле – фотосинтез: 6CO2 + 6H2O = C6H12O6 + 6O2.

Какой элемент служит окислителем, а какой – восстановителем в этом процессе?

Ответ:

В фотосинтезе углерод служит окислителем, а кислород – восстановителем.

Условно ОВР можно записать таким образом:

6CO2 + 6H2O = C6H12O6 + 6O2

2O-2 - 4ē ⟶ O2 |4  | 24 |6    восстановитель (окисление)
6C+4 + 24ē ⟶ 6C0 |24| |1    окислитель (восстановление)

§16

§16. Составление уравнений окислительно-восстановительных реакций

1. Напишите уравнение горения кальция в кислороде. Какое вещество теряет, а какое принимает электроны? Назовите окислитель и восстановитель.

Ответ:

2Ca + O2 ⟶ 2CaO

Ca0 - 2ē ⟶ Ca+2 |2| 4 |2    восстановитель (окисление)
O20 + 4ē ⟶ 2O-2 |4| |1    окислитель (восстановление)

2. Напишите уравнения реакций восстановления водородом оксида свинца (II), оксида меди (I), оксида азота (IV) до простых веществ. Расставьте в них коэффициенты, пользуясь методом электронного баланса. Назовите окислители и восстановители.

Ответ:

PbO + H2 ⟶ Pb + H2O

H20 - 2ē ⟶ 2H+ |2| 2 |1    восстановитель (окисление)
Pb+2 + 2ē ⟶ Pb0 |2| |1    окислитель (восстановление)

Cu2O + H2 ⟶ 2Cu + H2O

H20 - 2ē ⟶ 2H+ |2| 2 |1    восстановитель (окисление)
Cu+ + 1ē ⟶ Cu0 |1| |2    окислитель (восстановление)

2NO2 + 4H2 ⟶ N2 + 4H2O

H20 - 2ē ⟶ 2H+ |2| 8 |4    восстановитель (окисление)
2N+4 + 8ē ⟶ N20 |8| |1    окислитель (восстановление)

3. В бромиде алюминия алюминий находится в высшей степени окисления, а бром – в низшей. Могут ли выступать в роли окислителя входящие в состав этого соединения атомы: а) алюминия; б) брома? А в роли восстановителя? Ответ поясните, используя уравнения реакций:
AlBr3 + 3Na = 3NaBr + Al;
2AlBr3 + 3Cl2 = 2AlCl3 + 3Br2.

Ответ:

а) Атомы алюминия в бромиде алюминия находится в высшей степени окисления, а значит алюминий в этом соединении больше не в состоянии отдавать электроны, а может только принимать их, поэтому алюминий в данном соединении может выступать только в роли окислителя.

AlBr3 + 3Na = 3NaBr + Al

Na0 - 1ē ⟶ Na+ |1| 3 |3    восстановитель
Al+3 + 3ē ⟶ Al0 |3| |1    окислитель

б) Атомы брома в бромиде алюминия находится в низшей степени окисления, а значит бром в этом соединении больше не в состоянии принимать электроны, а может только отдавать их, поэтому бром в данном соединении может выступать только в роли восстановителя.

2AlBr3 + 3Cl2 = 2AlCl3 + 3Br2

2Br- - 2ē ⟶ Br20 |2| 2 |1    восстановитель
Cl20 + 2ē ⟶ 2Cl- |2| |1    окислитель

4. 

Составьте уравнения полуреакций окисления и восстановления, назовите атом-окислитель и атом-восстановитель. Используя метод электронного баланса, найдите коэффициенты в схемах следующих реакций:

a) Na + Cl2 ⟶ NaCl;
P + O2 ⟶ P2O5;
HgO ⟶ Hg + O2;
H2S + O2 ⟶ SO2 + H2O;
NH3 + O2 ⟶ N2 + H2O;
б) NO + O2 ⟶ NO2;
Cu(NO3)2 ⟶ CuO + NO2 + O2;
H2O2 + HI ⟶ I2 + H2O;
S + H2SO4(конц.) ⟶ SO2 + H2O.

Ответ:

a)

2Na + Cl2 ⟶ 2NaCl
Na0 - 1ē ⟶ Na+ |1| 2 |2    восстановитель (окисление)
Cl20 + 2ē ⟶ 2Cl- |2| |1    окислитель (восстановление)
4P + 5O2 ⟶ 2P2O5
P0 - 5ē ⟶ P+5 |5| 20 |4    восстановитель (окисление)
O20 + 4ē ⟶ 2O-2 |4| |5    окислитель (восстановление)
2HgO ⟶ 2Hg + O2
2O-2 - 4ē ⟶ O20 |4| 4 |1    восстановитель (окисление)
Hg+2 + 2ē ⟶ Hg0 |2| |2    окислитель (восстановление)
2H2S + 3O2 ⟶ 2SO2 + 2H2O
S-2 - 6ē ⟶ S+4 |6| 12 |2    восстановитель (окисление)
O20 + 4ē ⟶ 2O-2 |4| |3    окислитель (восстановление)
4NH3 + 3O2 ⟶ 2N2 + 6H2O
2N-3 - 6ē ⟶ N20 |6| 12 |2    восстановитель (окисление)
O20 + 4ē ⟶ 2O-2 |4| |3    окислитель (восстановление)

б)

2NO + O2 ⟶ 2NO2
N+2 - 2ē ⟶ N+4 |2| 4 |2 восстановитель (окисление)
O20 + 4ē ⟶ 2O-2 |4| |1 окислитель (восстановление)
2Cu(NO3)2 ⟶ 2CuO + 4NO2 + O2
2O-2 - 4ē ⟶ O20 |4| 4 |1 восстановитель (окисление)
N+5 + 1ē ⟶ N+4 |1| |4 окислитель (восстановление)
H2O2 + 2HI ⟶ I2 + 2H2O
2I- - 2ē ⟶ I20 |2| 2 |1 восстановитель (окисление)
O- + 1ē ⟶ O-2 |1| |2 окислитель (восстановление)
S + 2H2SO4(конц.) ⟶ 3SO2 + 2H2O
S0 - 4ē ⟶ S+4 |4| 4 |1 восстановитель (окисление)
S+6 + 2ē ⟶ S+4 |2| |2 окислитель (восстановление)

5. На хлорирование алюминия было затрачено 11,2 л хлора (в пересчёте на н. у.). Рассчитайте массу алюминия, вступившего в реакцию.

Ответ:

Дано:

`V(Cl_2) = 11.2 л`

`m(Al) = ?`

Решение

`2Al + 3Cl_2 ⟶ 2AlCl_3`

`n(Cl_2) = (V(Cl_2))/V_m = 11.2/22.4 = 0.5" моль"`

`n(Al) = (2*n(Cl_2))/3 = (2*0.5)/3 = 1/3" моль"`

`m(Al) = n(Al)*M(Al) = 1/3*27 = 9 г`

Ответ: `m(Al) = 9 г`.

2-й способ

Составим химическое уравнение:
х г     11,2 л
2Al + 3Cl2 = 2AlCl3
54 г   67,2 л
Над формулами соединений Al и Cl2 записываем неизвестную массу алюминия (х г) и приведенный в условии задачи объём хлора (11,2 л), а под формулами соединений ― массу и объём соответствующего количества вещества согласно коэффициентам в химическом уравнении. При н.у. 1 моль любого газа занимает объём 22,4 л, а 3 моль газа ― 67,2 л.
M(Al)=27 г/моль, масса 1 моль=27 г, а масса 2 моль=54 г
х=m(Al)=11,2 л • 54 г : 67,2 л =9 г

Ответ: m(Al)=9 г

6. Иногда элемент-окислитель и элемент-восстановитель входят в состав одного и того же соединения. В этом случае окислительно-восстановительную реакцию называют внутримолекулярной. Примером может служить разложение воды на простые вещества под действием электрического тока. Составьте уравнение реакции, назовите окислитель и восстановитель.

Ответ:

2H2O ⟶ 2H2 + O2

2O-2 - 4ē ⟶ O20 |4| 4 |1    восстановитель
2H+ + 2ē ⟶ H20 |2| |2    окислитель

7. Напишите уравнения реакций горения в кислороде следующих веществ: железа, водорода, фосфора, аммиака NH3, оксида углерода (II), метана CH4. В каждой реакции определите элемент-восстановитель.

Ответ:

4Fe + 3O2 ⟶ 2Fe2O3

Fe0 - 3ē ⟶ Fe+3 |3| 12 |4    восстановитель (окисление)
O20 + 4ē ⟶ 2O-2 |4| |3    окислитель (восстановление)

2H2 + O2 ⟶ 2H2O

H20 - 2ē ⟶ 2H+ |2| 4 |2    восстановитель (окисление)
O20 + 4ē ⟶ 2O-2 |4| |1    окислитель (восстановление)

4P + 5O2 ⟶ 2P2O5

P0 - 5ē ⟶ P+5 |5| 20 |4    восстановитель (окисление)
O20 + 4ē ⟶ 2O-2 |4| |5    окислитель (восстановление)

4NH3 + 3O2 ⟶ 2N2 + 6H2O

2N-3 - 6ē ⟶ N20 |6| 12 |2    восстановитель (окисление)
O20 + 4ē ⟶ 2O-2 |4| |3    окислитель (восстановление)

2CO + O2 ⟶ 2CO2

C+2 - 2ē ⟶ C+4 |2| 4 |2    восстановитель (окисление)
O20 + 4ē ⟶ 2O-2 |4| |1    окислитель (восстановление)

CH4 + 2O2 ⟶ CO2 + 2H2O

C-4 - 8ē ⟶ C+4 |8| 8 |1    восстановитель (окисление)
O20 + 4ē ⟶ 2O-2 |4| |2    окислитель (восстановление)

8. Напишите уравнения реакций восстановления водородом следующих веществ: оксида вольфрама (VI), оксида хрома (III), оксида титана (IV). В каждой реакции определите элемент-окислитель.

Ответ:

WO3 + 3H2 ⟶ W + 3H2O

H20 - 2ē ⟶ 2H+ |2| 6 |3    восстановитель (окисление)
W+6 + 6ē ⟶ W0 |6| |1    окислитель (восстановление)

Cr2O3 + 3H2 ⟶ 2Cr + 3H2O

H20 - 2ē ⟶ 2H+ |2| 6 |3    восстановитель (окисление)
Cr+3 + 3ē ⟶ Cr0 |3| |2    окислитель (восстановление)

TiO2 + 2H2 ⟶ Ti + 2H2O

H20 - 2ē ⟶ 2H+ |2| 4 |2    восстановитель (окисление)
Ti+4 + 4ē ⟶ Ti0 |4| |1    окислитель (восстановление)

9. Железо может быть получено восстановлением оксида железа (III) углеродом, водородом, алюминием. Напишите уравнения реакций. Расставьте коэффициенты методом электронного баланса. Назовите окислитель и восстановитель в каждой реакции.

Ответ:

Fe2O3 + 3C ⟶ 2Fe + 3CO

C0 - 2ē ⟶ C+2 |2| 6 |3    восстановитель (окисление)
Fe+3 + 3ē ⟶ Fe0 |3| |2    окислитель (восстановление)

Fe2O3 + 3H2 ⟶ 2Fe + 3H2O

H20 - 2ē ⟶ 2H+ |2| 6 |3    восстановитель (окисление)
Fe+3 + 3ē ⟶ Fe0 |3| |2    окислитель (восстановление)

Fe2O3 + 2Al ⟶ 2Fe + Al2O3

Al0 - 3ē ⟶ Al+3 |3| 1 |1    восстановитель (окисление)
Fe+3 + 3ē ⟶ Fe0 |3| |1    окислитель (восстановление)

10. При освещении фтор реагирует с инертным газом ксеноном с образованием фторида ксенона (II) XeF2. Напишите уравнение реакции. Сколько граммов фторида ксенона (II) можно получить из 11,2 л (н. у.) ксенона Xe, если выход продукта реакции составляет 20% от теоретически возможного?

Ответ:

Дано:

`V(Xe) = 11.2 л`

`η(XeF_2) = 20%`

`m_"практ."(XeF_2) = ?`

Решение

`Xe + F_2 ⟶ XeF_2`

`n(Xe) = (V(Xe))/V_m = 11.2/22.4 = 0.5" моль"`

`n_"теор."(XeF_2) = n(Xe) = 0.5" моль"`

`m_"теор."(XeF_2) = n_"теор."(XeF_2)*M(XeF_2) = 0.5*169 = 84.5 г`

`m_"практ."(XeF_2) = (η(XeF_2)*m_"теор."(XeF_2))/100 = (20*84.5)/100 = 16.9 г`

Ответ: `m_"практ."(XeF_2) = 16.9 г`.

11. Методом электронного баланса расставьте коэффициенты в следующих схемах реакций:

a) BaSO4 + C ⟶ BaS + CO;
Al + H2SO4 ⟶ Al2(SO4)3 + H2;
KClO3 + P ⟶ KCl + P2O5;
S + KOH ⟶ K2SO3 + K2S + H2O;

б) KClO3 ⟶ KClO4 + KCl;
Na2CO3 + C ⟶ Na + CO.

Ответ:

a)

BaSO4 + 4C ⟶ BaS + 4CO
C0 - 2ē ⟶ C+2 |2| 8 |4    восстановитель (окисление)
S+6 + 8ē ⟶ S-2 |8| |1    окислитель (восстановление)
2Al + 3H2SO4 ⟶ Al2(SO4)3 + 3H2
Al0 - 3ē ⟶ Al+3 |3| 6 |2    восстановитель (окисление)
2H+ + 2ē ⟶ H20 |2| |3    окислитель (восстановление)
5KClO3 + 6P ⟶ 5KCl + 3P2O5
P0 - 5ē ⟶ P+5 |5| 30 |6    восстановитель (окисление)
Cl+5 + 6ē ⟶ Cl- |6| |5    окислитель (восстановление)
3S + 6KOH ⟶ K2SO3 + 2K2S + 3H2O
S0 - 4ē ⟶ S+4 |4| 4 |1    восстановитель (окисление)
S0 + 2ē ⟶ S-2 |2| |2    окислитель (восстановление)

б)

4KClO3 ⟶ 3KClO4 + KCl
Cl+5 - 2ē ⟶ Cl+7 |2| 6 |3    восстановитель (окисление)
Cl+5 + 6ē ⟶ Cl- |6| |1    окислитель (восстановление)
Na2CO3 + 2C ⟶ 2Na + 3CO
C0 - 2ē ⟶ C+2 |2| 4 |2    восстановитель (окисление)
2Na+ + 2ē ⟶ 2Na0 |4| |1    окислитель (восстановление)
C+4 + 2ē ⟶ C+2

Лабораторный опыт 5. Окислительно-восстановительные реакции.

1. В пробирку с раствором сульфата меди (II) опустите гранулу цинка. Что вы наблюдаете? Напишите уравнение реакции, назовите окислитель и восстановитель.

2. К порошку цинка прилейте бромную воду и перемешайте раствор. Если окраска не исчезла, добавьте ещё немного цинка и слегка подогрейте пробирку. Напишите уравнение реакции, назовите окислитель и восстановитель.

Ответ:

1. При опускании гранулы цинка в раствор сульфата меди (II), наблюдается выделение меди на поверхности цинка.

Zn + CuSO4 ⟶ Cu + ZnSO4
Zn0 - 2ē ⟶ Zn+2 |2| 2 |1    восстановитель (окисление)
Cu+2 + 2ē ⟶ Cu0 |2| |1    окислитель (восстановление)

2

Zn + Br2 ⟶ ZnBr2
Zn0 - 2ē ⟶ Zn+2 |2| 2 |1    восстановитель (окисление)
Br20 + 2ē ⟶ 2Br- |2| |1    окислитель (восстановление)

§17

§17. Химические источники тока. Электрохимический ряд напряжений металлов

1. Укажите уравнения реакций, которые можно использовать в химических источниках тока:

a) NaOH + HCl = NaCl + H2O;
б) Fe + 2HCl = FeCl2 + H2↑;
в) CuCl2 + 2NaOH = Cu(OH)2↓ + 2NaCl;
г) CuCl2 + Zn = Cu + ZnCl2.

Ответ:

Для создания электрического тока использовать только ОВР, из данного перечня к ним относятся:
б) Fe + 2HCl = FeCl2 + H2↑;
г) CuCl2 + Zn = Cu + ZnCl2

2. В одном из простейших гальванических элементов протекает реакция: Zn + CuSO4 = ZnSO4 + Cu. Опишите устройство этого элемента и составьте уравнения полуреакций, которые в нём происходят. Назовите катод и анод.

Ответ:

Устройство данного гальванического элемента. В одном стакане налит раствор сульфата цинка (II), с опущенным в него цинковым стержнем, в другом стакане налит раствор сульфата меди (II), с опущенным в него медным стержнем. Стержни соединены проводом, а растворы – трубкой, заполненной раствором соли, чтобы замкнуть цепь. В таком гальваническом элементе электроны движутся от цинка к меди, т. е. протекают процессы окисления цинка, и восстановления меди.

Zn + CuSO4 = ZnSO4 + Cu
Анод: Zn0 - 2ē ⟶ Zn2+
Катод: Cu2+ + 2ē ⟶ Cu0

3. Напишите два уравнения реакций, соответствующих сокращённому ионному уравнению: Zn + Fe2+ = Zn2+ + Fe.

Ответ:

Zn + FeCl2 = ZnCl2 + Fe

Zn + FeSO4 = ZnSO4 + Fe

4. Назовите два металла, которые можно использовать для вытеснения железа из раствора сульфата железа (II).

Ответ:

Для вытеснения железа из раствора сульфата железа (II) можно использовать металлы, находящиеся между манием (включая) и железом в электрохимическом ряду напряжений металлов: Mg, Al, Mn, Zn.

5. Напишите уравнения реакций соляной кислоты с магнием, алюминием, цинком и железом.

Ответ:

Mg + 2HCl ⟶ MgCl2 + H2

2Al + 6HCl ⟶ 2AlCl3 + 3H2

Zn + 2HCl ⟶ ZnCl2 + H2

Fe + 2HCl ⟶ FeCl2 + H2

6. Некий металл вступает в реакцию с раствором нитрата серебра, но не реагирует с разбавленной серной кислотой. Какой это металл?

Ответ:

Так как этот металл вступает в реакцию с раствором нитрата серебра, но не реагирует с разбавленной серной кислотой, то данный метал располагается в ряду напряжений металлов между водородом и серебром. Это может быть медь:

Cu + 2AgNO3 ⟶ Cu(NO3)2 + 2Ag

Cu + H2SO4 ⇸

7. В каком порядке будут выделяться металлы из раствора, содержащего нитраты меди, серебра и свинца, если в него опустить железную пластинку?

Ответ:

Сначала будут выделяться наименее активные металлы, т. е. они будут выделяться в следующем порядке: серебро, медь, свинец.

8. Как можно доказать, что ионы Ag+ являются более сильными окислителями, чем ионы Na+?

Ответ:

Например, ионы Ag+ могут окислить железо, а ионы Na+ – нет. Возьмём раствор нитрата серебра и раствор нитрата натрия, и опустим в них по железному гвоздю. В растворе нитрата серебра будет наблюдаться восстановление серебра, а в растворе нитрата натрия изменений наблюдаться не будет.

Fe + 2AgNO3 ⟶ Fe(NO3)2 + 2Ag
Fe0 + 2Ag+ ⟶ Fe2+ + 2Ag

Fe + NaNO3 ⇸
Fe0 + Na+ ⇸

§18

§18. Электролиз

1. Через раствор хлорида меди (II) пропустили электрический ток. Какой процесс произошёл на катоде, а какой – на аноде? Напишите суммарное уравнение электролиза. Назовите окислитель и восстановитель.

Ответ:

Катод (-): Cu2+ + 2ē ⟶ Cu0 |1    окислитель (восстановление)
Анод (+): 2Cl- - 2ē ⟶ Cl20 |1    восстановитель (окисление)

Cu2+ + 2Cl- ⟶ Cu + Cl2

CuCl2   эл. ток ⟶ Cu + Cl2

2. Натрий впервые был получен электролизом расплава гидроксида натрия. Составьте молекулярное уравнение электролиза, если известно, что гидроксид-ион окисляется на аноде по уравнению: 4OH- – 4ē ⟶ O2↑ + 2H2O. Определите элемент-окислитель и элемент-восстановитель.

Ответ:

Катод (-): Na+ + 1ē ⟶ Na0 |4    восстановление
Анод (+): 4OH- – 4ē ⟶ O2↑ + 2H2O |1    окисление

4NaOH   эл. ток ⟶ 4Na + O2↑ + 2H2O

3. Составьте уравнения электролиза: а) расплава хлорида калия; б) раствора хлорида золота AuCl3. Укажите общие черты и найдите различия в этих процессах.

Ответ:

а) электролиз расплава хлорида калия

Катод (-): K+ + 1ē ⟶ K0 |2    восстановление
Анод (+): 2Cl- - 2ē ⟶ Cl20 |1    окисление

2K+ + 2Cl- ⟶ 2K + Cl2
2KCl   эл. ток ⟶ 2K + Cl2

б) раствора хлорида золота

Катод (-): Au3+ + 3ē ⟶ Au0 |2    восстановление
Анод (+): 2Cl- - 2ē ⟶ Cl20 |3    окисление

2Au3+ + 6Cl- ⟶ 2Au + 3Cl2
2AuCl3   эл. ток ⟶ 2Au + 3Cl2

Различие состоит в том, что электролиз хлорида калия протекает в виде расплава, в электролиз хлорида золота в виде раствора, и в том, что для восстановления ионов металлов требуется разное количество электронов.

Сходство состоит в том, что в обоих случаях в результате гидролиза на катоде восстанавливается металл, а на аноде протекает окисление хлора до молекулярного состояния Cl2.

4. Используя метод электронного баланса, расставьте коэффициенты в схемах электролиза расплавов:
a) Al2O3 ⟶ Al + O2↑;
б) KBr ⟶ K + Br2;
в) KOH ⟶ K + H2O↑ + O2↑;
г) NaI ⟶ Na + I2.

Ответ:

a) 2Al2O3 ⟶ 4Al + 3O2

2O-2 - 4ē ⟶ O20 |4| 12 |3    окисление
Al+3 + 3ē ⟶ Al0 |3| |4    восстановление

б) 2KBr ⟶ 2K + Br2

2Br- - 2ē ⟶ Br20 |2| 2 |1    окисление
K+ + 1ē ⟶ K0 |1| |2    восстановление

в) 4KOH ⟶ 4K + 2H2O↑ + O2

2O-2 - 4ē ⟶ O20 |4| 4 |1    окисление
K+ + 1ē ⟶ K0 |1| |4    восстановление

г) NaI ⟶ Na + I2

2I- - 2ē ⟶ I20 |2| 2 |1    окисление
Na+ + 1ē ⟶ Na0 |1| |2    восстановление

5. Используя источники информации, установите, какие металлы и какие неметаллы получают в промышленности с помощью электролиза.

Ответ:

Щелочные и щелочноземельные металлы получают с помощью электролиза расплавов гидроксидов и хлоридов соответствующих металлов.

Медь получают с помощью электролиза раствора сульфата меди (II).

Водород получают с помощью электролиза растворов гидроксидов активных металлов.

Хлор в основном получают с помощью электролиза раствора поваренной соли.

§19

§19. Тепловые эффекты химических реакций

1. Приведите по два примера экзотермических и эндотермических реакций.

Ответ:

Экзотермические реакции:
2H2(г) + O2(г) = 2Н2О(г) + 484 кДж
CH4(г) + 2O2(г) = CO2(г) + 2H2O(г) + 802 кДж

Эндотермические реакции:
CaCO3(тв) = CaO(тв) + CO2(г) - 170 кДж
N2(г) + O2(г) = 2NO(г) - 180 кДж

2. Даны термохимические уравнения нескольких реакций:
а) 4P(тв) + 5O2(г) = 2P2O5(тв) + 2984 кДж;
б) 2Al(тв) + Fe2O3(тв) = Al2O3(тв) + 2Fe(тв) + 850 кДж;
в) N2(г) + O2(г) = 2NO(г) - 180 кДж;
г) C6H12O6(тв) + 6O2(г) = 6CO2(г) + 6H2O(ж) + 2800 кДж.
Какие из них являются экзотермическими; эндотермическими?

Ответ:

Экзотермические реакции: а, б, г.

Эндотермические реакции: в.

3. При кристаллизации свинца из расплава происходит выделение теплоты. Какой это процесс – экзо- или эндотермический?

Ответ:

Кристаллизации свинца – экзотермический процесс, так как происходит выделение теплоты.

4. Пользуясь данными, приведёнными в параграфе, составьте термохимическое уравнение разложения жидкой воды на водород и кислород.

Ответ:

2О(ж) = 2H2(г) + O2(г) - 572 кДж

5. Дано термохимическое уравнение синтеза аммиака:
N2(г) + 3H2(г) = 2NH3(г) + 92 кДж.

Рассчитайте, сколько теплоты выделится при образовании: а) 10 моль; б) 10 г; в) 10 л (н. у.) аммиака.

Ответ:

Дано:

`"а) "n(NH_3) = 10" моль"`

`"б) "m(NH_3) = 10 г`

`"в) "V(NH_3) = 10 л`

`Q = ?`

Решение

`N_(2"(г)") + 3H_(2"(г)") = 2NH_(3"(г)") + 92" кДж"`

а)

`Q = (92*10)/2 = 460" кДж"`

б)

`n(NH_3) = (m(NH_3))/(M(NH_3)) = 10/17 = 0.59" моль"`

`Q = (Q_"х.р."*n(NH_3))/(n_"х.р."(NH_3)) = (92*0.59)/2 = 27.1" кДж"`

в)

`n(NH_3) = (V(NH_3))/V_m = 10/22.4 = 0.45" моль"`

`Q = (Q_"х.р."*n(NH_3))/(n_"х.р."(NH_3)) = (92*0.45)/2 = 20.7" кДж"`

Ответ: `"а) "Q = 460" кДж"`, `"б) "Q = 27.1" кДж"`, `"в) "Q = 20.7" кДж"`.

6. Теплота сгорания угля равна 393,5 кДж/моль. Напишите термохимическое уравнение реакции. Рассчитайте количество теплоты, которое выделится при сгорании 1 кг угля. Какой объём углекислого газа (н. у.) образовался, если при сгорании угля выделилось 157,4 кДж теплоты?

Ответ:

Дано:

`Q_"сгор"(C) = 393.5" кДж/моль"`

`"а) "m(C) = 1" кг" = 1000 г`

`"б) "Q = 157.4" кДж"`

`"а) "Q = ?`

`"б) "V(CO_2) = ?`

Решение

`C_("(т)") + O_(2"(г)") = CO_(2"(г)") + 393.5" кДж/моль"`

а)

`n(C) = (m(C))/(M(C)) = 1000/12 = 83.3" моль"`

`Q = n(C)*Q_"сгор"(C) = 83.3*393.5 = 32800" кДж" = 32.8" МДж"`

б)

`n(C) = (Q)/(Q_"сгор"(C)) = 157.4/393.5 = 0.4" моль"`

`n(CO_2) = n(C) = 0.4" моль"`

`V(CO_2) = n(CO_2)*V_m = 0.4*22.4 = 8.96 л`

Ответ: `"а) "Q = 32.8" МДж"`, `"б) "V(CO_2) = 8.96 л`.

7. На разложение 19,7 г карбоната бария на оксид бария и углекислый газ затратили 27,2 кДж теплоты. Напишите термохимическое уравнение этой реакции.

Ответ:

Дано:

`m(BaCO_3) = 19.7 г`

`Q = 27.2" кДж"`

Термохимическое уравнение - ?

Решение

`BaCO_(3"(т)") = BaO_("(т)") + CO_(2"(г)") + 272" кДж"`

`n(BaCO_3) = (m(BaCO_3))/(M(BaCO_3)) = 19.7/197 = 0.1" моль"`

`Q_"х.р." = (Q*n_"х.р."(BaCO_3))/(n(BaCO_3)) = (27.2*1)/0.1 = 272" кДж"`

8. Рассчитайте количество теплоты, которая выделится при сгорании 1 м3 (н. у.) природного газа, содержащего 90% метана CH4, 8% этана C2H6 и 2% пропана C3H8 (по объёму). Теплоты сгорания метана, этана и пропана равны соответственно 802, 1428 и 2045 кДж/моль.

Ответ:

Дано:

`V("ПГ") = 1 м^3 = 1000 л`

`φ(CH_4) = 90%`

`φ(C_2H_6) = 8%`

`φ(C_3H_8) = 2%`

`Q_"сгор"(CH_4) = 802" кДж/моль"`

`Q_"сгор"(C_2H_6) = 1428" кДж/моль"`

`Q_"сгор"(C_3H_8) = 2045" кДж/моль"`

`Q_"общ." = ?`

Решение

1)

`V(CH_4) = (φ(CH_4)*V("ПГ"))/100 = (90*1000)/100 = 900 л`

`n(CH_4) = (V(CH_4))/V_m = 900/22.4 = 40.18" моль"`

`Q_1 = n(CH_4)*Q_"сгор"(CH_4) = 40.18*802 = 32220" кДж"`

2)

`V(C_2H_6) = (φ(C_2H_6)*V("ПГ"))/100 = (8*1000)/100 = 80 л`

`n(C_2H_6) = (V(C_2H_6))/V_m = 80/22.4 = 3.57" моль"`

`Q_2 = n(C_2H_6)*Q_"сгор"(C_2H_6) = 3.57*1428 = 5100" кДж"`

3)

`V(C_3H_8) = (φ(C_3H_8)*V("ПГ"))/100 = (2*1000)/100 = 20 л`

`n(C_3H_8) = (V(C_3H_8))/V_m = 20/22.4 = 0.89" моль"`

`Q_3 = n(C_3H_8)*Q_"сгор"(C_3H_8) = 0.89*2045 = 1820" кДж"`

`Q_"общ." = Q_1 + Q_2 + Q_3 = 32220 + 5100 + 1820 = 39140" кДж" = 39.1" МДж"`

Ответ: `Q_"общ." = 39.1" МДж"`.

9. Термохимическое уравнение реакции разложения воды на простые вещества: 2H2O(г) = 2H2(г) + O2(г) - 484 кДж. Сколько теплоты надо затратить для разложения 90 г газообразной воды?

Ответ:

Дано:

`m(H_2O) = 90 г`

`Q = ?`

Решение

`2H_2O_("(г)") = 2H_(2"(г)") + O_(2"(г)") - 484" кДж"`

`n(H_2O) = (m(H_2O))/(M(H_2O)) = 90/18 = 5" моль"`

`Q = (Q_"х.р."*n(H_2O))/(n_"х.р."(H_2O)) = (-484*5)/2 = -1210" кДж"`

Ответ: для разложения 90 г газообразной воды необходимо затратить 1210 кДж теплоты.

10. Теплоты сгорания метана и ацетилена равны соответственно 802 и 1260 кДж/моль. Какой из двух газов даёт больше теплоты при сгорании: а) 1 кг; б) 1 м3 (н. у.) вещества?

Ответ:

Дано:

`Q_"сгор"(CH_4) = 802" кДж/моль"`

`Q_"сгор"(C_2H_2) = 1260" кДж/моль"`

а)

`m(CH_4) = m(C_2H_2) = 1" кг" = 1000 г`

б)

`V(CH_4) = V(C_2H_2) = 1 м^3 = 1000 л`

`"а) "Q_(CH_4)" "?" "Q_(C_2H_2)`

`"б) "Q_(CH_4)" "?" "Q_(C_2H_2)`

Решение

а)

`n(CH_4) = (m(CH_4))/(M(CH_4)) = 1000/16 = 62.5" моль"`

`n(C_2H_2) = (m(C_2H_2))/(M(C_2H_2)) = 1000/26 = 38.5" моль"`

`Q_(CH_4) = n(CH_4)*Q_"сгор"(CH_4) = 62.5*802 = 50100" кДж"`

`Q_(C_2H_2) = n(C_2H_2)*Q_"сгор"(C_2H_2) = 38.5*1260 = 48500" кДж"`

`Q_(CH_4) > Q_(C_2H_2)`

б)

`n(C_2H_2) = n(CH_4) = (V(CH_4))/V_m = 1000/22.4 = 44.6" моль"`

`Q_(CH_4) = n(CH_4)*Q_"сгор"(CH_4) = 44.6*802 = 35800" кДж"`

`Q_(C_2H_2) = n(C_2H_2)*Q_"сгор"(C_2H_2) = 44.6*1260 = 56200" кДж"`

`Q_(CH_4) < Q_(C_2H_2)`

Ответ: `"а) "Q_(CH_4) > Q_(C_2H_2)`, `"б) "Q_(CH_4) < Q_(C_2H_2)`.

11. Используя данные задачи 2, определите тепловой эффект реакции фотосинтеза: 6CO2(г) + 6H2O(ж) = C6H12O6(тв) + 6O2(г). Откуда берётся энергия, необходимая для этой реакции?

Ответ:

Дано:

`Q_("обр.")(CO_2) = 393.5" кДж/моль"`

`Q_("обр.")(H_2O) = 286" кДж/моль"`

`Q_("обр.")(C_6H_12O_6) = 1274" кДж/моль"`

`Q_("х.р.") = ?`

Решение

`6CO_(2"(г)") + 6H_2O_("(ж)") = C_6H_12O_(6"(тв)") + 6O_(2"(г)")`

`Q_("х.р.") = n_("х.р.")(C_6H_12O_6)*Q_("обр.")(C_6H_12O_6) - (n_("х.р.")(CO_2)*Q_("обр.")(CO_2) + n_("х.р.")(H_2O)*Q_("обр.")(H_2O)) =`

`= 1274 - (6*393.5 + 6*286) = -2803" кДж"`

Ответ: `Q_("х.р.") = -2803" кДж"`.

Энергия, необходимая для реакции фотосинтеза, берётся из солнечного света.

12. Теплота сгорания водорода равна 286 кДж/моль. Используя данные задач 6 и 8, определите, какое топливо – уголь, водород или метан – даёт больше теплоты в расчёте на единицу массы (например, 1 г).

Ответ:

Дано:

`Q_"сгор"(C) = 393.5" кДж/моль"`

`Q_"сгор"(H_2) = 286" кДж/моль"`

`Q_"сгор"(CH_4) = 802" кДж/моль"`

`m(C) = 1 г`

`m(H_2) = 1 г`

`m(CH_4) = 1 г`

`Q_("max") = ?`

Решение

`n(C) = (m(C))/(M(C)) = 1/12 = 0.0833" моль"`
`Q_(C) = n(C)*Q_"сгор."(C) = 0.0833*393.5 = 32.8" кДж"`

`n(H_2) = (m(H_2))/(M(H_2)) = 1/2 = 0.5" моль"`
`Q_(H_2) = n(H_2)*Q_"сгор."(H_2) = 0.5*286 = 143" кДж"`

`n(CH_4) = (m(CH_4))/(M(CH_4)) = 1/16 = 0.0625" моль"`
`Q_(CH_4) = n(CH_4)*Q_"сгор."(CH_4) = 0.0625*802 = 50.1" кДж"`

Ответ: при сгорании водород даёт больше теплоты в расчёте на единицу массы, чем уголь и метан.

§20

§20. Скорость химических реакций

1. Что называют скоростью реакции; катализатором; ингибитором?

Ответ:

Скорость химической реакции – масса или количество вещества, образовавшегося или израсходованного в единицу времени в единице объёма.

Катализатор – вещество, участвующее в реакции и изменяющее её скорость, но остающееся неизменным после того, как химическая реакция закончилась.

Ингибитор – вещество, замедляющее реакцию.

2. От каких факторов зависит скорость реакции? Продолжите заполнение таблицы 10. Постарайтесь привести примеры, отличные от описанных в тексте параграфа.

Ответ:

Факторы, влияющие на скорость реакции. Примеры Объяснение
1. Природа реагирующих веществ. Металлы реагируют с соляной кислотой с различными скоростями. Металлы обладают различной химической активностью, т. е. способностью отдавать электроны.
2. Температура. При нагревании цинк с соляной кислотой реагирует быстрее, чем при комнатной температуре. При увеличении температуры увеличивается скорость движений/колебаний молекул или атомов, за счёт этого увеличивается число соударений частиц реагирующих веществ.
3. Присутствие катализатора (ингибитора). Водород окисляется на платине кислородом воздуха при комнатной температуре, а без платины – не окисляется. Катализатор, связываясь с реагентом, понижает энергию активации химической реакции.
4. Площадь поверхности. При поджигании порошок цинка с порошком серы бурно реагируют между собой, а кусок серы на цинковой пластине практически не реагирует с цинком, сгорая в кислороде воздуха. Благодаря измельчению увеличивается площадь соприкосновения веществ, за счёт этого увеличивается число соударений частиц реагирующих веществ.

3. В вашем распоряжении находятся склянки с кусочками железа, порошком железа, порошком олова, 10%-й и 2%-й соляной кислотой. При взаимодействии веществ из каких склянок водород будет выделяться с наибольшей скоростью?

Ответ:

Водород будет выделяться с наибольшей скоростью при реакции порошка железа с 10%-ным раствором соляной кислоты.

Скорость химической реакции увеличивается: с возрастанием концентрации реагирующих веществ; с увеличением площади соприкосновения реагирующих веществ.

4. В вашем распоряжении имеются 3%-й и 10%-й растворы серной и уксусной кислот. В каком из указанных растворов быстрее растворится кусочек магния?

Ответ:

Кусочек магния растворится быстрее при реакции с 10%-ным раствором серной кислоты.

Скорость химической реакции увеличивается: с возрастанием концентрации реагирующих веществ; с увеличением силы кислоты.

5. От каких из перечисленных ниже факторов не зависит скорость растворения алюминия в растворе щёлочи: температуры, давления, степени измельчения алюминия, концентрации щёлочи, формы сосуда, в котором происходит реакция?

Ответ:

Скорость растворения алюминия в растворе щёлочи не зависит от давления и формы сосуда, в котором происходит реакция.

6. Проведены два опыта по получению кислорода нагреванием перманганата калия. В первом опыте получено 0,3 моль кислорода за 3 минуты, а во втором – 0,75 моль за 5 минут. В каком случае скорость реакции выше, если сосуды, в которых проводили опыты, имеют одинаковые объёмы?

Ответ:

Дано:

`V_1 = V_2 = V`

`n_1(O_2) = 0.3" моль"`

`t_1 = 3" мин"`

`n_2(O_2) = 0.75" моль"`

`t_2 = 5" мин"`

`υ_1" "?" "υ_2`

Решение

`υ_1 = (n_1(O_2))/(t_1*V) = 0.3/(3*V) = 0.1/V`

`υ_2 = (n_2(O_2))/(t_2*V) = 0.75/(5*V) = 0.15/V`

`υ_1" "<" "υ_2`

Ответ: `υ_1" "<" "υ_2`.

7. За месяц до начала занятий лаборант приготовил водный раствор пероксида водорода, в 1 л которого содержалось 0,3 моль H2O2. Первого сентября учитель химии при проведении опыта обнаружил, что количество H2O2 в колбе уменьшилось вдвое. Рассчитайте скорость разложения пероксида водорода, считая, что в месяце 30 дней. Какой объём кислорода (н. у.) выделился из 5 мл раствора за это время?

Ответ:

Дано:

1)

`V("р-ра") = 1 л`

`n_1(H_2O_2) = 0.3" моль"`

`n_2(H_2O_2) = 0.15" моль"`

`t = 30" сут"`

2)

`V("р-ра") = 5" мл" = 0.005 л`

`"1) "υ = ?`

`"2) "V(O_2) = ?`

Решение

`2H_2O_2 ⟶ 2H_2 + O_2`

1)

`n_"разл."(H_2O_2) = n_1(H_2O_2) - n_2(H_2O_2) = 0.3 - 0.15 = 0.15" моль"`

`υ = (n_"разл."(H_2O_2))/(t*V) = 0.15/(30*1) = 0.005" "("моль")/(л*"сут")"`

2)

`1 л "р-ра" – 0.15" моль" H_2O_2`
`0.005 л "р-ра" – n(H_2O_2)`

`n(H_2O_2) = (0.005*0.15)/1 = 0.00075" моль" = 0.75" ммоль"`

`n(O_2) = (n(H_2O_2))/2 = 0.75/2 = 0.375" ммоль"`

`V(O_2) = n(O_2)*V_m = 0.375*22.4 = 8.4" мл"`

Ответ: `υ = 0.005" "("моль")/(л*"сут")"`, `V(O_2) = 8.4" мл"`.

8. Охарактеризуйте общие свойства любого катализатора. Чем вызвано влияние катализатора на скорость реакции?

Ответ:

Катализатор участвует в реакции, увеличивает её скорость, не расходуется по окончании реакции. Катализатор, связываясь с реагентом, понижает энергию активации химической реакции, этим и вызвано влияние катализатора на скорость реакции.

9. Найдите информацию о нескольких конкретных процессах химической промышленности, которые требуют участия катализаторов.

Ответ:

Тримеризация ацетилена в присутствии активированного угля для получения бензола.

Гидрогенизация растительных жиров в присутствии никеля для получения маргарина.

Изомеризация бензиновых фракций в присутствии платины для получения высокооктанового бензина.

§21

§21. Обратимые реакции. Химическое равновесие

1. Что называют состоянием равновесия?

Ответ:

Состояние, при котором количества всех участвующих в обратимой реакции веществ не меняются во времени, называют состоянием равновесия.

2. Предскажите, в какую сторону сместится равновесие реакции H2(г) + S(г) ⇄ H2S(г) + Q:
а) при нагревании;
б) при уменьшении давления;
в) при увеличении концентрации водорода.

Ответ:

а) Влево.

б) Влево.

в) Вправо.

3. Приведите пример необратимой экзотермической окислительно-восстановительной реакции соединения.

Ответ:

CH4 + 2O2 ⟶ CO2 + 2H2O + Q

C + O2 ⟶ CO2 + Q

S + O2 ⟶ SO2 + Q

4. Воспользовавшись схемой 5, охарактеризуйте по всем признакам сравнения реакции: а) разложения карбоната кальция; б) взаимодействия натрия с водой; в) нейтрализации гидроксида натрия соляной кислотой.

Ответ:

а) разложение карбоната кальция

CaCO3(т) ⇄ CaO(т) + CO2(г) - Q

  • реакция разложения
  • эндотермическая
  • обратимая
  • не ОВР

б) взаимодействие натрия с водой

2Na(т) + 2H2O(ж) ⟶ 2NaOH(р) + H2(г) + Q

  • реакция замещения
  • экзотермическая
  • необратимая
  • ОВР

в) нейтрализация гидроксида натрия соляной кислотой

NaOH(р) + HCl(р) ⟶ NaCl(р) + H2O(ж) + Q

  • реакция обмена
  • экзотермическая
  • необратимая
  • не ОВР

5. Приведите по одному примеру обратимых реакций с участием: а) водорода; б) кислорода; в) воды; г) азота; д) метана; е) спирта; ж) иона водорода. Как можно сместить равновесие каждой из реакций в сторону образования продуктов?

Ответ:

а) H2(г) + Cl2(г) ⇄ 2HCl(г) + Q
Для смещения равновесия в сторону образования продуктов необходимо: уменьшить температуру, увеличить концентрацию водорода.

б) 2SO2(г) + O2(г) ⇄ 2SO3(г) + Q
Для смещения равновесия в сторону образования продуктов необходимо: уменьшить температуру, увеличить давление, увеличить концентрацию оксида серы (IV).

в) 3Fe(т) + 4H2O(г) ⇄ Fe3O4(т) + 4H2(г) + Q
Для смещения равновесия в сторону образования продуктов необходимо: уменьшить температуру, увеличить концентрацию водяного пара.

г) N2(г) + 3H2(г) ⇄ 2NH3(г) + Q
Для смещения равновесия в сторону образования продуктов необходимо: уменьшить температуру, увеличить давление, увеличить концентрацию водорода.

д) CH4(г) + H2O(г) ⇄ CO(г) + 3H2(г) - Q
Для смещения равновесия в сторону образования продуктов необходимо: увеличить температуру, уменьшить давление, увеличить концентрацию водяного пара.

е) CO(г) + 2H2(г) ⇄ CH3OH(г) - Q
Для смещения равновесия в сторону образования продуктов необходимо: увеличить температуру, увеличить давление, увеличить концентрацию водорода.

ж) HNO2(р) ⇄ H(р)+ + NO2(р)-

6. Приведите по одному примеру обратимых реакций: а) разложения; б) соединения; в) замещения. Какие условия способствуют протеканию прямой реакции в каждом случае?

Ответ:

а) CaCO3(т) ⇄ CaO(т) + CO2(г) - Q
Протеканию прямой реакции способствуют увеличение температуры и уменьшение давления.

б) N2(г) + 3H2(г) ⇄ 2NH3(г) + Q
Протеканию прямой реакции способствуют уменьшение температуры и увеличение давления.

в) 3Fe(т) + 4H2O(г) ⇄ Fe3O4(т) + 4H2(г) + Q
Протеканию прямой реакции способствует уменьшение температуры.

7. Приведите пример обратимой реакции между газами, в которой давление не влияет на положение равновесия.

Ответ:

H2(г) + Cl2(г) ⇄ 2HCl(г)

N2(г) + O2(г) ⇄ 2NO(г)

8. Какие факторы способствуют увеличению выхода продукта в следующих промышленно важных реакциях:
а) CH4(г) + H2O(г) ⇄ CO(г) + 3H2(г) - Q;
б) N2(г) + 3H2(г) ⇄ 2NH3(г) + Q;
в) 2SO2(г) + O2(г) ⇄ 2SO3(г) + Q?

Ответ:

а) CH4(г) + H2O(г) ⇄ CO(г) + 3H2(г) - Q
Чтобы увеличить выход продукта необходимо: увеличить температуру, уменьшить давление, увеличить концентрацию водяного пара.

б) N2(г) + 3H2(г) ⇄ 2NH3(г) + Q
Чтобы увеличить выход продукта необходимо: уменьшить температуру, увеличить давление, увеличить концентрацию водорода.

в) 2SO2(г) + O2(г) ⇄ 2SO3(г) + Q
Чтобы увеличить выход продукта необходимо: уменьшить температуру, увеличить давление, увеличить концентрацию оксида серы (IV).

Творческие задания 1

Творческие задания 1

1. Подумайте, можно ли по вкусу раствора определить, что он проводит электрический ток. Приведите аргументы "за" и "против". Помните, что в лаборатории пробовать вещества на вкус строго запрещено.

Ответ:

Если вкус раствора кислый, горький или сладкий, то нельзя наверняка определить, проводит ли раствор электрический ток или нет.

Например, кислый раствор уксусной кислоты не проводит электрический ток, а кислый раствор соляной кислоты – проводит.

Например, горький раствор алкалоидов не проводит электрический ток, а горький раствор сульфата магния – проводит.

Например, сладкий раствор сахара не проводит электрический ток, а сладкий раствор нитрата свинца (II) – проводит.

Если раствор имеет солёный вкус, то, как правило, этот раствор проводит электрический ток. Например, раствор хлорида натрия и калия имеют соленый вкус, и они оба проводят электрический ток.

2. Обсудите в классе, может ли сильный электролит быть нерастворимым в воде. Приведите аргументы.

Ответ:

Водные растворы или расплавы сильных электролитов проводят электрический ток. Сильный электролит может быть нерастворимым в воде, однако в виде расплава он будет проводить электрический ток, например, к такому веществу можно отнести сульфат бария.

3. Вспомните классификацию химических реакций. Реакции какого типа обязательно являются окислительно-восстановительными?

Ответ:

К окислительно-восстановительным реакциям относятся все реакции замещения.

4. Приведите примеры веществ, которые одновременно проявляют: а) кислотные и окислительные свойства; б) кислотные и восстановительные свойства; в) основные и восстановительные свойства.

Ответ:

а) HNO3, H2SO4.

б) HCl, H2S.

в) NH3, PH3.

5. Какое простое вещество можно назвать: а) самым сильным окислителем; б) самым сильным восстановителем?

Ответ:

а) Фтор F2.

б) Франций Fr.

6. ==

Ответ:

 ==

7. Сколько кубометров метана (при н. у.) нужно сжечь, чтобы в зимний период обеспечить в течение суток теплом однокомнатную квартиру (требуется 140 МДж тепловые потери не учитываем)?

Ответ:

Дано:

`Q = 140" МДж" = 1.4*10^5" кДж"`

`V(CH_4) = ?`

Решение

`CH_(4"(г)") + 2O_(2"(г)") ⟶ CO_(2"(г)") + 2H_2O_("(г)") + 802" кДж/моль"`

`n(CH_4) = Q/(Q_"х.р.") = (1.4*10^5)/802 = 175" моль"`

`V(CH_4) = n(CH_4)*V(CH_4) = 175*22.4 = 3920 л = 3.92 м^3`

Ответ: `V(CH_4) = 3.92 м^3`.

8. Проверьте с помощью индикаторов или pH-метра, какие из перечисленных ниже жидкостей имеют кислотную, какие – почти нейтральную, а какие – щелочную среду: газированная вода, перекись водорода, томатный сок, кофе, чай, молоко, жидкое мыло, этиловый спирт, уксус, нашатырный спирт, средство для мытья посуды. Попытайтесь объяснить полученные результаты.

Ответ:

Газированная вода имеет кислую среду, так как углекислый газ, реагируя с водой, образует угольную кислоту, которая диссоциирует с образованием катионов водорода:
CO2 + H2O ⇄ H2CO3
H2CO3 ⇄ H+ + HCO3-

Перекись водорода имеет кислую среду, так как она диссоциирует с образованием катионов водорода:
H2O2 ⇄ H+ + HO2-

Томатный сок, кофе и чай имеют кислую среду, так как содержат в своём составе органические кислоты, которые диссоциируют с образованием катионов водорода.

Молоко имеет слабо кислую среду, так как оно содержит продукт жизнедеятельности молочнокислых бактерий – молочную кислоту, которая диссоциирует с образованием катионов водорода.

Жидкое мыло и средство для мытья посуды имеют щелочную среду, так как они состоит из калийных солей жирных кислот, в результате гидролиза которых образуются гидроксид-анионы:
C17H35COOK ⇄ C17H35COO- + K+

Этиловый спирт имеет нейтральную среду.

Уксус имеет кислую среду, так как уксус это водный раствор уксусной кислоты, которая диссоциирует с образованием катионов водорода:
CH3COOH ⇄ CH3COO- + H+

Нашатырный спирт имеет щелочную среду, так как аммиак, реагируя с водой, образует гидроксид аммония, который диссоциирует с образованием гидроксид-анионов:
NH3 + H2O ⇄ NH4OH
NH4OH ⇄ NH4+ + OH-

§22

Глава 3. Неметаллы

§22. Общая характеристика неметаллов

1. Перечислите общие физические и химические свойства неметаллов.

Ответ:

Общие физические неметаллов. Неметаллы, как правило, не обладают металлическим блеском, не проводят электрический ток, являются плохими проводниками тепла.

Общие химические неметаллов. Все неметаллы чаще выступают в роли окислителей. Для неметаллов особенно характерны реакции с металлами.

2. Как объяснить, что среди неметаллов есть и газы, и тугоплавкие твёрдые тела?

Ответ:

Химическая связь в простых веществах неметаллов – ковалентная неполярная, а значит они имеют либо молекулярную, либо атомную кристаллическую решётку, поэтому среди неметаллов есть и газы, и тугоплавкие твёрдые тела.

3. Напишите уравнения реакций взаимодействия кальция с серой, фосфором, водородом и углеродом, зная, что в соединениях с металлами эти элементы проявляют низшие степени окисления.

Ответ:

Ca + S ⟶ CaS

3Ca + 2P ⟶ Ca3P2

Ca + H2 ⟶ CaH2

Ca + C ⟶ Ca2C

4. Вспомните, как изменяются неметаллические свойства элементов при движении по периоду, по группе (главной подгруппе).

Ответ:

Неметаллические свойства элементов при движении по периоду слева направо усиливаются.

Неметаллические свойства элементов при движении по группе (главной подгруппе) сверху вниз ослабевают.

5. В 100 г какого из веществ – сероводорода H2S или аммиака NH3 – содержится большее число молекул и во сколько раз?

Ответ:

Дано:

`m(H_2S) = 100 г`

`m(NH_3) = 100 г`

`N(H_2S)" : "N(NH_3) = ?`

Решение

`n(H_2S) = (m(H_2S))/(M(H_2S)) = 100/34 = 2.94" моль"`

`n(NH_3) = (m(NH_3))/(M(NH_3)) = 100/17 = 5.88" моль"`

`N(H_2S)" : "N(NH_3) = n(H_2S)*N_A" : "n(NH_3)*N_A`

`N(H_2S)" : "N(NH_3) = 2.94" : "5.88`

`N(H_2S)" : "N(NH_3) = 1" : "2`

Ответ: в 100 г аммиака содержится в 2 раза больше молекул.

6. Выведите формулу соединения меди с серой, в котором массовая доля меди составляет 0,8.

Ответ:

Дано:

`ω(Cu) = 0.8`

`Cu_xS_y - ?`

Решение

`ω(S) = 1 - ω(Cu) = 1 - 0.8 = 0.2`

`x" : "y = (ω(Cu))/(M(Cu))" : "(ω(S))/(M(S))`

`x" : "y = 0.8/64" : "0.2/32`

`x" : "y = 0.0125" : "0.00625`

`x" : "y = 2" : "1`

Формула вещества `Cu_2S`.

Ответ: `Cu_2S`.

7. В атоме элемента-неметалла на внешнем уровне находится столько же электронов, сколько не хватает до завершения этого уровня. Назовите два таких элемента.

Ответ:

Углерод и кремний содержат на внешнем уровне столько же электронов, сколько не хватает до его завершения (4ē).

8. Используя информационные источники, сравните между собой строение твёрдых веществ – неметаллов: серы, белого фосфора, красного фосфора, алмаза, кремния, бора. Что общего в их строении и в чём различия?

Ответ:

  сера белый фосфор красный фосфор алмаз кремний бор
Химическая связь с помощью которой образованы простые вещества ковалентная неполярная
Кристаллическая решётка молекулярная молекулярная атомная атомная атомная атомная
В узлах кристаллической решетки циклические молекулы S8 молекулы P4 в форме тетраэдра отдельные атомы
Элементарная ячейка решётки орторомбическая кубическая или гексагональная сложная (из пирамидально связанных атомов) тетраэдрическая (в центре и четырех вершинах которого расположены атомы) ромбоэдрическая (в вершинах которого расположены атомы)

§23

§23. Хлор

1. Найдите элемент хлор в Периодической системе. Изобразите электронное строение атома хлора.

Ответ:

17Cl +17 )2)8)7 

17Cl   ↑↓   ↑↓   ↑↓ ↑↓ ↑↓   ↑↓   ↑↓ ↑↓
  1s   2s   2p   3s   3p

1s2 2s2 2p6 3s2 3p5

2. Сколько протонов и электронов содержат атом хлора, молекула хлора, хлорид-ион?

Ответ:

  Число протонов Число электронов
Cl 17 17
Cl2 34 34
Cl- 17 18

3. Опишите физические и химические свойства хлора.

Ответ:

Физические свойства хлора. Хлор представляет собой зеленовато-жёлтый газ с едким запахом, он примерно в два с половиной раза тяжелее воздуха. Хлор легко сгущается в жидкость при охлаждении или повышении давления. В 100 объёмах воды при 0°C растворяется 150 объёмов хлора.

Химические свойства хлора. Хлор вступает в реакцию со многими простыми веществами – металлами и неметаллами, образуя хлориды, в этих реакциях хлор проявляет окислительные свойства. В реакциях с кислородом и фтором хлор проявляет восстановительные свойства. Взаимодействует с водой образуя соляную и слабую неустойчивую хлорноватистую кислоту HClO. Хлор может окислить множество сложных веществ, например, хлор разрушает красящие вещества, отнимая от них водород.

4. Хлор реагирует с железом с образованием хлорида железа (III), а с медью – с образованием хлорида меди (II). Напишите уравнения реакций, укажите окислитель и восстановитель.

Ответ:

2Fe + 3Cl2 ⟶ 2FeCl3

Fe0 - 3ē ⟶ Fe+3 |3| 6 |2    восстановитель (окисление)
Cl20 + 2ē ⟶ 2Cl- |2| |3    окислитель (восстановление)

5. В пробирку с оксидом марганца (IV) прилили соляную кислоту. К отверстию пробирки поднесли бумажку, смоченную раствором лакмуса. Она обесцветилась. Объясните это явление.

Ответ:

В результате взаимодействия соляной кислоты с оксидом марганца (IV) образуется газообразный хлор:
MnO2 + 4HCl ⟶ MnCl2 + Cl2↑ + 2H2O

Хлор, являясь сильным окислителем, окисляет лакмус, к результате чего последний обесцвечивается.

6. Определите массовую долю хлора в хлориде алюминия.

Ответ:

Дано:

`AlCl_3`

`ω(Cl_2) = ?`

Решение

`ω(Cl_2) = (100*n(Cl)*M(Cl))/(M(AlCl_3)) = (100*3*35.5)/133.5 = 79.8%`

Ответ: `ω(Cl_2) = 79.8%`.

7. Какая масса хлора соединяется с 6 г водорода и какая масса хлороводорода при этом получается?

Ответ:

Дано:

`m(H_2) = 6 г`

`m(Cl_2) = ?`

`m(HCl) = ?`

Решение

`H_2 + Cl_2 ⟶ 2HCl`

`n(H_2) = (m(H_2))/(M(H_2)) = 6/2 = 3" моль"`

`n(Cl_2) = n(H_2) = 3" моль"`

`m(Cl_2) = n(Cl_2)*M(Cl_2) = 3*71 = 213 г`

`n(HCl) = 2*n(H_2) = 2*3 = 6" моль"`

`m(HCl) = n(HCl)*M(HCl) = 6*36.5 = 219 г`

Ответ: `m(Cl_2) = 213 г`, `m(HCl) = 219 г`.

8. Найдите массу оксида марганца (IV), которую требуется взять для получения 14,2 г хлора из соляной кислоты.

Ответ:

Дано:

`m(Cl_2) = 14.2 г`

`m(MnO_2) = ?`

Решение

`MnO_2 + 4HCl ⟶ MnCl_2 + Cl_2"↑" + 2H_2O`

`n(Cl_2) = (m(Cl_2))/(M(Cl_2)) = 14.2/71 = 0.2" моль"`

`n(MnO_2) = n(Cl_2) = 0.2" моль"`

`m(MnO_2) = n(MnO_2)*M(MnO_2) = 0.2*87 = 17.4 г`

Ответ: `m(MnO_2) = 17.4 г`.

9. Во сколько раз хлор тяжелее воздуха? Рассчитайте плотность хлора при нормальных условиях.

Ответ:

Дано:

`Cl_2`

`D_("возд.")(Cl_2) = ?`

`ρ(Cl_2) = ?`

Решение

`D_("возд.")(Cl_2) = (M(Cl_2))/(M("возд.")) = 71/29 = 2.45`

`ρ(Cl_2) = (M(Cl_2))/(V_m) = 71/22.4 = 3.17" г/л"`

Ответ: хлор тяжелее воздуха в `2.45` раза, `ρ(Cl_2) = 3.17" г/л"`.

10. При комнатной температуре (25°C) в одном объёме воды растворяется два объёма хлора. Рассчитайте массовую долю и молярную концентрацию хлора в насыщенной хлорной воде (плотность хлорной воды равна 1,0 г/мл).

Ответ:

Дано:

`V(H_2O)`

`V(Cl_2) = 2*V(H_2O)`

`ρ("р-ра") = 1" г/мл"`

`ω(Cl_2) = ?`

`C_M(Cl_2) = ?`

Решение

Допустим имеется 1000 мл (1 л) воды, тогда:

`V(Cl_2) = 2*1000 = 2000" мл" = 2 л`

`n(Cl_2) = (V(Cl_2))/V_m = 2/22.4 = 0.0893" моль"`

`m(Cl_2) = n(Cl_2)*M(Cl_2) = 0.0893*71 = 6.34 г`

`m(H_2O) = ρ(H_2O)*V(H_2O) = 1*1000 = 1000 г`

`m("р-ра") = m(H_2O) + m(Cl_2) = 1000 + 6.34 = 1006.34 г`

`ω(Cl_2) = (100*m(Cl_2))/(m("р-ра")) = (100*6.34)/1006.34 = 0.63%`

`V("р-ра") = (m("р-ра"))/(ρ("р-ра")) = 1006.34/1 = 1006.34" мл" = 1.00634 л`

`C_M(Cl_2) = (n(Cl_2))/(V("р-ра")) = 0.0893/1.00634 = 0.0887" моль/л"`

Ответ: `ω(Cl_2) = 0.63%`, `C_M(Cl_2) = 0.0887" моль/л"`.

11. В реакциях с каким веществом хлор проявляет свойства восстановителя? Напишите уравнение реакции, если известно, что в продукте хлор имеет степень окисления +5.

Ответ:

Хлор при взаимодействии со фтором проявляет свойства восстановителя.

Cl2 + 5F2 ⟶ 2ClF5

Cl20 - 10ē ⟶ 2Cl+5 |10| 10 |1    восстановитель (окисление)
F20 + 2ē ⟶ 2F- |2  | |5    окислитель (восстановление)

§24

§24. Хлороводород и соляная кислота

1. Как получают соляную кислоту в промышленности?

Ответ:

В промышленности соляную кислоту получают с помощью растворения хлороводорода в воде, получаемого с помощью реакции горения водорода в хлоре:
H2 + Cl2 ⟶ 2HCl

2. Пользуясь рисунком 41, перечислите основные области применения соляной кислоты.

Ответ:

Соляная кислота применяется для получения её солей (например, для получения хлорида цинка).

Соляная кислота применяется для очистки стали от ржавчины и окалины.

Соляная кислота применяется в медицине при недостаточной кислотности желудочного сока.

Соляная кислота применяется для обнаружения карбонатных пород и минералов (например, мрамора).

3. Чем отличается действие на ржавый гвоздь чистой соляной кислоты и соляной кислоты, содержащей ингибитор?

Ответ:

Чистая соляная кислота активно реагирует с железом гвоздя, а соляная кислота, содержащая ингибитор, медленно реагирует с железом гвоздя.

4. Как доказать, что в состав соляной кислоты входят водород и хлор?

Ответ:

Доказать, что в состав соляной кислоты входит водород, можно с помощью магния:
Mg + 2HCl ⟶ MgCl2 + H2

Доказать, что в состав соляной кислоты входит хлор, можно с помощью оксида марганца (IV):
MnO2 + 4HCl ⟶ MnCl2 + Cl2↑ + 2H2O

5. На чём основано использование соляной кислоты при травлении?

Ответ:

Использование соляной кислоты при травлении металлических поверхностей основано на том, что соляная кислота взаимодействует с оксидами металлов, превращая их в растворимые соединения, тем самым очищая поверхность металла.

6. Сколько килограммов 36,5%-й соляной кислоты можно получить из 44,8 м3 хлора (н. у.)?

Ответ:

Дано:

`ω(HCl) = 36.5%`

`V(Cl_2) = 44.8 м^3`

`m("р-ра") = ?`

Решение

`H_2 + Cl_2 ⟶ 2HCl`

`n(Cl_2) = (V(Cl_2))/V_m = 44.8/22.4 = 2" кмоль"`

`n(HCl) = 2*n(Cl_2) = 2*2 = 4" кмоль"`

`m(HCl) = n(HCl)*M(HCl) = 4*36.5 = 146" кг"`

`m("р-ра") = (100*m(HCl))/(ω(HCl)) = (100*146)/36.5 = 400" кг"`

Ответ: `m("р-ра") = 400" кг"`.

7. В четырёх пробирках без этикеток находятся соляная кислота, хлорид натрия, карбонат натрия, гидроксид натрия. Как опытным путём установить содержание каждой пробирки? Напишите уравнения реакций.

Ответ:

Разделить содержимое каждой пробирок на 4 части.

  HCl Na2CO3 NaOH NaCl
1) Na2CO3 Выделяется бесцветный газ:
Na2CO3 + 2HCl ⟶ 2NaCl + H2O + CO2
Не реагируют. Не реагируют. Не реагируют.
2) Ca(OH)2   Выпадет белый осадок:
Ca(OH)2 + Na2CO3 ⟶ 2NaOH + CaCO3
Не реагируют. Не реагируют.
3) CuSO4   Выпадет голубой осадок:
CuSO4 + 2NaOH ⟶ Na2SO4 + Cu(OH)2
Не реагируют.
4) AgNO3   Выпадет белый осадок:
AgNO3 + NaCl ⟶ NaNO3 + AgCl↓

8. Даны: цинк, медь, оксид цинка, оксид углерода (IV), карбонат натрия, нитрат натрия, нитрат свинца (II), гидроксид магния. С какими из этих веществ реагирует соляная кислота? Напишите уравнения реакций.

Ответ:

Соляная кислота реагирует c: цинком, оксидом цинка, карбонатом натрия, нитратом свинца (II), гидроксидом магния.

Zn + 2HCl ⟶ ZnCl2 + H2

ZnO + 2HCl ⟶ ZnCl2 + H2O

Na2CO3 + 2HCl ⟶ 2NaCl + H2O + CO2

Pb(NO3)2 + 2HCl ⟶ 2HNO3 + PbCl2

Mg(OH)2 + 2HCl ⟶ MgCl2 + 2H2O

9. Напишите уравнения реакций, соответствующих схеме:
Cl2 ⟶ HCl ⟶ CuCl2 ⟶ BaCl2 ⟶ KCl ⟶ HCl ⟶ Cl2.

Ответ:

H2 + Cl2 ⟶ 2HCl

CuO + 2HCl ⟶ CuCl2 + H2O

CuCl2 + Ba(OH)2 ⟶ BaCl2 + Cu(OH)2

BaCl2 + K2SO4 ⟶ 2KCl + BaSO4

KCl(т) + H2SO4(конц.)   t ⟶ KHSO4 + HCl↑

MnO2 + 4HCl ⟶ MnCl2 + Cl2↑ + 2H2O

10. Может ли хлороводород проявлять свойства окислителя? Приведите пример.

Ответ:

Атом хлора в хлороводороде не может проявлять свойства окислителя, так как он находится в своей низшей степени окисления, а атом водорода в хлороводороде может проявлять свойства окислителя, так как находится в своей высшей степени окисления.

Mg + 2HCl ⟶ MgCl2 + H2

Mg0 - 2ē ⟶ Mg+2 |2| 2 |1    восстановитель (окисление)
2H+ + 2ē ⟶ H20 |2| |1    окислитель (восстановление)

Лабораторный опыт 6. Качественные реакции на соляную кислоту.

1. Напишите уравнение диссоциации соляной кислоты. Ознакомьтесь с внешним видом раствора, проверьте, имеет ли он запах. Отметьте в тетради его окраску. Вспомните, сильный или слабый это электролит.

2. Докажите, что в соляной кислоте присутствуют ионы водорода. Для этого налейте в пробирку примерно 1 мл соляной кислоты и добавьте раствор лакмуса. Как изменилась окраска индикатора?

3. Докажите, что в соляной кислоте присутствуют хлорид-ионы. Для этого налейте в пробирку примерно 1 мл соляной кислоты и добавьте одну каплю раствора нитрата серебра. Что наблюдается? Отметьте цвет и характер (кристаллический, творожистый) выпавшего осадка.

Ответ:

1. Уравнение диссоциации соляной кислоты:
HCl ⟶ H+ + Cl-

Раствор соляной кислоты не имеет цвета, обладает резким запахом.

Соляная кислота – сильный электролит.

2. При добавлении лакмуса к соляной кислоте, окраска индикатора изменяется на красную, это говорит о том, что в соляной кислоте присутствуют ионы водорода.

3. При добавлении раствора нитрата серебра к соляной кислоте наблюдается выпадение белого творожистого осадка:
HCl + AgNO3 ⟶ HNO3 + AgCl↓

§25

§25. Галогены

1. Какие вещества называют галогенами? Как изменяется химическая активность галогенов при движении вниз по подгруппе?

Ответ:

Галогенами называют простые вещества образованные элементами главной подгруппы VII группы Периодической системы.

2. Чему равны низшая и высшая степени окисления: а) фтора; б) хлора?

Ответ:

а) Низшая степень окисления фтора -1, а высшая 0.

б) Низшая степень окисления хлора -1, а высшая +7.

3. Какой из галогенов наиболее энергично реагирует с водородом? Напишите уравнение реакции.

Ответ:

Среди галогенов с водородом наиболее энергично реагирует фтор:
H2 + F2 ⟶ 2HF

4. Какой из галогенов реагирует с твёрдым хлоридом натрия? Напишите уравнение реакции.

Ответ:

С твёрдым хлоридом натрия реагирует фтор:
2NaCl + F2 ⟶ 2NaF + Cl2

5. Приведите пример реакции между простым и сложным веществом, если в состав их молекул входят только галогены и водород.

Ответ:

2HBr + Cl2 = 2HCl + Br2

2HI + Br2 = 2HBr + I2

6. Даны следующие вещества: оксид марганца (IV), хлорид калия, бромид калия, магний, серная кислота. Требуется получить соляную кислоту, хлор, бром, хлорид магния, бромид магния. Напишите уравнения реакций.

Ответ:

Если полученные вещества тоже можно использовать:

KCl(т) + H2SO4(конц.)   t ⟶ KHSO4 + HCl↑

MnO2 + 4HCl ⟶ MnCl2 + Cl2↑ + 2H2O

2KBr + Cl2 ⟶ 2KCl + Br2

Mg + 2HCl ⟶ MgCl2 + H2

Mg + Br2 ⟶ MgBr2

7. Фторид серебра хорошо растворим в воде. Напишите уравнения реакций его взаимодействия с раствором: а) хлорида магния; б) бромида алюминия; в) иодида калия.

Ответ:

а) 2AgF + MgCl2 ⟶ 2AgCl↓ + MgF2

б) 3AgF + AlBr3 ⟶ AlCl3 + 3AgBr↓

в) AgF + KI ⟶ KF + AgI↓

8. Как обнаружить в растворе свободный иод?

Ответ:

Свободный иод в растворе можно обнаружить с помощью крахмала, при его добавлении будет наблюдаться окрашивание раствора в фиолетовый цвет.

9. Одну чайную ложку мёда, купленного на рынке, растворили в воде, в которую добавили одну каплю йодной настойки. Раствор окрасился в фиолетовый цвет. О чём это свидетельствует?

Ответ:

Окрашивание раствора в фиолетовый цвет свидетельствует о том, что в мёд был добавлен крахмал.

10. Сколько граммов 10%-го раствора плавиковой кислоты можно получить, растворив в воде фтороводород, полученный взаимодействием 117 г фторида кальция с серной кислотой?

Ответ:

Дано:

`ω(HF) = 10%`

`m(CaF_2) = 117 г`

`m("р-ра") = ?`

Решение

`CaF_2 + 2H_2SO_(4"(конц.)") ⟶ Ca(HSO_4)_2 + 2HF"↑"`

`n(CaF_2) = (m(CaF_2))/(M(CaF_2)) = 117/78 = 1.5" моль"`

`n(HF) = 2*n(CaF_2) = 2*1.5 = 3" моль"`

`m(HF) = n(HF)*M(HF) = 3*20 = 60 г`

`m("р-ра") = (100*m(HF))/(ω(HF)) = (100*60)/10 = 600 г`

Ответ: `m("р-ра") = 600 г`.

11. С какими веществами из ряда: NaOH, Cu, CuO, SiO2, AgNO3 – может реагировать плавиковая кислота?

Ответ:

Плавиковая кислота может реагировать с: NaOH, CuO, SiO2.

NaOH + HF ⟶ NaF + H2O

CuO + 2HF ⟶ CuF2 + H2O

SiO2 + HF(конц.) ⟶ H2[SiF6] + 2H2O

12. Напишите уравнения реакций, соответствующих схеме: KOH ⟶ KF ⟶ CaF2 ⟶ HF ⟶ F2.

Ответ:

KOH + HF ⟶ KF + H2O

2KF + Ca(NO3)2 ⟶ 2KNO3 + CaF2

CaF2 + 2HCl ⟶ CaCl2 + 2HF

2HF   электролиз ⟶ H2 + F2

§26

§26. Сера и её соединения

1. Охарактеризуйте физические и химические свойства серы.

Ответ:

Физические свойства серы

Самородная (ромбическая) сера – хрупкое, светло-жёлтое вещество, нерастворимое в воде. Часто кристаллы самородной серы имеют вид правильных октаэдров. При 119°C сера плавится, а при 445°C – кипит. При кристаллизации серы образуется ромбическая сера, её кристаллы имеют форму тонких призм. Обе эти формы состоят из молекул S8, но отличаются цветом, плотностью, формой кристаллов. При хранении моноклинная сера рассыпается в порошок, состоящий из очень мелких кристаллов серы ромбической.

Химические свойства серы

Сера реагирует с металлами:
2Na + S = Na2S
Zn + S = ZnS

Сера реагирует с неметаллами:
H2 + S = H2S
S + O2 = SO2

2. В реакциях с какими веществами сера проявляет свойства: а) окислителя; б) восстановителя?

Ответ:

а) Сера проявляет свойства окислителя со всеми простыми веществами (в основном это металлы), корме тех, которые образованны N, O и галогенами.

б) Сера проявляет свойства восстановителя простыми веществами, которые образованны N, O и галогенами.

3. Что называют аллотропией? Приведите примеры.

Ответ:

Аллотропия – способность атомов одного и того же элемента образовывать несколько простых веществ.

  • сера – ромбическая, моноклинная и пластическая сера;
  • кислород – кислород, озон;
  • углерод – алмаз, графит, фуллерен, карбин, графен, углеродные нанотрубки;
  • фосфор – белый, красный и чёрный фосфор.

4. Приведите три уравнения реакций, в результате которых образуется сернистый газ.

Ответ:

S + O2 ⟶ SO2

2H2S + 3O2 ⟶ 2SO2 + 2H2O

PbO2 + 2S   t ⟶ PbS + SO2

5. Раствор сернистого газа в воде сохраняет запах сернистого газа. Какой вывод о силе сернистой кислоты вы можете сделать? Обоснуйте свой ответ.

Ответ:

Можно сделать вывод о том, что сернистая кислота является слабой и неустойчивой.
H2SO3 ⇄ H2O + SO2

6. При обжиге сульфида свинца (II) на воздухе образуется оксид свинца (II) и сернистый газ. Какой объём сернистого газа, измеренный при н. у., можно получить из 23,9 г сульфида свинца (II)?

Ответ:

Дано:

`m(PbS) = 23.9 г`

`V(SO_2) = ?`

Решение

`2PbS + 3O_2 ⟶ 2PbO + 2SO_2`

`n(PbS) = (m(PbS))/(M(PbS)) = 23.9/239 = 0.1" моль"`

`n(SO_2) = n(PbS) = 0.1" моль"`

`V(SO_2) = n(SO_2)*M(SO_2) = 0.1*22.4 = 2.24 л`

Ответ: `V(SO_2) = 2.24 л`.

7. Напишите уравнения реакций, отвечающие следующей схеме превращений:
S ⟶ H2S ⟶ K2S ⟶ FeS ⟶ H2S ⟶ SO2 ⟶ BaSO3.

Ответ:

H2 + S   t ⟶ H2S

2KOH + H2S ⟶ K2S + 2H2O

K2S + FeCl2 ⟶ 2KCl + FeS↓

FeS + 2HCl ⟶ FeCl2 + H2S↑

2H2S + 3O2 ⟶ 2SO2 + 2H2O

Ba(OH)2 + SO2 ⟶ BaSO3↓ + H2O

8. Сернистый газ подобно углекислому газу, вызывает помутнение известковой воды (раствора гидроксида кальция). Напишите уравнение реакции.

Ответ:

Ca(OH)2 + SO2 ⟶ CaSO3 + H2O

9. Если через йодную настойку пропустить ток сероводорода, бурая окраска исчезает, а раствор становится мутным. Объясните это явление.

Ответ:

Йод восстанавливает серу из сероводорода, поэтому бурая окраска раствора исчезает, и раствор становится мутным.
H2S + I2 ⟶ 2HI + S↓

10. Приведите уравнения реакций, соответствующие сокращённым ионным уравнениям:
Cu2+ + S2- = CuS;
FeS + 2H+ = Fe2+ + H2S;
SO2 + 2OH- = SO32- + H2O.

Ответ:

CuSO4 + Na2S = Na2SO4 + CuS↓

FeS + 2HCl = FeCl2 + H2S↑

SO2 + 2NaOH = Na2SO3 + H2O

11. Где в природе встречаются сероводород и сернистый газ?

Ответ:

Cероводород встречается в составе попутных нефтяных газов, природного газа, вулканических газах, в растворённом виде в природных водах. Образуется при гниении белков.

Сернистый газ в природе в основном встречается в вулканических газах.

Лабораторный опыт 7. Вытеснение галогенами друг друга из растворов солей.

В две пробирки налейте по 1 мл раствора иодида натрия. В одну из пробирок осторожно добавьте несколько капель хлорной воды, о в другую – бромной воды. Что наблюдаете? Проделайте аналогичный опыт с бромидом натрия, добавляя в него хлорную и йодную воду, и с хлоридом натрия, добавляя в него йодную и бромную воду. В каких случаях реакции протекают? Запишите уравнения реакций. Выпишите формулы простых веществ – галогенов в порядке уменьшения их химической активности.

Ответ:

При добавлении хлорной воды к раствору иодида натрия наблюдается потемнение раствора:
2NaI + Cl2 ⟶ 2NaCl + I2

При добавлении бромной воды к раствору иодида натрия наблюдается потемнение раствора:
2NaI + Br2 ⟶ 2NaBr + I2

 

При добавлении хлорной воды к раствору бромида натрия наблюдается пожелтение раствора:
2NaBr + Cl2 ⟶ 2NaCl + Br2

При добавлении йодной воды к раствору бромида натрия изменений не наблюдается.
NaBr + I2 ⇸

 

При добавлении бромной и йодной воды к раствору хлорида натрия изменений не наблюдается:
NaCl + Br2 ⇸
NaCl + I2 ⇸

Ряд галогенов в порядке уменьшения их химической активности: F2, Cl2, Br2, I2.

§27

§27. Серная кислота

1. В двух закрытых склянках одинакового объёма находятся вода и концентрированная серная кислота. Как определить содержимое каждой склянки, не открывая их?

Ответ:

Необходимо перевернуть пробирки, вода сразу стечёт на другой конец пробирки, а серная кислота – нет, так как является гораздо более вязкой жидкостью, чем вода.

2. Неизвестная жидкость смешивается с водой. Её водный раствор окрашивает лакмус в красный цвет и даёт белый осадок с нитратом бария. Назовите неизвестную жидкость.

Ответ:

Так как вещество является жидкостью, а его водный раствор окрашивается лакмусом в красный цвет, то вещество является кислотой. С нитратом бария раствор серной кислоты даёт белый осадок, также в чистом виде серная кислота является жидкостью.
Ba(NO3)2 + H2SO4 ⟶ 2HNO3 + BaSO4
Это вещество – серная кислота.

3. Для производства сульфата свинца (II) используются нитрат свинца (II) и серная кислота. Напишите уравнение реакции производства сульфата свинца и объясните, почему она идёт до конца.

Ответ:

Pb(NO3)2 + H2SO4 ⟶ 2HNO3 + PbSO4
Реакция идёт до конца, потому что образуется нерастворимый сульфат свинца (II).

4. Какова массовая доля воды в медном купоросе?

Ответ:

Дано:

`CuSO_4*5H_2O`

`ω(H_2O) = ?`

Решение

`ω(H_2O) = (100*n(H_2O)*M(H_2O))/(M(CuSO_4*5H_2O)) = (100*5*18)/250 = 36%`

Ответ: `ω(H_2O) = 36%`.

5. Напишите уравнения реакций взаимодействия цинка с разбавленной и концентрированной серной кислотой.

Ответ:

Zn + H2SO4(разб.) ⟶ ZnSO4 + H2

4Zn + 5H2SO4(конц.) ⟶ 4ZnSO4 + H2S↑ + 4H2O

6. Сернистый газ можно получить взаимодействием концентрированной серной кислоты с ртутью. Напишите уравнение реакции, зная, что ртуть в образующейся соли двухвалентна.

Ответ:

Hg + 2H2SO4(конц.) ⟶ HgSO4 + SO2↑ + 2H2O

7. Почему серную кислоту иногда называют "хлебом" химической промышленности?

Ответ:

Серную кислоту иногда называют "хлебом" химической промышленности, потому нет почти ни одного производства, где бы она не применялась. Её используют для производства удобрений, пластмасс, искусственных волокон, очистки нефтепродуктов (бензина, керосина, смазочных масел) от вредных примесей, получения красителей, лекарств, взрывчатых веществ.

8. Какой элемент является окислителем при взаимодействии металла с разбавленной и концентрированной серной кислотой?

Ответ:

При взаимодействии металла с разбавленной серной кислотой окислителем является водород:

Fe + H2SO4 ⟶ FeSO4 + H2

Fe0 - 2ē ⟶ Fe+2 |2| 2 |1    восстановитель (окисление)
2H+ + 1ē ⟶ H20 |2| |1    окислитель (восстановление)

При взаимодействии металла с концентрированной серной кислотой окислителем является сера:

Cu + 2H2SO4(конц.) ⟶ CuSO4 + SO2↑ + 2H2O

Cu0 - 2ē ⟶ Cu+2 |2| 2 |1    восстановитель (окисление)
S+6 + 2ē ⟶ S+4 |2| |1    окислитель (восстановление)

9. Сколько граммов твёрдого гидроксида натрия потребуется для полной нейтрализации 200 г 9,8%-го раствора серной кислоты?

Ответ:

Дано:

`m("р-ра") = 200 г`

`ω(H_2SO_4) = 9.8%`

`m(NaOH) = ?`

Решение

`2NaOH + H_2SO_4 ⟶ Na_2SO_4 + H_2O`

`m(H_2SO_4) = (ω(H_2SO_4)*m("р-ра"))/100 = (9.8*200)/100 = 19.6 г`

`n(H_2SO_4) = (m(H_2SO_4))/(M(H_2SO_4)) = 19.6/98 = 0.2" моль"`

`n(NaOH) = 2*n(H_2SO_4) = 2*0.2 = 0.4" моль"`

`m(NaOH) = n(NaOH)*M(NaOH) = 0.4*40 = 16 г`

Ответ: `m(NaOH) = 16 г`.

10. Определите массовую долю меди в медно-цинковом сплаве, если известно, что 1 г сплава реагирует с разбавленной серной кислотой с выделением 112 мл газа (н. у.).

Ответ:

Дано:

`m("сплава") = 1 г = 1000" мг"`

`V(H_2) = 112" мл"`

`ω(Cu) = ?`

Решение

`Zn + H_2SO_4 ⟶ ZnSO_4 + H_2"↑"`

`n(H_2) = (V(H_2))/V_m = 112/22.4 = 5" ммоль"`

`n(Zn) = n(H_2) = 5" ммоль"`

`m(Zn) = n(Zn)*M(Zn) = 5*65 = 325" мг"`

`ω(Zn) = (100*m(Zn))/(m("сплава")) = (100*325)/1000 = 32.5%`

`ω(Cu) = 100 - ω(Zn) = 100 - 32.5 = 67.5%`

Ответ: `ω(Cu) = 67.5%`.

11. Под действием концентрированной серной кислоты сахароза C12H22O11 превращается в чёрную массу, из которой затем начинают выделяться газы. Объясните эти процессы и напишите уравнения реакций.

Ответ:

 Концентрированная серная кислота обладает сильными гигроскопическим свойствами, поэтому под действием этой кислоты сахароза теряет воду, и превращается в уголь. Отнятая вода гидратирует молекулы серной кислоты, в результате чего выделяется большое количество тепла, которое нагревает воду до кипения.
C12H22O11   H₂SO₄ ⟶ 12C + 11H2O

12. Концентрированная серная кислота используется для осушения газов. Какие из перечисленных ниже газов: воздух, водород, аммиак, углекислый газ, сероводород – можно осушать с помощью H2SO4?

Ответ:

С помощью концентрированной серной кислоты можно осушать воздух и углекислый газ, так как они не реагируют с H2SO4.

Концентрированная H2SO4 восстанавливается аммиаком, сероводородом и частично водородом, из-за чего не может применяться для их осушки.

Лабораторный опыт 8. Изучение свойств серной кислоты.

1. Налейте в пробирку 1 мл разбавленной серной кислоты и добавьте в неё лакмус. Как изменился цвет индикатора?

2. Положите в пробирку кусок цинка и добавьте к нему разбавленную серную кислоту. Что наблюдаете? Проведите аналогичные опыты с алюминиевой и медной проволокой.

3. Получите осадок гидроксида меди (II) и прилейте к нему разбавленную серную кислоту. Что наблюдаете?

4. На кусок фильтровальной бумаги нанесите несколько капель раствора серной кислоты и осторожно нагрейте его, держа над плиткой. Что происходит? Объясните наблюдения, зная, что при нагревании вода, содержащаяся в растворе кислоты, испаряется и концентрация раствора возрастает.

5. К раствору серной кислоты добавьте одну-две капли раствора хлорида бария. Запишите цвет выпавшего осадка. Проделайте аналогичный опыт с растворами сульфата, карбоната и хлорида натрия, подкисленными серной кислотой. Опишите качественную реакцию на серную кислоту и её соли.

Ответ:

1. Цвет лакмуса в растворе серной кислоты изменяется на красный.

2. При добавлении кусочка цинка к разбавленной серной кислоте наблюдается выделение бесцветного газа на поверхности цинка.
Zn + H2SO4 ⟶ ZnSO4 + H2

При добавлении кусочка алюминия к разбавленной серной кислоте наблюдается выделение бесцветного газа на поверхности алюминия.
2Al + 3H2SO4 ⟶ Al2(SO4)3 + 3H2

При добавлении кусочка меди к разбавленной серной кислоте изменений не наблюдается.
Cu + H2SO4 ⇸

3. Получим осадок гидроксида меди (II) с помощью раствора щелочи и раствора сульфата меди (II):
2NaOH + CuSO4 ⟶ Na2SO4 + Cu(OH)2

При добавлении серной кислоты к гидроксиду меди (II) наблюдается растворение осадка:
Cu(OH)2 + H2SO4 ⟶ CuSO4 + 2H2O

4. Концентрированная серная кислота обугливает лист бумаги, значит концентрированная серная кислота обладает сильными гигроскопическими свойствами.

5. При добавлении хлорида бария к раствору серной кислоты наблюдается выпадение белого осадка.
BaCl2 + H2SO4 ⟶ 2HCl + BaSO4

При добавлении хлорида бария к раствору сульфата натрия наблюдается выпадение белого осадка.
BaCl2 + Na2SO4 ⟶ 2NaCl + BaSO4
При подкислении изменений не наблюдается.

При добавлении хлорида бария к раствору карбоната натрия наблюдается выпадение белого осадка.
BaCl2 + Na2CO3 ⟶ 2NaCl + BaCO3
При подкислении серной кислотой наблюдается выделение газа:
BaCO3 + H2SO4 ⟶ BaSO4 + H2O + CO2

При добавлении хлорида бария к хлорида натрия изменений не наблюдается.

Качественной реакцией на серную кислоту и её соли является реакция с растворимым соединением бария. В результате такой реакции выпадет белый осадок не взаимодействующий с кислотами.

§28

§28. Азот

1. Опишите физические свойства азота.

Ответ:

Азот – газ без цвета и запаха, он немного легче воздуха. Азот превращается в жидкость при очень низкой температуре. Азот кипит при более низкой температуре, чем кислород. Азот малорастворим в воде: при комнатной температуре в 100 объёмах воды растворяется только два объёма азота.

2. При взаимодействии азота с кислородом образуется оксид азота (II), при взаимодействии азота с водородом – аммиак NH3, а при взаимодействии азота с литием – нитрид лития Li3N. Напишите уравнения реакций, укажите условия их протекания. В каких случаях азот выступает как окислитель, а в каком – как восстановитель?

Ответ:

N2 + O2 `overset(t)(⟶)` 2NO

N2 + 3H2 `overset(t," "p," кат.")(⇄)` 2NH3

6Li + N2 ⟶ 2Li3N

3. Объясните, почему галогены не встречаются в природе в виде простых веществ, а азот, наоборот, более распространён в форме простого вещества.

Ответ:

Азот и галогены являются наиболее электроотрицательными элементами. Однако, молекулы азота с трудом вступают в химические реакции, так как атомы в молекулах азота связаны тройными связями, поэтому азот в природе более распространён в форме простого вещества. Галогены, напротив, гораздо легче вступают в химические реакции, так как атомы в молекулах галогенов связаны одинарными связями, поэтому галогены не встречаются в природе в виде простых веществ.

4. Найдите массовую долю азота в калийной селитре – нитрате калия.

Ответ:

Дано:

`KNO_3`

`ω(N) = ?`

Решение

`ω(N) = (100*n(N)*M(N))/(M(KNO_3)) = (100*1*14)/101 = 13.9%`

Ответ: `ω(N) = 13.9%`.

5. Объясните, почему фтор и кислород поддерживают горение, а азот – нет.

Ответ:

В молекуле фтора между атомами одинарная связь, в молекуле кислорода – двойная связь, а в молекуле азота – тройная связь. Одинарную и двойную связи разрушить легче, чем тройную, именно этим объясняется, что фтор и кислород поддерживают горение, а азот – нет.

6. Природный азот состоит из двух изотопов: 14N и 15N. Сколько разных молекул азота N2 существует в природе? Каковы их относительные молекулярные массы?

Ответ:

В природе существует три различных молекул азота:

  Mr
14N14N 28
14N15N 29
15N15N 30

§29

§29. Аммиак

1. Опишите физические и химические свойства аммиака.

Ответ:

Физические свойства аммиака

Аммиак – бесцветный газ с очень резким характерным запахом. Аммиак сравнительно легко превращающихся в жидкость. Для перевода его в жидкое состояние при 0°C достаточно давления 4,2 атм. Аммиак прекрасно растворим в воде, в одном объёме воды при комнатной температуре может быть растворено до 700 объёмов аммиака.

Химические свойства аммиака

На воздухе аммиак горит с трудом, но в кислороде сгорает легко с образованием азота и водяного пара:
4NH3 + 3O2 ⟶ 2N2 + 6H2O

Если смесь аммиака с кислородом пропускать над платинированным асбестом, то реакция протекает по другому пути:
4NH3 + 5O2   t, Pt ⟶ 4NO + 6H2O

Водный раствор аммиака окрашивает лакмус в синий цвет, а фенолфталеин — в малиновый, так как аммиак частично реагирует с молекулами воды, присоединяя водород и оставляя гидроксид-анионы:
NH3 + H2O ⇄ NH4+ + OH-

Аммиак легко реагирует с кислотами, образуя соли:
NH3 + HCl ⟶ NH4Cl

2. Изобразите строение молекулы аммиака и иона аммония.

Ответ:

      H
      ..
H : N :
      ..
     H
Молекула аммиака
      H
      ..
[H : N : H]+
       ..
       H
Ион аммония

3. Напишите формулы нитрата, хлорида, сульфата, карбоната, фосфата аммония.

Ответ:

NH4NO3

NH4Cl

(NH4)2SO4

(NH4)2CO3

(NH4)3PO4

4. Предложите способ очистки воздуха от примеси аммиака.

Ответ:

Можно несколько раз пропустить воздух через воду, так как аммиак хорошо растворяется в воде.

5. Сколько граммов нитрата аммония образуется при взаимодействии 6,8 г аммиака с азотной кислотой?

Ответ:

Дано:

`m(NH_3) = 6.8 г`

`m(NH_4NO_3) = ?`

Решение

`NH_3 + HNO_3 ⟶ NH_4NO_3`

`n(NH_3) = (m(NH_3))/(M(NH_3)) = 6.8/17 = 0.4" моль"`

`n(NH_4NO_3) = n(NH_3) = 0.4" моль"`

`m(NH_4NO_3) = n(NH_4NO_3)*M(NH_4NO_3) = 0.4*80 = 32 г`

Ответ: `m(NH_4NO_3) = 32 г`.

6. Сравните по химическим свойствам аммиак, хлороводород и сероводород.

Ответ:

  Аммиак Хлороводород Сероводород
Металлы Не реагирует Реагируют с металлами стоящими до водорода в ряду активности металлов.
Mg + 2HCl ⟶ MgCl2 + H2 Mg + H2S ⟶ MgCl2 + H2
Оксиды металлов Не реагирует Реагируют.
MgO + 2HCl ⟶ MgCl2 + H2O MgO + H2S ⟶ MgS + H2O
Кислород Сгорает:
4NH3 + 3O2 ⟶ 2N2 + 6H2O
Не реагирует. Сгорает:
2H2S + 3O2 ⟶ 2SO2 + 2H2O
Щелочи Не реагирует Реагируют.
NaOH + HCl ⟶ NaCl + H2O 2NaOH + H2S ⟶ Na2S + 2H2O
Кислоты NH3 + HCl ⟶ NH4Cl Не реагируют.
Окраска лакмуса Синяя. Красная. Красная.
ОВР Проявляет сильные восстановительные свойства. С сильными окислителями проявляет восстановительные свойства. С металлами проявляет окислительные свойства. Проявляет сильные восстановительные свойства. С металлами проявляет окислительные свойства.

7. Какие частицы присутствуют в водном растворе аммиака? Ответ подтвердите уравнением реакции.

Ответ:

В водном растворе аммиака присутствуют молекулы аммиака, молекулы гидроксида аммония, катионы аммония и гидроксид-анионы:
NH3 + H2O ⇄ NH4OH
NH4OH ⇄ NH4+ + OH-

8. Что общего в свойствах гидроксида натрия и водного раствора аммиака? Напишите уравнения реакций взаимодействия этих веществ с серной кислотой, сульфатом алюминия.

Ответ:

Гидроксид натрия и водный раствор аммиака обладают основными свойствами.

2NaOH + H2SO4 ⟶ Na2SO4 + 2H2O

2NH3 + H2SO4 ⟶ (NH4)2SO4

6NaOH + Al2(SO4)3 ⟶ 3Na2SO4 + 2Al(OH)3

6NH4OH + Al2(SO4)3 ⟶ 3(NH4)2SO4 + 2Al(OH)3

9. Напишите уравнения реакций, соответствующих схеме превращений: N2 ⟶ NH3 ⟶ NH4NO3 ⟶ NH3 ⟶ NO.

Ответ:

N2 + 3H2 `overset(t," "p," кат.")(⇄)` 2NH3

NH3 + HNO3 ⟶ NH4NO3

NH4NO3 + KOH   t ⟶ KNO3 + NH3↑ + H2O

4NH3 + 5O2   t, Pt ⟶ 4NO + 6H2O

10. Неизвестная соль при нагревании разлагается, с раствором нитрата серебра даёт белый творожистый осадок, а с гидроксидом натрия реагирует с выделением газа. Назовите соль и напишите уравнения реакций.

Ответ:

Это хлорид аммония:
AgNO3 + NH4Cl ⟶ NH4NO3 + AgCl↓
NaOH + NH4Cl ⟶ NaCl + NH3↑ + H2O

Лабораторный опыт 9. Изучение свойств водного раствора аммиака.

1. Откройте склянку с водным раствором аммиака и определите его запах.

2. Налейте в пробирку 1 мл раствора аммиака и добавьте каплю фенолфталеина Что наблюдаете? Сделайте вывод о кислотно-основных свойствах раствора аммиака.

3. Прилейте к приготовленному вами раствору аммиака, содержащему фенолфталеин, серную кислоту до исчезновения окраски индикатора. Напишите уравнение реакции.

4. Добавьте к раствору аммиака одну каплю раствора перманганата калия и нагрейте раствор. Что происходит? Бурый осадок – это оксид марганца (IV), а выделяющийся газ – азот. В какой роли выступает аммиак в этом опыте?

Ответ:

1. Аммиак обладает резким специфическим запахом.

2. Фенолфталеин окрашивает раствора аммиака в малиновый цвет, что говорит о щелочных свойствах раствора аммиака.

3

2NH3 + H2SO4 ⟶ (NH4)2SO4

4. При добавлении раствора перманганата калия к раствору аммиака наблюдается выпадение осадка и выделение газа. В этой реакции аммиак выступает в роли восстановителя.

2NH3 + 2KMnO4 ⟶ 2KOH + 2MnO2↓ + N2↑ + 2H2O

2N0 - 6ē ⟶ N20 |6| 6 |1    восстановитель (окисление)
Mn+7 + 3ē ⟶ Mn+4 |3| |2    окислитель (восстановление)

§30

§30. Азотная кислота

1. Какую степень окисления имеет азот в азотной кислоте и её солях? О каких свойствах этих соединений это свидетельствует?

Ответ:

Азот, в азотной кислоте и в её солях, находится в своей максимальной степени окисления +5, что свидетельствует о том, что эти соединения могут проявлять только окислительные свойства.

2. При действии аммиака на азотную кислоту образуется нитрат аммония, используемый в качестве удобрения. Напишите уравнение реакции.

Ответ:

NH3 + HNO3 ⟶ NH4NO3

3. В трёх склянках без этикеток находятся соляная, серная и азотная кислоты. Как химическим путём определить содержимое каждой склянки? Напишите уравнения реакций.

Ответ:

1. Найти серную кислоту с помощью хлорида бария, в результате их взаимодействия образуется белый осадок:
BaCl2 + H2SO4 ⟶ 2HCl + BaSO4

2. Найти соляную кислоту с помощью нитрата серебра, в результате их взаимодействия образуется белый творожистый осадок:
AgNO3 + HCl ⟶ HNO3 + AgCl↓

3. Найти азотную кислоту с помощью кусочка меди, в результате их взаимодействия образуется газ:
3Cu + 8HNO3(разб.) ⟶ 3Cu(NO3)2 + 2NO↑ + 4H2O

4. Объясните, почему реакция между разбавленным раствором серной кислоты и нитратом калия не протекает до конца, а при действии на твёрдую соль концентрированной серной кислоты образуется азотная кислота.

Ответ:

Реакция между разбавленным раствором серной кислоты и нитратом калия не протекает до конца, потому что все вещества, являясь сильными электролитами, находятся в диссоциированном состоянии.
KNO3(р) + H2SO4(разб.) ⇄ HNO3 + KHSO4
K+ + NO3- + H+ + HSO4- ⇄ H+ + NO3- + K+ + HSO4-

При действии на твёрдый нитрат калия концентрированной серной кислоты образуется азотная кислота, потому что она уходит из зоны реакции в виде газа:
KNO3(т) + H2SO4(конц.) ⟶ HNO3↑ + KHSO4

5. Как можно получить азотную кислоту в лаборатории?

Ответ:

Азотную кислоту в лаборатории можно получить с помощью концентрированной серной кислоты и нитрата натрия:
NaNO3(т) + H2SO4(конц.) ⟶ HNO3↑ + NaHSO4

6. Какую массу меди требуется взять для получения 5,6 л (н. у.) оксида азота (IV)?

Ответ:

Дано:

`V(NO_2) = 5.6 л`

`m(Cu) = ?`

Решение

`Cu + 4HNO_(3"(конц.)") ⟶ Cu(NO_3)_2 + 2NO_2"↑" + 2H_2O`

`n(NO_2) = (V(NO_2))/V_m = 5.6/22.4 = 0.25" моль"`

`n(Cu) = (n(NO_2))/2 = 0.25/2 = 0.125" моль"`

`m(Cu) = n(Cu)*M(Cu) = 0.125*64 = 8 г`

Ответ: `m(Cu) = 8 г`.

7. Сколько килограммов нитрата натрия требуется для приготовления 2 кг 63%-го раствора азотной кислоты?

Ответ:

Дано:

`m("р-ра") = 2" кг"`

`ω(HNO_3) = 63%`

`m(NaNO_3) = ?`

Решение

`NaNO_(3(т)) + H_2SO_(4"(конц.)") ⟶ HNO_3"↑" + NaHSO_4`

`m(HNO_3) = (ω(HNO_3)*m("р-ра"))/100 = (63*2)/100 = 1.26" кг"`

`n(HNO_3) = (m(HNO_3))/(M(HNO_3)) = 1.26/63 = 0.02" кмоль"`

`n(NaNO_3) = n(HNO_3) = 0.02" кмоль"`

`m(NaNO_3) = n(NaNO_3)*M(NaNO_3) = 0.02*85 = 1.7" кг"`

Ответ: `m(NaNO_3) = 1.7" кг"`.

8. На каком свойстве селитры основано использование пороха?

Ответ:

Использование селитры в порохе основано на её окислительных свойствах.

2KNO3 + 3C + S = K2S + N2 + 3CO2

9. Сульфат аммония наряду с аммиачной селитрой является важным азотным удобрением. Предложите метод его получения из азота.

Ответ:

Из азота получить аммиак:
N2 + 3H2 `overset(t," "p," кат.")(⇄)` 2NH3

Из аммиака получить сульфат аммония:
2NH3 + H2SO4 ⟶ (NH4)2SO4

10. Как реагирует разбавленная азотная кислота с медью, оксидом меди (II), карбонатом натрия, аммиаком? Напишите уравнения реакций.

Ответ:

3Cu + 8HNO3(разб.) ⟶ 3Cu(NO3)2 + 2NO↑ + 4H2O

CuO + 2HNO3 ⟶ Cu(NO3)2 + H2O

Na2CO3 + 2HNO3 ⟶ 2NaNO3 + H2O + CO2

NH3 + HNO3 ⟶ NH4NO3

11. Как в одну стадию осуществить следующие превращения: а) HCl ⟶ HNO3; б) H2SO4 ⟶ HNO3; в) KNO3 ⟶ HNO3? Приведите уравнения реакций и укажите их условия.

Ответ:

а) HCl + AgNO3 ⟶ HNO3 + AgCl↓

б) Ba(NO3)2 + H2SO4 ⟶ 2HNO3 + BaSO4

в) KNO3(т) + H2SO4(конц.) ⟶ HNO3↑ + KHSO4

12. В трёх пробирках без этикеток находятся водные растворы азотной кислоты, нитрата аммония и нитрата магния. Как с помощью одного реактива различить эти растворы? Напишите уравнения реакций и укажите их признаки.

Ответ:

Растворы азотной кислоты, нитрата аммония и нитрата магния можно различить с помощью щелочи, например, гидроксида натрия.

При взаимодействии с азотной кислоты будет наблюдаться нагревание пробирки:
NaOH + HNO3 ⟶ NaNO3 + H2O

При взаимодействии с нитратом аммония будет наблюдаться появление запаха аммиака:
NaOH + NH4NO3 ⟶ NaNO3 + NH3↑ + H2O

При взаимодействии с нитратом магния будет наблюдаться появление белого осадка:
2NaOH + Mg(NO3)2 ⟶ 2NaNO3 + Mg(OH)2

13. Неизвестный нитрат при разложении не даёт твёрдого остатка. Определите формулу нитрата и напишите уравнение реакции.

Ответ:

2NH4NO3 = 2N2 + O2 + 4H2O

§31

§31. Фосфор

1. Сопоставьте свойства белого и красного фосфора.

Ответ:

  Белый фосфор Красный фосфор
Агрегатное состояние при н. у. Твёрдое вещество Твёрдое вещество
Цвет Бесцветный/желтоватый Темно-красный
Плотность 1,8 г/см3 2,3 г/см3
Растворимость в воде Не растворяется Не растворяется
Температура плавления 44°C При атмосферном давлении и сильном нагревании превращается в пары белого фосфора.
Температура воспламенения 40°C. В измельченном состоянии воспламеняется при обычной температуре. Примерно 260°C.
Свечение В темноте светится. Не светится.
Действие на организм Сильный яд. Не ядовит.

2. Как вы думаете, какую кристаллическую решётку имеет белый фосфор? Мотивируйте свой ответ.

Ответ:

Белый фосфор имеет молекулярную кристаллическую решетку, так является мягким веществом с низкими температурами плавления и кипения.

3. Какое применение находит фосфор в повседневной жизни?

Ответ:

Фосфор в повседневной жизни используется в спичках (на боковых сторонах коробка).

4. Какие вещества нанесены на головку спички и на торцевые стороны спичечного коробка?

Ответ:

На головку спички нанесена бертолетова соль с горючим (например, сульфидом сурьмы), а на торцевые стороны спичечного коробка – красный фосфор с тонко измельчённым стеклом.

5. Почему белый фосфор светится в темноте?

Ответ:

Белый фосфор светится в темноте вследствие его окисления в кислороде воздуха.

6. Какую массу фосфорного ангидрида можно получить сжиганием 9,1 г фосфора?

Ответ:

Дано:

`m(P) = 9.1 г`

`m(P_2O_5) = ?`

Решение

`4P + 5O_2 = 2P_2O_5`

`n(P) = (m(P))/(M(P)) = 9.1/31 = 0.294" моль"`

`n(P_2O_5) = (n(P))/2 = 0.294/2 = 0.147" моль"`

`m(P_2O_5) = n(P_2O_5)*M(P_2O_5) = 0.147*142 = 20.9 г`

Ответ: `m(P_2O_5) = 20.9 г`.

7. Сколько молекул содержится в 24,8 г белого фосфора?

Ответ:

Дано:

`m(P_4) = 24.8 г`

`N(P_4) = ?`

Решение

`n(P_4) = (m(P_4))/(M(P_4)) = 24.8/124 = 0.2" моль"`

`N(P_4) = n(P_4)*N_A = 0.2*6*10^23 = 1.2*10^23`

Ответ: `N(P_4) = 1.2*10^23`.

8. Рассчитайте массовую долю фосфора в следующих соединениях: гидроксоапатит Ca5(OH)(PO4)3 (входит в состав костей), фторапатит Ca5F(PO4)3 (входит в состав зубной эмали) и аденозинфосфат C10H14N5O7P (входит в состав клеток и тканей).

Ответ:

Дано:

`"1) "Ca_5(OH)(PO_4)_3`

`"2) "Ca_5F(PO_4)_3`

`"3) "C_10H_14N_5O_7P`

`ω_1(P) = ?`

`ω_2(P) = ?`

`ω_3(P) = ?`

Решение

`ω_1(P) = (100*n(P)*M(P))/(M(Ca_5(OH)(PO_4)_3)) = (100*3*31)/502 = 18.53%`

`ω_2(P) = (100*n(P)*M(P))/(M(Ca_5F(PO_4)_3)) = (100*3*31)/504 = 18.45%`

`ω_3(P) = (100*n(P)*M(P))/(M(C_10H_14N_5O_7P)) = (100*1*31)/347 = 8.93%`

Ответ: `ω_1(P) = 18.53%`, `ω_2(P) = 18.45%`, `ω_3(P) = 8.93%`.

§32

§32. Фосфорная кислота

1. Напишите формулы всех солей магния и фосфорной кислоты и назовите их.

Ответ:

`underset("гидроксофосфат магния")((MgOH)_3PO_4)`

`underset("фосфат магния")(Mg_3(PO_4)_2)`

`underset("гидрофосфат магния")(MgHPO_4)`

`underset("дигидрофосфат магния")(Mg(H_2PO_4)_2)`

2. Какие вещества могут быть получены при пропускании аммиака через раствор фосфорной кислоты? Напишите формулы этих веществ и уравнения реакций их получения.

Ответ:

`NH_3 + H_3PO_4 ⟶ underset("дигидрофосфат аммония")(NH_4H_2PO_4)`

`2NH_3 + H_3PO_4 ⟶ underset("гидрофосфат аммония")((NH_4)_2HPO_4)`

`3NH_3 + H_3PO_4 ⟶ underset("фосфат аммония")((NH_4)_3PO_4)`

3. Эффективность фосфорных удобрений принято оценивать, рассчитывая долю P2O5 в каждом из них. Рассчитайте долю P2O5 в простом и двойном суперфосфате.

Ответ:

В задаче не указаны соотношения дигидрофосфата кальция и гипса в простом суперфосфате. Допустим, формула простого суперфосфата `Ca(H_2PO_4)_2*CaSO_4*2H_2O`.

Дано:

`Ca(H_2PO_4)_2*CaSO_4*2H_2O`

`ω(P_2O_5) = ?`

Решение

`ω(P_2O_5) = (100*n(P_2O_5)*M(P_2O_5))/(M(Ca(H_2PO_4)_2*CaSO_4*2H_2O)) = (100*1*142)/406 = 35%`

Ответ: `ω(P_2O_5) = 35%`.


Дано:

`Ca(H_2PO_4)_2`

`ω(P_2O_5) = ?`

Решение

`ω(P_2O_5) = (100*n(P_2O_5)*M(P_2O_5))/(M(Ca(H_2PO_4)_2)) = (100*1*142)/234 = 60.7%`

Ответ: `ω(P_2O_5) = 60.7%`.

4. Обработка фосфорита Ca3(PO4)2 горячей концентрированной азотной кислотой вместо серной кислоты приводит к образованию раствора, действуя на который аммиаком получают комплексное удобрение. Напишите уравнения реакций.

Ответ:

Ca3(PO4)2 + 6HNO3(конц.) ⟶ 3Ca(NO3)2 + 2H3PO4

NH3 + H3PO4 ⟶ NH4H2PO4
2NH3 + H3PO4 ⟶ (NH4)2HPO4

5. Какое количество гидроксида натрия необходимо добавить к 4,8 моль фосфорной кислоты для получения: а) дигидрофосфата натрия; б) гидрофосфата натрия; в) эквимолярной (т. е. содержащей равные количества вещества) смеси гидрофосфата натрия и среднего фосфата натрия?

Ответ:

Дано:

`n(H_3PO_4) = 4.8" моль"`

`"в) "n(Na_2HPO_4) = n(Na_3PO_4)`

`n_а(NaOH) = ?`

`n_б(NaOH) = ?`

`n_в(NaOH) = ?`

Решение

а)

`NaOH + H_3PO_4 ⟶ NaH_2PO_4 + H_2O`
`n_а(NaOH) = n(H_3PO_4) = 4.8" моль"`

б)

`2NaOH + H_3PO_4 ⟶ Na_2HPO_4 + 2H_2O`
`n_б(NaOH) = 2*(n(H_3PO_4)) = 2*4.8 = 9.6" моль"`

в)

`5NaOH + 2H_3PO_4 ⟶ Na_3PO_4 + Na_2HPO_4 + 5H_2O`
`n_в(NaOH) = (5*(n(H_3PO_4)))/2 = (5*4.8)/2 = 12" моль"`

Ответ: `n_а(NaOH) = 4.8" моль"`, `n_б(NaOH) = 9.6" моль"`, `n_в(NaOH) = 12" моль"`.

6. Какие фосфорные удобрения вы знаете? Опишите их свойства.

Ответ:

Аммофос – азотно-фосфорное растворимое удобрение, состоит в основном из NH4H2PO4 и частично из (NH4)2HPO4. Рекомендуется его использование в начале сезона.

Суперфосфат – наиболее распространённое фосфорное удобрение, однако оно плохо растворимо в воде, представляет собой смесь Ca(H2PO4)2 и CaSO4∙2H2O.

Двойной суперфосфат – по сравнению с простым фосфатом не содержит гипс, благодаря этому оно лучше растворяется в воде, представляет собой Ca(H2PO4)2.

7. Как из белого фосфора получить гидрофосфат натрия? Напишите уравнения реакций.

Ответ:

1. Сжечь белый фосфор:
4P + 5O2 ⟶ 2P2O5

2. Прилить к образовавшемуся ангидриду фосфора раствор гидроксида натрия:
P2O5 + 6NaOH ⟶ 2Na3PO4 + 3H2O

3. Прилить фосфорную кислоту к полученному раствору фосфата натрия:
2Na3PO4 + H3PO4 ⟶ 3Na2HPO4

8. Составьте уравнения реакций в соответствии со схемой: P ⟶ P2O5 ⟶ H3PO4 ⟶ Ca(H2PO4)2 ⟶ Ca3(PO4)2 ⟶ H3PO4.

Ответ:

4P + 5O2 ⟶ 2P2O5

P2O5 + 3H2O ⟶ 2H3PO4

Ca(OH)2 + 2H3PO4(изб.) ⟶ Ca(H2PO4)2 + 2H2O

2Ca(OH)2 + Ca(H2PO4)2 ⟶ Ca3(PO4)2↓ + 4H2O

Ca3(PO4)2 + 6HNO3(конц.) ⟶ 3Ca(NO3)2 + 2H3PO4

§33

§33. Углерод

1. Сопоставьте строение и свойства алмаза и графита, заполняя таблицу 11.

Строение и свойства Алмаз Графит
1. Строение. Каркасная структура Слоистая структура
2. Цвет.    
3. Блеск.    
4. Растворимость.    
5. Твёрдость.    
6. Хрупкость.    
7. Электропроводность.    

Ответ:

Строение и свойства Алмаз Графит
1. Строение. Каркасная структура Слоистая структура
2. Цвет. Бесцветный Тёмно-серый
3. Блеск. Обладает стеклянным (алмазным) блеском. На срезе обладает металлическим блеском.
4. Растворимость. Не растворим в воде. Не растворим в воде.
5. Твёрдость. Самое твёрдое вещество. Мягкий.
6. Хрупкость. Хрупкий. Чаще хрупкий.
7. Электропроводность. Диэлектрик. Проводник.

2. Римский учёный I в. Плиний Старший писал об алмазе: "Когда удаётся успешно разбить алмаз, он раскалывается на такие мелкие крупинки, что их с трудом можно увидеть. Они нужны резчикам и, заключённые в железо, с лёгкостью делают вырезы на каком бы то ни было твёрдом материале". Какие свойства алмаза он упоминает?

Ответ:

Римский учёный I в. Плиний Старший упоминает о хрупкости и высокой твёрдости алмаза.

3. Алмаз "Орлов", подаренный императрице Екатерине II её фаворитом, весит 185 каратов. Сколько граммов это составляет? Сколько атомов углерода он содержит?

Ответ:

Дано:

`m(C) = 185" карат"`

`m(C) = ? г`

`N(C) = ?`

Решение

1 карат равен 0,2 грамма.

`m(C) = 0.2*185 = 37 г`

`n(C) = (m(C))/(M(C)) = 37/12 = 3.08" моль"`

`N(C) = n(C)*N_A = 3.08*6.02*10^23 = 1.85*10^24`

Ответ: `m(C) = 37 г`, `N(C) = 1.85*10^24`.

4. Воспользовавшись рисунком 75, расскажите о применении графита и алмаза. В каждом случае отметьте, на каком свойстве вещества основано его использование (например, благодаря электропроводности графит используют как материал для электродов и т. д.).

Ответ:

Благодаря высокой твёрдости алмаз используют как абразивный материал для режущего инструмента и наконечника буров.

Благодаря прозрачности и высокой преломляющей способности алмаз используют в ювелирных изделиях.

Благодаря электропроводности графит используют как материал для электродов.

Благодаря мягкости графит используют как смазочный материал и в качестве грифеля для карандашей.

Благодаря тому, что графит замедляет нейтроны, его используют в ядерных реакторах.

Благодаря высокой температурной стойкости графит используют для изготовления литейных форм.

5. Охарактеризуйте химические свойства углерода. Напишите по два уравнения реакций, в которых чистый углерод является: а) окислителем; б) восстановителем.

Ответ:

Химические свойства углерода

При комнатной температуре углерод не вступает в химические реакции с большинством веществ. Углерод устойчив к действию растворов кислот, но окисляется до углекислого газа горячими концентрированными растворами серной и азотной кислот. С металлами углерод образует солеобразные вещества – карбиды. Углерод – хороший восстановитель, подобно водороду, используемый для получения металлов из оксидов.

При нормальных условиях углерод – малоактивное вещество. Особую химическую инертность проявляет такая его аллотропная модификация, как алмаз.

При нагреве углерод сгорает в кислороде воздуха с образованием углекислого газа, проявляя восстановительные свойства:
`overset(0)(C) + overset(0)(O_2) = overset(+4)(C)overset(-2)(O_2)`

Восстановительные свойства углерод проявляет и в реакциях со сложными веществами – окислителями. Например, концентрированная серная кислота взаимодействует с углеродом, при этом наблюдается бурное выделение углекислого и сернистого газов:
`C + 2H_2SO_4 = CO_2"↑" + 2SO_2"↑" + 2H_2O`

При нагревании углерод взаимодействует с металлами. Как неметалл он проявляет в таких реакциях окислительные свойства. Образующиеся вещества носят общее название карбиды. Спекание кокса с алюминием приводит к получению карбида алюминия:
`overset(0)(4Al) + overset(0)(3C) overset(t)(=) overset(+3)(Al_4)overset(-4)(C_3)`

Углерод выступает в роли окислителя и при взаимодействии с неметаллами, имеющими меньшее значение электроотрицательности. Так, при прокаливании смеси углерода с кремнием получают карбид кремния, или карборунд:
`overset(0)(Si) + overset(0)(C) overset(t)(=) overset(+4)(Si)overset(-4)(C)`

а) C + 2H2   t ⟶ CH4

H20 - 2ē ⟶ 2H+ |2| 4 |2    восстановитель (окисление)
C0 + 4ē ⟶ C-4 |4| |1    окислитель (восстановление)

б) 2CuO + C   t ⟶ 2Cu + CO2

C0 - 4ē ⟶ C+4 |4| 4 |1    восстановитель (окисление)
Cu+2 + 2ē ⟶ Cu0 |2| |2    окислитель (восстановление)

6. Напишите уравнения реакций восстановления углём свинца из оксида свинца (II), железа – из оксида железа (III), меди – из оксида меди (I). В каждом случае назовите окислитель и восстановитель.

Ответ:

2PbO + C   t ⟶ 2Pb + CO2

C0 - 4ē ⟶ C+4 |4| 4 |1    восстановитель (окисление)
Pb+2 + 2ē ⟶ Pb0 |2| |2    окислитель (восстановление)

2Fe2O3 + 3C   t ⟶ 4Fe + 3CO2

C0 - 4ē ⟶ C+4 |4| 12 |3    восстановитель (окисление)
Fe+3 + 3ē ⟶ Fe0 |3| |4    окислитель (восстановление)

2Cu2O + C   t ⟶ 4Cu + CO2

C0 - 4ē ⟶ C+4 |4| 4 |1    восстановитель (окисление)
Cu+ + 1ē ⟶ Cu0 |1| |4    окислитель (восстановление)

7. Какой объём углекислого газа, измеренный при н. у., образуется при окислении 6 г угля: а) концентрированной серной кислотой; б) концентрированной азотной кислотой; в) при сжигании такого же количества угля? Решите задачу устно и мотивируйте свой ответ.

Ответ:

Во всех случаях при окислении 6 г (0,5 моль) угля может образоваться 0,5 моль углекислого газа или 11,2 л.

§34

§34. Уголь

1. Что представляют собой древесный уголь; сажа; каменный уголь; кокс?

Ответ:

Древесный уголь представляют собой твёрдую темно-серую пористую массу, повторяющая структуру исходной древесины.

Сажа представляют собой чёрный, жирный на ощупь порошок

Каменный уголь представляют собой продукт разложения без доступа воздуха древней растительности.

Кокс представляют собой чёрную массу из аморфного углерода, похожую на сажу.

2. Какой уголь называют активированным? Где он используется?

Ответ:

Уголь, обработанный водяным паром, называют активированным.

Активированный уголь применяется для производства противогазов, фильтров, фармакологических адсорбентов.

3. Что такое адсорбция? Приведите пример.

Ответ:

Адсорбция – способность поглощать газы и растворённые вещества.

Например, активированный уголь адсорбирует оксид азота (IV).

4. Сажа, наряду с водородом, образуется при пиролизе метана CH4 – нагревании его без доступа воздуха. Напишите уравнение реакции. Рассчитайте массу сажи, образующейся при пиролизе 56 м3 метана, если выход продукта реакции составляет 95%.

Ответ:

Дано:

`V(CH_4) = 56 м^3`

`η(C) = 95%`

`m_("практ.")(C) = ?`

Решение

`CH_4 overset(t)(⟶) C + 2H_2`

`n(CH_4) = (V(CH_4))/V_m = 56/22.4 = 2.5" кмоль"`

`n_("теор.")(C) = n(CH_4) = 2.5" кмоль"`

`m_("теор.")(C) = n_("теор.")(C)*M(C) = 2.5*12 = 30" кг"`

`m_("практ.")(C) = (η(C)*m_("теор.")(C))/100 = (95*30)/100 = 28.5" кг"`

Ответ: `m_("практ.")(C) = 28.5" кг"`.

5. Пользуясь рисунком 78, опишите устройство противогаза.

Ответ:

Противогаз представляет собой маску, со стеклянными вставками для глаз. В нижней части маски имеется клапан для выдыхания, к которому присоединена гофрированная трубка. На другом конце гофрированная трубка соединена с коробкой, в которой находится слой поглотителя. На противоположной стороне коробки с поглотителем находится клапан для вдыхания.

6. На какой реакции основано применение каменного угля в качестве исходного вещества для получения водорода?

Ответ:

Применение каменного угля в качестве исходного вещества для получения водорода основано на реакции:
C + H2  1000°C ⟶ CO + H2

7. Каменный уголь используют на теплоэлектростанциях для производства энергии. К каким экологическим последствиям приводит работа таких станций?

Ответ:

Уголь содержит в небольшом количестве тяжелые металлы и радиоактивные изотопы некоторых элементов, поэтому при его сжигании они попадают в атмосферу, и осаждаются вокруг станции, тем самым загрязняя почву надолго. Уголь также содержит некоторое количество серы, поэтому сжигание угля является одной из причин кислотных дождей.

§35

§35. Угарный и углекислый газы

1. Опишите физические свойства углекислого газа.

Ответ:

Углекислый газ – это бесцветный газ с кисловатым запахом и вкусом, он почти в полтора раза тяжелее воздуха. При 0°C и давлении 38 атм углекислый газ превращается в легкоподвижную жидкость, которая, испаряясь, частично застывает в белую массу – «сухой лёд». В одном объёме воды при комнатной температуре и атмосферном давлении растворяется равный объём углекислого газа. Растворимость углекислого газа в воде можно существенно увеличить, повысив давление.

2. На чём основано использование углекислого газа при тушении пожаров, при приготовлении газированных напитков?

Ответ:

Использование углекислого газа при тушении пожаров основано на том, что он не поддерживает горение.

Использование углекислого газа при приготовлении газированных напитков основано на том, что он хорошо растворим в воде при повышенном давлении.

3. Если на кусок мрамора подействовать серной кислотой вместо соляной, выделение углекислого газа быстро прекращается. Чем это можно объяснить?

Ответ:

Если на кусок мрамора подействовать серной кислотой, то на поверхности мрамора образуется нерастворимый сульфат кальция, который препятствует дальнейшему протеканию химической реакции.
CaCO3 + H2SO4 ⟶ CaSO4 + H2O + CO2

4. Известны случаи смертельных отравлений углекислым газом, скопившимся на дне погребов, в которых хранятся овощи. Как следует убедиться в безопасности нахождения в погребе?

Ответ:

Можно зажечь спичку, если она погаснет, то в погребе находиться небезопасно.

5. Что такое парниковый эффект и каковы его возможные последствия?

Ответ:

Парниковый эффект – это повышение температуры поверхности земли по причине нагрева нижних слоев атмосферы скоплением парниковых газов.

Последствия парникового эффекта:

  • таяние ледников;
  • повышение уровня моря;
  • изменение циркуляции океанических течений;
  • ослабление/усиление муссонов;
  • таяние вечной мерзлоты.

6. Расскажите об использовании углекислого газа (рис. 84).

Ответ:

Углекислый газ применяется как средство для тушения пожаров. В пищевой промышленности CO2 используется как консервант, разрыхлитель, а также используется для газирования напитков. Углекислый газ в виде сухого льда используется как хладагент. Оксид углерода (IV) используется для производства карбонатов, например, поташа, кальцинированной, пищевой и кристаллической соды.

7. Почему при топке печи опасно преждевременно закрывать заслонку?

Ответ:

Если преждевременно закрывать заслонку при топке печи, то образовавшийся в печи углекислый газ восстановится углём до угарного газа, который является токсичным веществом.

8. Свойства угарного газа были подробно изучены в 1776 г химиком Ларссоном "Из смеси цинковой окалины (оксида цинка) и большого количества угольного порошка, заключённой в пистолетное дуло, я извлёк в жару кузнечного горна 96 л газа", – записал он в дневнике. Приведите уравнение реакции. Какую массу цинковой окалины взял учёный?

Ответ:

Дано:

`V(CO_2) = 96 л`

`m(ZnO) = ?`

Решение

`2ZnO + C ⟶ 2Zn + CO_2"↑"`

`n(CO_2) = (V(CO_2))/V_m = 96/22.4 = 4.29" моль"`

`n(ZnO) = 2*n(CO_2) = 2*4.29 = 8.58" моль"`

`m(ZnO) = n(ZnO)*M(ZnO) = 8.58*81 = 695 г`

Ответ: `m(ZnO) = 695 г`.

9. Сколько литров угарного газа (н. у.) необходимо для получения 1000 кДж теплоты, если известно, что при сгорании 1 г этого вещества выделяется 10,1 кДж?

Ответ:

Дано:

`Q = 1000" кДж"`

`Q_("сгор.") = 10.1" "("кДж")/г`

`V(CO) = ?`

Решение

`2CO + O_2 ⟶ 2CO_2`

`m(CO) = Q/(Q_("сгор.")) = 1000/10.1 = 99 г`

`n(CO) = (m(CO))/(M(CO)) = 99/28 = 3.54" моль"`

`V(CO) = n(CO)*V_m = 3.54*22.4 = 79.3 л`

Ответ: `V(CO) = 79.3 л`.

10. Какие первые признаки отравления угарным газом? Что нужно делать при отравлении?

Ответ:

Признаки отравления: головная боль и головокружение; отмечается шум в ушах, одышка, учащённое сердцебиение, мерцание перед глазами, покраснение лица, общая слабость, тошнота, иногда рвота; в тяжёлых случаях судороги, потеря сознания, кома.

При отравлении рекомендуются вынести пострадавшего на свежий воздух, и вызвать скорую помощь. При отравлении лёгкой степени достаточно гипервентиляции лёгких кислородом.

11. Сравните физические и химические свойства оксидов углерода.

Ответ:

Сравнение физических свойств оксидов углерода

Свойство при н. у. CO CO2
Агрегатное состояние Газ. Газ.
Цвет. Бесцветный. Бесцветный.
Запах. Не имеет. Не имеет.
Плотность по воздуху. 1,52 0,97
tзам. -205°C -78,5°C
tкип. -191,5°C Не имеет (сублимирует).

Сравнение химических свойств оксидов углерода

  CO CO2
Окислительно-восстановительные свойства Проявляет как окислительные так и восстановительные свойства, т. к. углерод находится в промежуточной степени окисления +2. Но всё же является сильным восстановителем. Проявляет только окислительные свойства, т. к. углерод находится максимальной степени окисления +4.
Взаимодействие с водой при н. у. Не реагирует, т. к. оксид несолеобразующий. CO2 + H2O ⇄ H2CO3
Взаимодействие с щелочами. Не реагирует, т. к. оксид несолеобразующий. 2NaOH + CO2 ⟶ Na2CO3 + H2O
Взаимодействие с оксидами металлов. Восстанавливает металлы из их оксидов:
CO + Fe2O3   t ⟶ Fe + CO2
Образует карбонаты:
MgO + CO2 ⟶ MgCO3
Взаимодействие с кислородом Сгорает в кислороде:
2CO + O2 ⟶ 2CO2
Не реагирует.
Взаимодействие с углеродом Не реагирует. Восстанавливается до оксида углерода (II):
C + CO2   t ⟶ 2CO

12. Напишите уравнения реакций по схеме: Ca(OH)2 ⟶ CaCO3 ⟶ CO2 ⟶ CO ⟶ CO2.

Ответ:

Ca(OH)2 + CO2 ⟶ CaCO3↓ + H2O

CaCO3 + 2HCl ⟶ CaCl2 + H2O + CO2

CO2 + C   t ⟶ 2CO

2CO + O2 ⟶ 2CO2

Лабораторный опыт 10. Изучение свойств раствора карбоната натрия.

Насыпьте в пробирку несколько кристалликов кальцинированной соды – среднего карбоната натрия. Добавьте в пробирку 3-4 мл воды и встряхните, ударяя по той части, где находится раствор, указательным пальцем. Сделайте вывод о растворимости соды в воде. Полученный раствор разделите на две части. К одной из них прилейте соляную кислоту, а к другой – раствор хлорида кальция. Что наблюдаете? Напишите уравнения реакций в молекулярном, полном и сокращённом ионном виде.

Ответ:

Карбоната натрия хорошо растворим в воде.

При добавлении соляной кислоты к раствору карбона натрия наблюдается выделение бесцветного газа.
Na2CO3 + 2HCl ⟶ 2NaCl + H2Cl + CO2
2Na+ + CO32- + 2H+ + 2Cl- ⟶ 2Na+ + 2Cl- + H2O + CO2
2H+ + CO32- ⟶ H2O + CO2

При добавлении хлорида кальция к раствору карбона натрия наблюдается выпадение осадка белого цвета.
Na2CO3 + CaCl2 ⟶ 2NaCl + CaCO3
2Na+ + CO32- + Ca2+ + 2Cl- ⟶ 2Na+ + 2Cl- + CaCO3
Ca2+ + CO32- ⟶ CaCO3

§36

§36. Угольная кислота и её соли

1. Напишите формулы карбонатов и гидрокарбонатов калия и кальция.

Ответ:

K2CO3 – карбонат калия

KHCO3 – гидрокарбонат калия

CaCO3 – карбонат кальция

Ca(HCO3)2 – гидрокарбонат кальция

2. Какие соли угольной кислоты встречаются в природе? Назовите 5-7 минералов.

Ответ:

Зелёный малахит – гидрокарбонат меди (II).

Витерит – карбонат бария.

Кальцит – карбонат кальция.

Магнезит – карбонат магния.

Смитсонит – карбонат цинка.

Сидерит – карбонат железа (II).

Церуссит – карбонат свинца (II).

Родохрозит – карбонат марганца (II).

Термонатрит – моногидрат карбоната натрия.

3. При кипячении раствор гидрокарбоната кальция разлагается, подобно гидрокарбонату натрия. Напишите уравнение реакции.

Ответ:

Ca(HCO3)2   t ⟶ CaCO3↓ + H2O + CO2

4. Как распознать, есть ли: а) в питьевой соде примесь поваренной соли; б) в поваренной соли примесь кальцинированной соды?

Ответ:

а)

1. Взять образец смеси, и растворить его в воде.

2. Нейтрализовать питьевую соду азотной кислотой:
NaHCO3 + HNO3 ⟶ NaNO3 + H2O + CO2

3. Прилить раствор нитрата серебра. Если в смеси имеется примесь поваренной соли, то образуется белый творожистый осадок хлорида серебра:
NaCl + AgNO3 ⟶ NaNO3 + AgCl↓

б)

1. Взять образец смеси, и растворить его в воде.

2. Прилить соляной кислоты, если образуется газ, недодерживающий горение, то в смеси присутствует кальцинированная сода.
Na2CO3 + 2HCl ⟶ 2NaCl + H2O + CO2

5. Получите из угля карбонат бария в минимальное число стадий.

Ответ:

C + O2 ⟶ CO2

CO2 + Ba(OH)2 ⟶ BaCO3↓ + H2O

6. Почему при кипячении раствора питьевой соды его щёлочность возрастает?

Ответ:

При кипячении раствора питьевой соды гидрокарбонат натрия распадается на карбонат натрия, углекислый газ и воду:
2NaHCO3   t ⟶ Na2CO3 + H2O + CO2

Гидролиз гидрокарбоната натрия протекает в одну стадию:
NaHCO3 + HOH ⇄ NaOH + H2CO3

Гидролиз карбоната натрия протекает в две стадии:
Na2CO3 + HOH ⇄ NaOH + NaHCO3
NaHCO3 + HOH ⇄ NaOH + H2CO3

Что говорит о том, что при гидролизе карбоната натрия образуется больше гидроксид-анионов, чем при гидролизе гидрокарбоната натрия. А чем больше концентрация гидроксид-анионов, тем более щелочной является среда.

7. При обработке 21,2 г смеси карбоната и сульфата натрия соляной кислотой выделилось 2,24 л газа (н. у.). Определите состав смеси в массовых процентах.

Ответ:

Дано:

`m("смеси") = 21.2 г`

`V(CO_2) = 2.24 л`

`ω(Na_2CO_3) = ?`

`ω(Na_2SO_4) = ?`

Решение

`Na_2CO_3 + 2HCl ⟶ 2NaCl + H_2O + CO_2"↑"`

`n(CO_2) = (V(CO_2))/V_m = 2.24/22.4 = 0.1" моль"`

`n(Na_2CO_3) = n(CO_2) = 0.1" моль"`

`m(Na_2CO_3) = n(Na_2CO_3)*M(Na_2CO_3) = 0.1*106 = 10.6 г`

`ω(Na_2CO_3) = (100*m(Na_2CO_3))/(m("смеси")) = (100*10.6)/21.2 = 50%`

`ω(Na_2SO_4) = 100 - ω(Na_2CO_3) = 100 - 50 = 50%`

Ответ: `ω(Na_2CO_3) = 50%`, `ω(Na_2SO_4) = 50%`.

8. Неизвестная соль при нагревании разлагается, не оставляя твёрдого остатка, а с растворами кислот и щелочей реагирует с образованием газа. О какой соли идёт речь? Предложите два варианта ответа.

Ответ:

Речь идёт о карбонате аммония или о гидрокарбонате аммония.

(NH4)2CO3   t ⟶ 2NH3↑ + H2O↑ + CO2
(NH4)2CO3 + 2HCl ⟶ 2NH4Cl + H2O + CO2
(NH4)2CO3 + 2NaOH ⟶ Na2CO3 + 2H2O + 2NH3

NH4HCO3   t ⟶ NH3↑ + H2O↑ + CO2
NH4HCO3 + HCl ⟶ NH4Cl + H2O + CO2
NH4HCO3 + 2NaOH ⟶ Na2CO3 + 2H2O + NH3

9. Напишите уравнения реакций, соответствующих схеме: Ca(HCO3)2 ⟶ NaHCO3 ⟶ Na2CO3 ⟶ CO2 ⟶ CaCO3.

Ответ:

Ca(HCO3)2 + Na2CO3 ⟶ 2NaHCO3 + CaCO3

2NaHCO3   t ⟶ Na2CO3 + H2O + CO2

Na2CO3 + 2HCl ⟶ 2NaCl + H2O + CO2

CO2 + Ca(OH)2 ⟶ CaCO3↓ + H2O

§37

§37. Круговорот углерода в природе

1. Опишите круговорот углерода в живой и неживой природе.

Ответ:

Содержащийся в атмосфере углекислый газ поглощается растениями в результате фотосинтеза, превращаясь в глюкозу и другие органические вещества. Атомы углерода в составе органических веществ последовательно переходят из одного живого организма в другой, от растений к животным, употребляющим их в пищу, и т. д., пока не попадают в землю в форме органических остатков. Под действием микроорганизмов они разлагаются, а углерод в форме углекислого газа возвращается в атмосферу.

Углекислый газ, содержащийся в атмосфере, участвует в процессах выветривания горных пород, превращаясь при этом в карбонаты кальция – гидрокарбонат Ca(HCO3)2, а из него – и в средний карбонат CaCO3. В результате вулканической деятельности карбонатные породы разлагаются, а образующийся при этом углекислый газ возвращается в атмосферу.

2. В виде каких минеральных и органических соединений углерод содержится в земной коре; Мировом океане?

Ответ:

Углерод содержится в земной коре и Мировом океане в виде углерода, углеводородов, карбонатов, гидрокарбонатов.

3. В одной из двух склянок имеется раствор гидроксида натрия, в другой – карбоната натрия. Как распознать содержимое каждой склянки? Приведите уравнения реакций.

Ответ:

Чтобы распознать растворы гидроксида натрия и карбоната натрия, необходимо к ним прилить соляную кислоту.

При добавлении соляной кислоты к раствору гидроксида натрия будет наблюдаться нагревание пробирки.
NaOH + HCl ⟶ NaCl + H2O + Q

При добавлении соляной кислоты к раствору гидроксида натрия будет наблюдаться выделение газа.
Na2CO3 + 2HCl ⟶ 2NaCl + H2O + CO2

4. Напишите уравнения реакций, соответствующие схеме:
C ⟶ CO2 ⟶ Na2CO3 ⟶ CaCO3 ⟶ CO2.

Ответ:

C + O2 ⟶ CO2

CO2 + 2NaOH ⟶ Na2CO3 + H2O

Na2CO3 + CaCl2 ⟶ 2NaCl + CaCO3

CaCO3   t ⟶ CaO + CO2

5. Схемы реакций, происходящих в процессе круговорота углерода, имеют вид:
CO2 ⟶ O2 ⟶ CO2 ⟶ Ca(HCO3)2 ⟶ CaCO3 ⟶ CO2.

Напишите уравнения реакций, соответствующие этим схемам.

Ответ:

6CO2 + 6H2  фотосинтез ⟶ C6H12O6 + 6O2

C + O2 ⟶ CO2

CO2 + H2O + CaCO3 ⟶ Ca(HCO3)2

Ca(HCO3)2   t ⟶ CaCO3 + H2O + CO2

CaCO3   t ⟶ CaO + CO2

§38

§38. Кремний и его соединения

1. Какова химическая формула кварца; горного хрусталя; яшмы?

Ответ:

Химическая формула кварца, горного хрусталя и яшмы – SiO2

2. Как можно из оксида кремния получить кремниевую кислоту? Напишите уравнения реакций.

Ответ:

1) Сплавить оксид кремния (IV) со щелочью:
SiO2 + 2NaOH   t ⟶ Na2SiO3 + H2O↑

2) Прилить образовавшемуся силикату натрия соляную кислоту:
Na2SiO3 + 2HCl ⟶ 2NaCl + H2SiO3

3. Какие продукты получают из глины? Где они применяются?

Ответ:

Из глины получают гончарные изделия (в быту), кирпичи (в строительстве), керамические изделия (в быту, в электротехнике).

4. Какие вещества называют аморфными? Приведите примеры.

Ответ:

Вещества, которые не имеет определённой кристаллической структуры, называются аморфными.

К аморфным веществам относятся: натриевое стекло, кварцевое стекло, парафин, полиэтилен.

5. Чем кварцевое стекло отличается от оконного?

Ответ:

Кварцевое стекло состоит из чистого оксида кремния (IV), а оконное стекло состоит из силикатов натрия и кальция.

6. Объясните, почему в водном растворе углекислый газ вытесняет из силиката натрия кремниевую кислоту, а при сплавлении кварца с содой реакция фактически протекает в обратном направлении.

Ответ:

Силикат натрия – соль кремниевой кислоты, а углекислый газ – ангидрид угольной кислоты. Так как кремниевая кислота крайне слабая, то её способна вытеснить из раствора соли даже угольная кислота, поэтому в водном растворе углекислый газ вытесняет из силиката натрия кремниевую кислоту.
Na2SiO3 + CO2 ⟶ Na2CO3 + SiO2

При сплавлении кварца с содой реакция фактически протекает в обратном направлении, потому что углекислый газ покидает зону реакции.
SiO2 + Na2CO3 ⟶ Na2SiO3 + CO2

7. Внутренняя поверхность стеклянных склянок, в которых длительное время хранились крепкие растворы щелочей, становится неровной. Чем это можно объяснить?

Ответ:

Стекло состоит из компонентов, которые медленно взаимодействует с крепким раствором щелочи (в основном из оксида кремния (IV)), и компонентов, которые не взаимодействуют со щелочами. Этим и объясняется неровная внутренняя поверхность стеклянных склянок, в которых длительное время хранились крепкие растворы щелочей.

8. Какую массу оксида кремния (IV) можно получить из 250 г 1,22%-го раствора силиката натрия?

Ответ:

Дано:

`m("р-ра") = 250 г`

`ω(Na_2SiO_3) = 1.22%`

`m(SiO_2) = ?`

Решение

`Na_2SiO_3 + 2HCl ⟶ 2NaCl + H_2O + SiO_2"↓"`

`m(Na_2SiO_3) = (ω(Na_2SiO_3)*m("р-ра"))/100 = (1.22*250)/100 = 3.05 г`

`n(Na_2SiO_3) = (m(Na_2SiO_3))/(M(Na_2SiO_3)) = 3.05/122 = 0.025" моль"`

`n(SiO_2) = n(Na_2SiO_3) = 0.025" моль"`

`m(SiO_2) = n(SiO_2)*M(SiO_2) = 0.025*60 = 1.5 г`

Ответ: `m(SiO_2) = 1.5 г`.

9. Оксиды кремния (IV) и углерода (IV) описываются одинаковой общей формулой – RO2. Почему их свойства сильно отличаются? Какие кристаллические решётки характерны для этих веществ в твёрдом состоянии? Существует ли молекула SiO2?

Ответ:

В твёрдом состоянии оксид углерода (IV) имеет молекулярную кристаллическую решетку, а оксид кремния (IV) – атомную, поэтому их свойства сильно отличаются.

В земных условиях молекулы SiO2 не существует, однако она была получена путём синтеза из оксида кремния (II) и кислорода в матрице аргона.

10. Напишите уравнения реакций, соответствующие схеме:
Si ⟶ SiO2 ⟶ Na2SiO3 ⟶ H2SiO3.

Ответ:

Si + O2   t ⟶ SiO2

SiO2 + 2NaOH   t ⟶ Na2SiO3 + H2O↑

Na2SiO3 + 2HCl ⟶ 2NaCl + H2SiO3

11. Обычно сырьём при варке стекла служат сода Na2CO3, известняк CaCO3 и кремнезём SiO2. Сколько килограммов каждого из этих веществ потребуется для производства 20 кг стекла состава Na2O∙CaO∙6SiO2?

Ответ:

Дано:

`m(Na_2O*CaO*6SiO_2) = 20" кг"`

`m(Na_2CO_3) = ?`

`m(CaCO_3) = ?`

`m(SiO_2) = ?`

Решение

`n(Na_2O*CaO*6SiO_2) = (m(Na_2O*CaO*6SiO_2))/(M(Na_2O*CaO*6SiO_2)) = 20/478 = 0.0418" кмоль"`

`n(Na_2O) = n(CaO) = n(Na_2O*CaO*6SiO_2) = 0.0418" кмоль"`

`m(Na_2O) = n(Na_2O)*M(Na_2O) = 0.0418*62 = 2.59" кг"`

`m(CaO) = n(CaO)*M(CaO) = 0.0418*56 = 2.34" кг"`

`m(SiO_2) = m(Na_2O*CaO*6SiO_2) - m(Na_2O) - m(CaO) = 20 - 2.59 - 2.34 = 15.07" кг"`

Ответ: `m(Na_2O) = 2.59" кг"`, `m(CaO) = 2.34" кг"`, `m(SiO_2) = 15.07" кг"`.

Творческие задания 2

Творческие задания 2

2. Используя справочную литературу и ресурсы Интернета, найдите названия и изображения минералов, состав которых соответствует формуле SO2.

Ответ:

SO2 - это кварц и его разновидности. В Википедии есть статья про "кварц".

3. Предложите способ определения наличия в кварцевой породе примеси карбонатов.

Ответ:

Для обнаружения примеси карбонатов в кварцевой породе, необходимо добавить к смеси соляной кислоты. Выделение газа, будет говорить о наличии карбонатов в породе.
CaCO3 + 2HCl ⟶ CaCl2 + H2O + CO2

4. Найдите в литературе и Интернете информацию о том, когда иодную настойку впервые начали использовать в медицине.

Ответ:

В 1908 г. хирург Антонио Гроссих впервые применил настойку йода для стерилизации кожи человека в операционном поле.

5. ==

Ответ:

 ==

6. Человек при дыхании расходует около 20 л кислорода и выделяет около 20 л углекислого газа в час. Оцените, какое время можно находиться в непроветриваемой комнате объёмом 50 м3 без угрозы для здоровья.

Ответ:

Безопасное содержание CO2 в воздухе до 0,1% по объёму.

Дано:

`V("комнаты") = 50 м^3 = 5*10^4 л`

`ν_"расх."(O_2) = ν_"выд."(CO_2) = 20" "л/ч`

`φ_1(CO_2) = 0.04%`

`φ_2(CO_2) = 0.1%`

`t = ?`

Решение

Рассчитаем количество углекислого газа, который должен выдохнуть человек, для достижения верхней планки безопасного порога `CO_2`.

`V(CO_2) = ((φ_2(CO_2) - φ_1(CO_2))*V("комнаты"))/100 = ((0.1 - 0.04)*5*10^4)/100 = 30 л`

`t = (V(CO_2))/(ν_"выд."(CO_2)) = 30/20 = 1.5 ч`

Ответ: `t = 1.5 ч`.

7. Используя информационные ресурсы, установите, сколько сернистого газа попадает в атмосферу из природных источников, а сколько – в результате деятельности человека.

Ответ:

В результате вулканической деятельности ежегодно в атмосферу Земли попадает примерно 20-25 млн т. сернистого газа, а в результате деятельности человека – 60-70 млн т. двуокиси серы.

8. Какова основная область применения серной кислоты? На что расходуется наибольшее её количество?

Ответ:

Химическая промышленность – основная область применения серной кислоты. Наибольшее количество серной кислоты расходуется на производство минеральных удобрений.

9. Азот входит в состав атмосферы не только Земли, но и ещё нескольких планет Солнечной системы, и даже одного из их спутников. Назовите их.

Ответ:

Азот входит в состав атмосферы Урана (планета), Плутона (карликовая планета), Нептуна (планета), Титана (спутник Сатурна) и Тритона (спутник Нептуна).

10. Аммиак – второе после серной кислоты по объёму промышленного производства вещество: ежегодно в мире получают 140 млн т аммиака. На какие цели расходуется такое огромное количество аммиака?

Ответ:

В основном аммиак используется для производства азотных удобрений, взрывчатых веществ и азотной кислоты.

§39

Глава 4. Металлы

§39. Общие свойства элементов-металлов

1. Какими общими свойствами характеризуются элементы-металлы?

Ответ:

Атомы металлов на внешнем уровне содержат не более четырёх электронов, как правило, от одного до трёх. Металлы в химических реакциях являются восстановителями, они имеют низкую электроотрицательность – меньше 1,8.

2. Напишите электронные конфигурации атомов бериллия и магния.

Ответ:

4Be 1s2 2s2

12Mg 1s2 2s2 2p6 3s2

3. Сопоставьте строение атома и свойства химических элементов – металлов (натрий, алюминий) и неметаллов (фтор, сера).

Ответ:

Na +11 )2)8)1 
Al +13 )2)8)3 

Натрий обладает более сильными восстановительными свойствами, чем алюминий, это связано с тем, что у натрия на внешнем энергетическом уровне расположен 1ē, а у алюминия - 3ē, т. е. для получения устойчивой электронной конфигурации натрию проще отдать 1ē, чем алюминию отдать 3ē.

 F +9 )2)7 
+16 )2)8)6 

Фтор обладает более сильными окислительными свойствами, чем сера, это связано с тем, что у фтора на внешнем энергетическом уровне расположено 7ē, а у серы - 6ē, т. е. для получения устойчивой электронной конфигурации фтору проще принять 1ē, чем сере принять 2ē.

4. Почему щелочные металлы и галогены в природе не встречаются в форме простых веществ?

Ответ:

Щелочные металлы и галогены обладают большой химической активностью, поэтому в природе они не встречаются в форме простых веществ.

5. Сурьма Sb по некоторым свойствам (металлический блеск) близка металлам. Согласуется ли это с её положением в Периодической системе?

Ответ:

Сурьма расположена в VА группе между мышьяком и висмутом. В группе металлические свойства усиливаются сверху вниз. Мышьяк является неметаллом, сурьма обладает свойствами металлов и неметаллов, висмут является металлом, что согласуется с положением сурьмы в Периодической системе.

6. Рассчитайте массовые доли металлов в медном блеске и боксите.

Ответ:

Дано:

`Cu_2S`

`AlOOH`

`ω(Cu) = ?`

`ω(Al) = ?`

Решение

`ω(Cu) = (100*n(Cu)*M(Cu))/(M(Cu_2S)) = (100*2*64)/160 = 80%`

`ω(Al) = (100*n(Al)*M(Al))/(M(AlOOH)) = (100*1*27)/60 = 45%`

Ответ: `ω(Cu) = 80%`, `ω(Al) = 45%`.

7. Первые 83 элемента Периодической системы Д. И. Менделеева устойчивы, т. е. имеют нерадиоактивные изотопы. Сколько из этих элементов относится к металлам?

Ответ:

Среди первых 83 элементов в Периодической системе 63 элемента относится к металлам.

Всего к неметаллам относятся 22 химических элемента, два из них имеют порядковые номера больше 83.

§40

§40. Простые вещества – металлы

1. Перечислите общие свойства металлов. С чем они связаны?

Ответ:

Общие свойства металлов: металлический блеск, пластичность, высокая электро- и теплопроводность, ковкость. Эти свойства связаны с тем, что металлы в твёрдом состоянии имеют металлическую кристаллическую решётку.

2. Приведите примеры металлов с высокими и низкими значениями тепло- и электропроводности.

Ответ:

Примеры металлов с высокими значениями тепло- и электропроводности: серебро, медь, алюминий, золото.

Примеры металлов с низкими значениями тепло- и электропроводности: железо, олово, свинец.

3. Какие металлы называют тугоплавкими; легкоплавкими; лёгкими; тяжёлыми? Приведите примеры.

Ответ:

Металлы с температурой плавления выше 1000°C называют тугоплавкими. К ним относятся, например, медь, железо, вольфрам.

Металлы с температурой плавления ниже 1000°C называют легкоплавкими. К ним относятся, например, галлий, олово, цинк.

Металлы с плотностью ниже 5 г/см3 называют лёгкими. К ним относятся, например, литий, магний, алюминий.

Металлы с плотностью выше 5 г/см3 называют тяжёлыми. К ним относятся, например, железо, свинец, золото.

4. Воспользовавшись рисунками 97, 98, приведите по одному примеру: а) твёрдого тугоплавкого металла; б) мягкого тугоплавкого металла; в) мягкого легкоплавкого металла.

Ответ:

а) вольфрам.

б) медь.

в) натрий.

5. Какая масса 10%-го раствора сульфата меди (II) потребуется для полного растворения железного гвоздя массой 1,12 г?

Ответ:

Дано:

`ω(CuSO_4) = 10%`

`m(Fe) = 1.12 г`

`m("р-ра") = ?`

Решение

`Fe + CuSO_4 ⟶ Cu + FeSO_4`

`n(Fe) = (m(Fe))/(M(Fe)) = 1.12/56 = 0.02" моль"`

`n(CuSO_4) = n(Fe) = 0.02" моль"`

`m(CuSO_4) = n(CuSO_4)*M(CuSO_4) = 0.02*160 = 3.2 г`

`m("р-ра") = (100*m(CuSO_4))/(ω(CuSO_4)) = (100*3.2)/10 = 32 г`

Ответ: `m("р-ра") = 32 г`.

6. Олово выше 13°C существует в β-модификации (белое олово), имеющей металлическую решётку, а при более низкой температуре – в виде α-олова (серое олово) со структурой алмаза. Какая из двух модификаций: а) пластична и обладает высокой электропроводностью; б) является хрупкой и проявляет свойства полупроводника?

Ответ:

а) β-олово (белое олово), так как вещества с металлической кристаллической решёткой обладают пластичностью и высокой электропроводностью.

б) α-олово (серое олово), так как вещества с атомной кристаллической решёткой являются хрупкими.

7. Как можно осуществить следующие превращения: Zn ⟶ Fe ⟶ Cu ⟶ Ag? Напишите уравнения реакций.

Ответ:

Zn + FeCl2 ⟶ Fe + ZnCl2

Fe + CuSO4 ⟶ Cu + FeSO4

Cu + 2AgNO3 ⟶ 2Ag + Cu(NO3)2

8. Воспользовавшись рисунком 101, объясните, почему многие металлы пластичны, а ионные кристаллы – хрупки.

Ответ:

При обычных условиях в веществах с ионной кристаллической решеткой в узлах напротив отрицательно заряженных ионов располагаются положительно заряженные ионы. При надавливании на ионные кристаллы происходит сдвиг кристаллической решетки, и в момент когда напротив ионов располагаются одноименные заряженные ионы происходит разрушение кристалла, за счёт сил электростатического отталкивания. Поэтому ионные кристаллы – хрупки.

В металлах в узлах кристаллической решетки расположены ионы металла, в узлах напротив расположены такие же ионы металла. Ионы металла скреплены между собой с помощью обобществлённых электронов. При надавливании на металлы происходит сдвиг кристаллической решетки, при этом в узлах напротив всё также будут находится ионы металла, скреплённых между собой с помощью обобществлённых электронов. Поэтому многие металлы пластичны.

9. Охарактеризуйте общие химические свойства металлов, заполнив последнюю графу таблицы 12.

Таблица 12

Реакция Условия протекания реакции Пример
1. Взаимодействие с водой. В реакцию вступают лишь наиболее активные металлы.  
2. Взаимодействие с кислотами. Металл должен быть расположен в ряду активности левее водорода.  
3. Взаимодействие с солями. Более активный металл вытесняет менее активный.  
4. Взаимодействие с неметаллами. Чем активнее металл и неметалл, тем энергичнее протекает реакция.  
5. Взаимодействие с оксидами. Более активный металл вытесняет менее активный.  

Ответ:

Реакция Условия протекания реакции Пример
1. Взаимодействие с водой. В реакцию вступают лишь наиболее активные металлы. 2Na + 2H2O ⟶ 2NaOH + H2
2. Взаимодействие с кислотами. Металл должен быть расположен в ряду активности левее водорода. Zn + 2HCl ⟶ ZnCl2 + H2
3. Взаимодействие с солями. Более активный металл вытесняет менее активный. Fe + CuSO4 ⟶ Cu + FeSO4
4. Взаимодействие с неметаллами. Чем активнее металл и неметалл, тем энергичнее протекает реакция. Zn + S ⟶ ZnS
5. Взаимодействие с оксидами. Более активный металл вытесняет менее активный. Al + Fe2O3 ⟶ Fe + Al2O3

§41

§41. Получение металлов. Применение металлов в технике

1. В виде каких веществ встречаются металлы в природе? Что называют рудами; самородными металлами?

Ответ:

Металлы в природе встречаются в виде оксидов, солей, простых веществ.

Минералы и горные породы, содержащие соединения металлов и пригодные для получения металлов промышленным способом, называют рудами.

Металлы, находящиеся в природе в свободном состоянии, называют самородными.

2. Можно ли использовать щелочные металлы для вытеснения менее активных металлов из водных растворов их солей? Почему?

Ответ:

Нельзя использовать щелочные металлы для вытеснения менее активных металлов из водных растворов их солей, потому что активные металлы вступают в химическую реакцию с водой.

3. Какие восстановители используют в металлургии? Приведите примеры.

Ответ:

В качестве восстановителей в металлургии используют кокс, угарный газ, водород, магний, алюминий.

Например, для восстановления железа из красного железняка используют кокс.
2Fe2O3 + 3C   t ⟶ 4Fe + 3CO2

Водород используют для получения дорогих металлов, например вольфрама.
WO3 + 3H2   t ⟶ W + 3H2O

В некоторых случаях в качестве восстановителя используют другой, более активный металл, например алюминий.
2Al + Cr2O3   t ⟶ Al2O3 + 2Cr

4. ==

Ответ:

 ==

5. Железо получают восстановлением красного железняка Fe2O3 коксом (C) и угарным газом. Напишите уравнения реакций. Почему в промышленности выгоднее использовать кокс?

Ответ:

Fe2O3 + 3C ⟶ 2Fe + 3CO

Fe2O3 + 3CO ⟶ 2Fe + 3CO2

В промышленности выгоднее использовать кокс, потому что он занимает меньший объем, и потому что при восстановлении металлов с помощью кокса образуется угарный газ, который также учувствует в восстановлении металла.

6. Цинк получают в промышленности восстановлением углём оксида, полученного при прокаливании на воздухе сфалерита ZnS. Какую массу цинка можно получить из 106,7 кг сфалерита, содержащего 10% пустой породы?

Ответ:

Дано:

`m("сфалерита") = 106.7" кг"`

`ω("примесей") = 10%`

`m(Zn) = ?`

Решение

`2ZnS + 3O_2 overset(t)(⟶) 2ZnO + 2SO_2"↑"`

`2ZnO + C overset(t)(⟶) 2Zn + CO_2"↑"`

`ω(ZnS) = 100 - ω("примесей") = 100 - 10 = 90%`

`m(ZnS) = (ω(ZnS)*m("сфалерита"))/100 = (90*106.7)/100 = 96.03" кг"`

`n(ZnS) = (m(ZnS))/(M(ZnS)) = 96.03/97 = 0.99" кмоль"`

`n(Zn) = n(ZnS) = 0.99" кмоль"`

`m(Zn) = n(Zn)*M(Zn) = 0.99*65 = 64.35" кг"`

Ответ: `m(Zn) = 64.35" кг"`.

7. Напишите уравнения электролиза расплавов солей: хлорида калия, бромида кальция.

Ответ:

2KCl(распл.)   эл. ток ⟶ 2K + Cl2

CaBr2(распл.)   эл. ток ⟶ Ca + Br2

8. Производство натрия основано на электролизе расплава его хлорида. Напишите уравнение реакции. Какой объём хлора (в пересчёте на н. у.) выделился на аноде, если известно, что масса образовавшегося металла равна 69 кг?

Ответ:

Дано:

`m(Na) = 69" кг"`

`V(Cl_2) = ?`

Решение

`2NaCl_("распл.") overset("эл. ток")(⟶) 2Na + Cl_2"↑"`

`n(Na) = (m(Na))/(M(Na)) = 69/23 = 3" кмоль"`

`n(Cl_2) = (n(Na))/2 = 3/2 = 1.5" кмоль"`

`V(Cl_2) = n(Cl_2)*V_m = 1.5*22.4 = 33.6 м^3`

Ответ: `V(Cl_2) = 33.6 м^3`.

§42

§42. Щелочные металлы

1. Какие общие свойства характерны для щелочных металлов? С чем это связано?

Ответ:

Все щелочные металлы легко вступают в химические реакции. Они легко окисляются на воздухе, взаимодействуют с водой при комнатной температуре с выделением водорода.

Высокая химическая активность щелочных металлов связана с тем, что на их внешнем уровне находится один валентный электрон, который они легко отдают, для получения устойчивой электронной конфигурации благородного газа.

2. Что общего и в чём отличие между электронными конфигурациями: а) натрия и калия; б) натрия и магния?

Ответ:

а)

Na +11 )2)8)1 
+19 )2)8)8)1  

Общее. Натрий и калий имеют по 1ē на внешнем энергетическом уровне.

Отличие. Натрий имеет три энергетических уровня, а калий – четыре.

б)

Na +11 )2)8)1 
Mg +12 )2)8)2 

Общее. Натрий и магний имеют три энергетических уровня.

Отличие. Натрий имеет 1ē на внешнем энергетическом уровне, а магний – 2ē.

3. При хранении на воздухе гидроксид натрия быстро превращается в карбонат натрия. Напишите уравнение реакции.

Ответ:

2NaOH + CO2 ⟶ Na2CO3 + H2O

4. Предложите три различных способа получения хлорида натрия.

Ответ:

2Na + Cl2 ⟶ 2NaCl

NaOH + HCl ⟶ NaCl + H2O

Na2CO3 + 2HCl ⟶ 2NaCl + H2O + CO2

5. В склянке без этикетки находится белое вещество, растворимое в воде. Оно окрашивает пламя в жёлтый цвет, а с нитратом серебра даёт белый творожистый осадок, нерастворимый в кислотах. Какое вещество находится в склянке?

Ответ:

Ионы натрия окрашивают пламя в желтый цвет. Хлорид-ионы с нитратом серебра дают белый творожистый осадок, нерастворимый в кислотах. Соответственно этим веществом является хлорид натрия.
NaCl + AgNO3 ⟶ NaNO3 + AgCl↓

6. Напишите уравнения реакций, отвечающие следующей схеме превращений:
Na ⟶ NaOH ⟶ Na2CO3 ⟶ NaCl ⟶ NaNO3.

Ответ:

2Na + 2H2O ⟶ 2NaOH + H2

2NaOH + CO2 ⟶ Na2CO3 + H2O

Na2CO3 + 2HCl ⟶ 2NaCl + H2O + CO2

NaCl + AgNO3 ⟶ NaNO3 + AgCl↓

7. При растворении 2 г соды в воде получили 400 г раствора. Определите массовую долю соды в растворе.

Ответ:

Дано:

`m("соды") = 2 г`

`m("р-ра") = 400 г`

`ω("соды") = ?`

Решение

`ω("соды") = (100*m("соды"))/(m("р-ра")) = (100*2)/400 = 0.5%`

Ответ: `ω("соды") = 0.5%`.

8. В промышленности натрий получают электролизом расплава поваренной соли. Какую массу соли необходимо взять для получения 2,3 кг натрия?

Ответ:

Дано:

`m(Na) = 2.3" кг"`

`m(NaCl) = ?`

Решение

`2NaCl overset("эл. ток")(" "⟶" ") 2Na + Cl_2"↑"`

`n(Na) = (m(Na))/(M(Na)) = 2.3/23 = 0.1" кмоль"`

`n(NaCl) = n(Na) = 0.1" кмоль"`

`m(NaCl) = n(NaCl)*M(NaCl) = 0.1*58.5 = 5.85" кг"`

Ответ: `m(NaCl) = 5.85" кг"`.

9. Какой объём водорода (н. у.) можно выделить из воды при помощи 0,69 г натрия?

Ответ:

Дано:

`m(Na) = 0.69 г`

`V(H_2) = ?`

Решение

`2Na + 2H_2O ⟶ 2NaOH + H_2"↑"`

`n(Na) = (m(Na))/(M(Na)) = 0.69/23 = 0.03" моль"`

`n(H_2) = (n(Na))/2 = 0.03/2 = 0.015" моль"`

`V(H_2) = n(H_2)*V_m = 0.015*22.4 = 0.336 л`

Ответ: `V(H_2) = 0.336 л`.

10. Напишите по одному уравнению реакций, в которых: а) атомы натрия превращаются в ионы натрия; б) ионы натрия превращаются в атомы натрия.

Ответ:

а) 2Na + 2H2O ⟶ 2NaOH + H2

б) 2NaCl(распл.)   эл. ток ⟶ 2Na + Cl2

Лабораторный опыт 12. Свойства гидроксида натрия.

1. Растворите в воде одну гранулу гидроксида натрия. На ощупь определите, как изменяется температура нижней части пробирки.

2. Приготовленный раствор разделите на две примерно равные части. К одной из них добавьте каплю раствора фенолфталеина. Что наблюдаете? Затем прилейте к раствору соляную кислоту до исчезновения окраски. Окуните в раствор конец стальной проволоки, а затем внесите её в бесцветное пламя спиртовки. Что наблюдаете?

3. Ко второй части раствора добавьте раствор сульфата меди (II). Что происходит?

4. Напишите уравнения реакций гидроксида натрия с соляной кислотой и гидроксидом меди (II) в полном и сокращённом ионном виде.

Ответ:

1. При растворении гранулы гидроксида натрия температура в нижней части пробирки увеличивается.

2. Фенолфталеин в растворе гидроксида натрия приобретает малиновый цвет.

При прилевании соляной кислоты к раствору гидроксида натрия протекает реакция:
NaOH + HCl ⟶ NaCl + H2O
Na+ + OH- + H+ + Cl- ⟶ Na+ + Cl- + H2O
H+ + OH- ⟶ H2O

Стальная проволока, смоченная в образовавшемся растворе, окрашивает пламя спиртовки в желтый цвет.

3. При добавлении раствора сульфата меди (II) к раствору гидроксида натрия наблюдается выпадение осадка голубого цвета:
2NaOH + CuSO4 ⟶ Na2SO4 + Cu(OH)2
2Na+ + 2OH- + Cu2+ + SO42- ⟶ 2Na+ + SO42- + Cu(OH)2
Cu2+ + 2OH- ⟶ Cu(OH)2

4. Реакции написаны в пункте 2 и 3.

§43

§43. Кальций

1. Опишите физические и химические свойства кальция.

Ответ:

Физические свойства. Кальций представляет собой серебристо-серый, похожий на никель металл, по твёрдости почти равный меди. Пламя горящего кальция кирпично-красного цвета.

Химические свойства. Кальций проявляет сильные восстановительные свойства.

При нагревании на воздухе или в кислороде кальций сгорает:
2Ca + O2 ⟶ 2CaO

Кальций реагирует с водой, но реакция протекает более спокойно, чем с натрием:
Ca + 2H2O ⟶ Ca(OH)2 + H2

Кальций реагирует с другими неметаллами, например с серой:
Ca + S ⟶ CaS

2. В двух запаянных пробирках без этикеток находятся кальций и натрий. Как, не разбивая пробирок, определить, в какой из них находится какой металл?

Ответ:

Необходимо опустить пробирки без этикеток в кипящую воду. В пробирке где металл расплавится – натрий, в другой – кальций.

3. Почему при взаимодействии кальция с водой раствор мутнеет, а скорость реакции замедляется?

Ответ:

При взаимодействии кальция с водой образуется гидроксид кальция, который является малорастворимым в воде веществом, поэтому раствор мутнеет. Ввиду малой растворимости гидроксида кальция, он накапливается на поверхности кальция, поэтому скорость реакции замедляется.

4. Известковая вода в присутствии углекислого газа сначала мутнеет, а при дальнейшем пропускании углекислого газа выпавший осадок растворяется. Напишите уравнения реакций.

Ответ:

Ca(OH)2 + CO2 ⟶ CaCO3↓ + H2O

CaCO3 + H2O + CO2 ⟶ Ca(HCO3)2

5. Почему строители называют негашёную известь "кипелкой"

Ответ:

При добавлении воды к негашёной извести выделяется большое количество теплоты, которое доводит воду до кипения, поэтому строители называют негашёную известь "кипелкой".

6. Неизвестная соль нерастворима в воде, но реагирует с соляной кислотой с образованием газа, вызывающего помутнение известковой воды. Образовавшийся раствор окрашивает пламя горелки в кирпично-красный цвет. Назовите неизвестную соль (задача может иметь два решения).

Ответ:

Пламя горелки окрашивается в кирпично-красный цвет в присутствии ионов кальция.

Этой солью может являться карбонат кальция:
CaCO3 + 2HCl ⟶ CaCl2 + H2O + CO2
Ca(OH)2 + CO2 ⟶ CaCO3↓ + H2O

7. На каком свойстве обезвоженного гипса основано его использование при наложении гипсовых повязок?

Ответ:

Использование обезвоженного гипса основано на быстром затвердевании в присутствии воды.

8. Определите массу осадка, который образуется при действии на 200 г 5%-го раствора хлорида кальция избытком раствора карбоната натрия.

Ответ:

Дано:

`m("р-ра") = 200 г`

`ω(CaCl_2) = 5%`

`m(CaCO_3) = ?`

Решение

`CaCl_2 + Na_2CO_3 ⟶ 2NaCl + CaCO_3"↓"`

`m(CaCl_2) = (ω(CaCl_2)*m("р-ра"))/100 = (5*200)/100 = 10 г`

`n(CaCl_2) = (m(CaCl_2))/(M(CaCl_2)) = 10/111 = 0.09" моль"`

`n(CaCO_3) = n(CaCl_2) = 0.09" моль"`

`m(CaCO_3) = n(CaCO_3)*M(CaCO_3) = 0.09*100 = 9 г`

Ответ: `m(CaCO_3) = 9 г`.

9. Как различить следующие вещества: а) соду (Na2CO3) и мел (CaCO3); б) мел и гипс (CaSO4)?

Ответ:

а) Соду и мел можно отличить с помощью воды. Сода растворяется в воде, а мел – нет.

б) Мел и гипс можно отличить с помощью раствора соляной кислоты. При действии соляной кислоты на мел, будет выделяться газ, а при действии на гипс – нет.

10. Напишите полные уравнения реакций, соответствующие следующим ионным уравнениям:
а) Ca2+ + CO2 = CaCO3↓;
б) CaCO3 + 2H+ = Ca2+ + CO2↑ + H2O.

Ответ:

а) Ca(OH)2 + CO2 ⟶ CaCO3↓ + H2O

б) CaCO3 + 2HCl ⟶ CaCl2 + H2O + CO2

11. Составьте уравнения химических реакций, позволяющих осуществить следующие превращения: CaCO3 ⟶ CaO ⟶ Ca(OH)2 ⟶ CaCO3 ⟶ Ca(NO3)2.

Ответ:

CaCO3   t ⟶ CaO + CO2

CaO + H2O ⟶ Ca(OH)2

Ca(OH)2 + CO2 ⟶ CaCO3↓ + H2O

CaCO3 + 2HNO3 ⟶ Ca(NO3)2 + H2O + CO2

§44

§44. Алюминий

1. Дайте характеристику элемента алюминия, исходя из его положения в Периодической системе.

Ответ:

Алюминий (Al) располагается во 3 периоде, в III группе, главной подгруппе, имеет порядковый номер 13.

A (массовое число) = 27
P (число протонов) = 13
N (число нейтронов) = A - Z = 27 - 13 = 14
ē (число электронов) = 13

13Al 1s2 2s2 2p6 3s2 3p1

Алюминий – p-элемент, металл.

2. Почему в XIX в. в России алюминий называли глинием?

Ответ:

В XIX в. в России алюминий называли глинием, так как главной составной частью глины является глинозём Al2O3.

3. Напишите уравнение реакции алюминия с расплавленным гидроксидом натрия, предполагая, что при этом образуется алюминат натрия состава Na3AlO3. Назовите окислитель и восстановитель.

Ответ:

2Al + 6NaOH ⟶ 2Na3AlO3 + 3H2

Al0 - 3ē ⟶ Al+3 |3| 6 |2    восстановитель (окисление)
2H+ + 2ē ⟶ H20 |2| |3    окислитель (восстановление)

4. Как доказать наличие на поверхности алюминия оксидной плёнки?

Ответ:

Алюминий покрытый оксидной плёнкой не реагирует с водой. Если очистить алюминий от оксидной плёнки, он энергично реагирует с водой:
2Al + H2O ⟶ 2Al(OH)3 + 3H2

5. Воспользовавшись рисунком 118, расскажите о применении алюминия.

Ответ:

Благодаря малой плотности, прочности, коррозионной стойкости алюминий применяется в автомобилестроении, авиастроении, кораблестроении.

Благодаря малой плотности, прочности, коррозионной стойкости и высокой электропроводности алюминий используется в качестве проводника высоковольтных ЛЭП.

Благодаря коррозионной стойкости, высокой теплопроводности, образованию неядовитых соединений алюминий применяется для изготовления посуды, пищевых упаковок, фольги.

6. Почему нельзя пользоваться алюминиевой посудой для хранения щелочных растворов?

Ответ:

Алюминиевой посудой нельзя пользоваться для хранения щелочных растворов, потому что алюминий вступает в химическую реакцию с щелочами.
2Al + 2NaOH + 6H2O ⟶ 2Na[Al(OH)4] + 3H2

7. С помощью каких реакций можно отличить друг от друга хлориды натрия, кальция и алюминия? Напишите уравнения реакций.

Ответ:

Отличить друг от друга хлориды натрия, кальция и алюминия можно с помощью избытка раствора щелочи.

При добавлении щелочи к раствору хлорида алюминия в начале образуется осадок, а при дальнейшем добавлении раствора щелочи наблюдается растворение осадка:
AlCl3 + 3NaOH ⟶ 3NaCl + Al(OH)3
Al(OH)3 + NaOH ⟶ Na[Al(OH)4]

При добавлении щелочи к раствору хлорида кальция образуется осадок, а при дальнейшем добавлении раствора щелочи растворение осадка не наблюдается:
CaCl2 + 2NaOH ⟶ 2NaCl + Ca(OH)2

При добавлении щелочи к раствору хлорида натрия изменений не наблюдается.

8. При обработке 10 г сплава меди с алюминием соляной кислотой выделилось 6,72 л водорода (н. у.). Определите состав сплава в процентах по массе.

Ответ:

Дано:

`m("сплава") = 10 г`

`V(H_2) = 6.72 л`

`ω(Al) = ?`

`ω(Cu) = ?`

Решение

`2Al + 6HCl ⟶ 2AlCl_3 + 3H_2"↑"`

`n(H_2) = (V(H_2))/V_m = 6.72/22.4 = 0.3" моль"`

`n(Al) = (2*n(H_2))/3 = (2*0.3)/3 = 0.2" моль"`

`m(Al) = n(Al)*M(Al) = 0.2*27 = 5.4 г`

`ω(Al) = (100*m(Al))/(m("сплава")) = (100*5.4)/10 = 54%`

`ω(Cu) = 100 - ω(Al) = 100 - 54 = 46%`

Ответ: `ω(Al) = 54%`, `ω(Cu) = 46%`.

9. Растворение алюминия в разбавленной азотной кислоте происходит без выделения газов и приводит к образованию двух нитратов. Напишите уравнение реакции, составьте электронный баланс, укажите окислитель и восстановитель.

Ответ:

8Al + 30HNO3(разб.) ⟶ 8Al(NO3)3 + 3NH4NO3 + 9H2O

Al0 - 3ē ⟶ Al+3 |3| 24 |8    восстановитель (окисление)
N+5 + 8ē ⟶ N-3 |8| |3    окислитель (восстановление)

10. Напишите уравнения реакций алюминия с оксидами металлов: Fe3O4, CuO, CaO.

Ответ:

3Fe3O4 + 8Al   t ⟶ 4Al2O3 + 9Fe

3CuO + 2Al   t ⟶ Al2O3 + 3Cu

CaO + 2Al   t ⟶ Al2O3 + 3Ca

§45

§45. Железо

1. В каком виде в земной коре встречаются соединения железа? Назовите 3-4 минерала.

Ответ:

В земной коре железо встречается в виде соединений с кислородом или серой.

Минералы железа: красный железняк Fe2O3, магнитный железняк Fe3O4, пирит FeS2.

2. Что называют сталью; чугуном?

Ответ:

Сплав железа с углеродом, содержащий 0,3-2% углерода, называют сталью.

Сплав железа с углеродом, содержащий более 2% углерода, называют чугуном.

3. Как по свойствам отличаются чугун и сталь; отпущенная и закалённая сталь?

Ответ:

В отличие от стали, чугун твёрд, но хрупок, легко ломается и разбивается.

Закалённая сталь, в отличие от отпущенной стали, твёрдая, прочная, и плохо подвергается механической обработке.

4. Как из железа получить хлорид железа (II); хлорид железа (III)? Напишите уравнения реакций.

Ответ:

Хлорид железа (II) из железа можно получить с помощью соляной кислоты:
Fe + 2HCl ⟶ FeCl2 + H2

Хлорид железа (II) из железа можно получить с помощью хлора:
2Fe + 3Cl2 ⟶ 2FeCl3

5. Предложите химический способ очистки железа от ржавчины.

Ответ:

Железо от ржавчины можно очистить с помощью раствора соляной кислоты с ингибитором.
Fe(OH)3 + 3HCl ⟶ FeCl3 + 3H2O

6. Как отличить соли железа (II) от солей железа (III)?

Ответ:

Соли железа (II) от солей железа (III) можно отличить с помощью роданида калия KCNS. При действии роданида калия на соли железа (III) образуется ярко-красное окрашивание.

7. Предложите три способа получения железа из магнитного железняка.

Ответ:

Fe3O4 + 2C   t ⟶ 3Fe + 2CO2

Fe3O4 + 4H2   t ⟶ 3Fe + 4H2O

3Fe3O4 + 8Al   t ⟶ 9Fe + 4Al2O3

8. При растворении стальной проволоки массой 57 г в соляной кислоте выделился 1 моль водорода. Определите массовую долю углерода в стали.

Ответ:

Дано:

`m("стали") = 57 г`

`n(H_2) = 1" моль"`

`ω(C) = ?`

Решение

`Fe + 2HCl ⟶ FeCl_2 + H_2"↑"`

`n(Fe) = n(H_2) = 1" моль"`

`m(Fe) = n(Fe)*M(Fe) = 1*56 = 56 г`

`m(C) = m("стали") - m(Fe) = 57 - 56 = 1 г`

`ω(C) = (100*m(C))/(m("стали")) = (100*1)/57 = 1.75%`

Ответ: `ω(C) = 1.75%`.

9. Напишите уравнения реакций, соответствующие следующим схемам:
а) Fe ⟶ X ⟶ Fe(OH)2;
б) Fe ⟶ Y ⟶ Fe(OH)3.
Определите неизвестные вещества.

Ответ:

а)

Fe + 2HCl ⟶ FeCl2 + H2

FeCl2 + 2NaOH ⟶ 2NaCl + Fe(OH)2

б)

2Fe + 3Cl2 ⟶ 2FeCl3

FeCl3 + 3NaOH ⟶ 3NaCl + Fe(OH)3

10. В составе каких соединений железо присутствует в организме человека? Сколько примерно граммов железа в вашем организме?

Ответ:

Железо в организме человека входит в состав таких белков как гемоглобин, ферритин, миоглобин, трансферрин. В организме взрослого человека содержится около 3-4 граммов железа.

Творческие задания 3

Творческие задания 3

1. Объясните, почему калий не может вытеснить натрий из водного раствора поваренной соли. Попытайтесь описать результаты такого опыта.

Ответ:

Калий вступает в реакцию с водой, поэтому он не может вытеснить натрий из водного раствора поваренной соли.

2. ==

Ответ:

 ==

3. Какие металлы называют благородными? Почему их так называют? Опишите свойства благородных металлов.

Ответ:

Золото, серебро, платина, рутений, родий, палладий, осмий, иридий – благородные металлы.

Их так называют, потому что они не подвержены коррозии, окислению, не реагируют с соляной кислотой.

§46

Глава 5. Обобщение сведений об элементах и неорганических веществах

§46. Закономерности изменения свойств элементов и простых веществ

1. Какую высшую степень окисления проявляет элемент кремний? Ответ поясните.

Ответ:

Высшая степень окисления кремния +4, потому что он имеет 4 валентных электрона.

2. Назовите наиболее типичный металл 3-го периода, наиболее типичный неметалл 2-го периода.

Ответ:

Литий – наиболее типичный металл 2-го периода.

Натрий – наиболее типичный металл 3-го периода.

3. Найдите ошибки в некоторых фразах и прокомментируйте их: "молекула воды имеет температуру плавления 0°C", "элемент уголь входит в состав мела", "атом меди тяжелее атома железа", "атомы меди имеют оранжево-красную окраску", "хлоридионы являются сильными окислителями", "для атомов натрия характерны восстановительные свойства", "атом азота имеет меньший радиус, чем атом кислорода".

Ответ:

"Молекула воды имеет температуру плавления 0°C" – молекулы не могут плавиться, температуру плавления имеет вещество состоящее из молекула воды.

"Элемент уголь входит в состав мела" – элемента под названием уголь не существует, уголь это вещество состоящее из химического элемента – углерода. Мел это вещество в состав которого входит углерод.

"Атомы меди имеют оранжево-красную окраску" – атомы веществ не имеют окраски, окраску имеет вещество состоящее из атомов/молекул.

"Хлорид-ионы являются сильными окислителями" – хлорид-ионы не могу вообще проявлять окислительные свойства, так как в хлорид-ионах хлор находится в своей низшей степени окисления.

"Атом азота имеет меньший радиус, чем атом кислорода" – у атома кислорода меньше радиус, чем у атома азота, потому что радиусы атомов в периоде слева направо уменьшаются.

4. Напишите уравнения взаимодействия алюминия с простыми веществами – неметаллами 2-го периода, зная, что в полученных соединениях неметалл находится в низшей степени окисления, а алюминий – в высшей.

Ответ:

Al + B ⟶ AlB

4Al + 3C ⟶ Al4C3

2Al + N2 ⟶ 2AlN

4Al + 3O2 ⟶ 2Al2O3

2Al + 3F2 ⟶ 2AlF3

5. Какие из элементов 3-го периода образуют простые вещества с металлической, атомной, молекулярной кристаллической решёткой?

Ответ:

Вещества с металлической кристаллической решёткой образуют элементы: натрий, магний, алюминий.

Вещества с атомной кристаллической решёткой образуют элементы: кремний, фосфор.

Вещества с молекулярной кристаллической решёткой образуют элементы: сера, хлор, аргон.

6. Постройте схемы, аналогичные схемам 6 и 7, для элементов и простых веществ 3-го периода. Объясните наблюдаемые закономерности.

Ответ:

Изменение свойств элементов 3-ого периода

Элемент Na Mg Al Si P S Cl Ar
Порядковый номер 11 12 13 14 15 16 17 18
Радиус атома, нм 0,19 0,16 0,143 0,132 0,128 0,127 0,099 0,071
⟵Радиус атома возрастает—
Строение атома Na +11 )2)8)1    Mg +12 )2)8)2  Mg +13 )2)8)3  Si +14 )2)8)4  P +15 )2)8)5  S +16 )2)8)6  Cl +17 )2)8)7  Ar +18 )2)8)8
Число валентных электронов 1 2 3 4 5 6 7 8
Электроотрицательность 0,93 1,31 1,61 1,90 2,19 2,58 3,16
—Электроотрицательность возрастает⟶
Металлические свойства мет мет мет нем нем нем нем нем
⟵Металлические свойства возрастают—
Окислительные свойства —Окислительные свойства возрастают⟶
Восстановительные свойства ⟵Восстановительные свойства возрастают—
Высшая степень окисления +1 +2 +3 +4 +5 +6 +7
Низшая степень окисления 0 0 0 -4 -3 -2 -1

​Изменение свойств простых веществ 3-го периода

Простое вещество Na Mg Al Si P, P4 S8 Cl2 Ar
Металлические свойства мет мет мет нем нем нем нем нем
⟵Металлические свойства возрастают—
Тип кристаллической решетки мет мет мет ат ат, мол мол мол мол
Температура плавления, °C 98 650 660 1415 возг., 44 113 -101 -189
  —Окислительные свойства возрастают⟶ инертен
⟵Восстановительные свойства возрастают— инертен

§47

§47. Закономерности изменения свойств соединений элементов

1. Как изменяются кислотно-основные свойства высших оксидов при движении по периоду? Ответ проиллюстрируйте на примере элементов 3-го периода.

Ответ:

Кислотно-основные свойства высших оксидов при движении по периоду слева направо изменяются от основных к кислотным через амфотерные.

NaOH – основный оксид

Mg(OH)2 – основный оксид

Al(OH)3 – амфотерный оксид

SiO2 – кислотный оксид

P2O5 – кислотный оксид

SO3 – кислотный оксид

Cl2O7 – кислотный оксид

2. Почему водный раствор фтороводорода имеет кислотную реакцию среды, а водный раствор аммиака – щелочную?

Ответ:

Фтороводорода в воде диссоциирует с образованием катионов водорода, поэтому его раствор имеет кислотную реакцию среды.
HF ⇄ H+ + F-

Аммиак вступает во взаимодействие с водой с образованием слабого гидроксида аммония, который диссоциирует с образованием гидроксид-анионов, поэтому водный раствор аммиака имеет щелочную реакцию среды.
NH3 + H2O ⇄ NH4OH
NH4OH ⇄ NH4+ + OH-

3. Чем отличаются выражения "сильная кислота" и "кислота – сильный окислитель"? Ответ проиллюстрируйте примерами.

Ответ:

В сильной кислоте в реакциях учувствуют протоны водорода.

MgO + 2HCl ⟶ MgCl2 + H2O

Mg + H2SO4(разб.) ⟶ MgSO4 + H2

Mg0 - 2ē ⟶ Mg+2 |2| 2 |1    восстановитель
2H+ + 2ē ⟶ H20 |2| |1    окислитель

Кислота – сильный окислитель – в реакциях учувствует кислотный остаток в качестве окислителя.

MgO + 2HCl ⟶ MgCl2 + H2O

Cu0 - 2ē ⟶ Cu+2 |2| 2 |1    восстановитель
S+6 + 2ē ⟶ S+4 |2| |1    окислитель

4. Какие два летучих водородных соединения 2-го периода взаимодействуют друг с другом? Напишите уравнение реакции.

Ответ:

N2 + 3F2   эл. разряд ⟶ 2NF3

O2 + F2   эл. разряд ⟶ O2F2

5. Препарат, который вы видите на фотографии (рис. 123), был синтезирован в Московском университете в 1867 г. Запишите истинную и простейшую формулу соединения, если известно, что оно содержит 10,9% фосфора и 89,1% иода по массе, а его молярная масса равна 570 г/моль. Назовите вещество, определите тип связи в молекуле. Изобразите структурную формулу соединения, зная, что все связи в нём одинарные.

Ответ:

Дано:

`ω(P) = 10.9%`

`ω(I) = 89.1%`

`M(P_xI_y) = 570" г/моль"`

`P_xI_y - ?`

Решение

`(100*31*x)/570 = 10.9`

`x = 2`

`(100*127*y)/570 = 89.1`

`y = 4`

Формула вещества `P_2I_4`.

Проверка:

`M(P_2I_4) = 2*31 + 4*127 = 570" г/моль"`

Ответ: `P_2I_4`.

I      I
 \ P /
 /    \
I      I
Тетраиодид дифосфора

Типы связей в молекуле. Связь P–P ковалентная неполярная, связи P–I ковалентные полярные.

6. На рисунке 124 приведена микрофотография нитевидных кристаллов никеля, выращенных в пористом оксиде алюминия. Такие металлические нити обладают высокой прочностью – они не гнутся и не ломаются. Предложите химический способ очистки этих кристаллов от оксида алюминия.

Ответ:

Очистить кристаллы никеля от оксида алюминия можно с помощью раствора щелочи.
Al2O3 + 2NaOH + 3H2O ⟶ 2Na[Al(OH)4]

7. ==

Ответ:

 ==

8. В двух склянках без этикеток находятся растворы сульфата алюминия и гидроксида натрия. Как можно определить содержимое каждой склянки, не имея реактивов, а используя только одну пустую пробирку?

Ответ:

Прилить из одной пробирки несколько капель раствора в другую пробирку.

Если будет наблюдаться появление белого осадка и его растворение, то в приливаемой пробирке находится гидроксид натрия:
6NaOH + Al2(SO4)3 = 3Na2SO4 + 2Al(OH)3
NaOH + Al(OH)3 = Na[Al(OH)4]

Если будет наблюдаться появление белого осадка без его растворения, то в приливаемой пробирке находится сульфат алюминия:
Al2(SO4)3 + 6NaOH = 3Na2SO4 + 2Al(OH)3

9. Имеются три одинаковые колбы, одна из которых заполнена углекислым газом, вторая – азотом, третья – аргоном. Колба, заполненная азотом, на 16 г легче колбы с углекислым газом. Найдите массу газа в третьей колбе.

Ответ:

Дано:

`V(CO_2) = V(N_2) = V(Ar)`

`m(CO_2) - m(N_2) = 16 г`

`m(Ar) = ?`

Решение

`(V(CO_2))/V_m*M(CO_2) - (V(N_2))/V_m*M(N_2) = 16 г`

`(V(CO_2))/V_m*(M(CO_2) - M(N_2)) = 16 г`

`(V(CO_2))/22.4*(44 - 28) = 16`

`V(CO_2) = 22.4 л`

`n(Ar) = (V(Ar))/V_m = 22.4/22.4 = 1" моль"`

`m(Ar) = n(Ar)*M(Ar) = 1*40 = 40 г`

Ответ: `m(Ar) = 40 г`.

§48

Глава 6. Начальные сведения об органических соединениях

§48. Классификация и строение органических веществ

1. Какие вещества называют органическими? Приведите примеры.

Ответ:

Органическими веществами называют соединения углерода, за исключением оксидов углерода, угольной кислоты и её солей, карбидов и некоторых других веществ. Примеры органических веществ: метан, этан, пропан, этилен, ацетилен, этиловый спирт, глицерин, уксусная кислота.

2. Какими общими свойствами обладают органические соединения?

Ответ:

Почти все органические соединения имеют молекулярное строение. Для таких соединений характерны сравнительно низкие температуры плавления и кипения. В отличие от большинства неорганических соединений, органические вещества, как правило, горючи и при нагревании разлагаются. При полном сжигании органических веществ образуются углекислый газ и вода, а также азот, если он входил в их состав.

3. Объясните, почему органических соединений так много.

Ответ:

Многообразие органических молекул объясняется способностью атомов углерода образовывать одинарные, двойные и тройные связи, а также соединяться в цепи и кольца.

4. Какова валентность атомов углерода в органических веществах?

Ответ:

В органических веществах валентность атомов углерода равна 4.

5. Зная валентности углерода, кислорода и водорода, составьте структурную формулу какого-либо органического соединения, имеющего формулу C3H6O2.

Ответ:

        H    H   O
         |     |     ||
H — C - C - C — O — H
         |     |    
        H    H

6. Найдите формулу углеводорода, содержащего 82,76% углерода по массе, если его молярная масса равна 58 г/моль.

Ответ:

Дано:

`C_xH_y`

`ω(C) = 82.76%`

`M(C_xH_y) = 58" г/моль"`

`C_xH_y - ?`

Решение

`ω(C) = (100*x*M(C))/(M(C_xH_y))`

`(100*x*12)/58 = 82.76`

`x = (82.76*58)/(100*12) = 4`

`x*M(C) + y*M(H) = 58`

`12*x + y = 58`

`y = 58 - 12*x = 58 - 12*4 = 10`

Формула углеводорода `C_4H_10`.

Ответ: `C_4H_10`.

7. Сколько химических связей в молекуле пропана C3H8? Сколько из них связей C–C и C–H?

Ответ:

        H      H     H
         |       |       |
H — C — C — C — H
         |       |       |
        H      H     H

В молекуле пропана 10 химических связей, из них 3 C–C связи и 7 C–H связей.

8. Напишите структурные формулы двух изомеров состава: а) C2H4Cl2; б) C4H10.

Ответ:

а)

        H      H
         |       |  
H — C — C — H
         |       |  
        Cl    Cl  

        Cl     H
         |       |  
H — C — C — H
         |       |  
        Cl     H  

б)

        H      H     H    H
         |       |       |      |
H — C — C — C— C — H
         |       |       |      |
        H      H     H     H

        H      H     H
         |       |       |
H — C — C — C — H
         |       |       |
        H      C     H
                /|\
             H H H

Лабораторный опыт 13. Сравнение кислотно-основных свойств водородных соединений неметаллов.

В пробирки налейте по 1 мл раствора аммиака, воды, соляной кислоты и сероводородной воды В каждую пробирку добавьте по капле нейтрального раствора метилоранжа. Как изменилась окраска? Сделайте вывод о кислотно-основных свойствах каждого из веществ. Результаты перенесите в таблицу 13.

Таблица 13. Изменение окраски индикаторов в различных средах

Ответ:

Изменение окраски индикаторов в различных средах

Формула водородного соединения Окраска метилоранжа Кислотно-основные свойства Уравнение диссоциации
NH3 Желтая. Щелочные. NH3 + H2O ⇄ NH4OH
NH4OH ⇄ NH4+ + OH-
H2O Оранжевая. Нейтральные. H2O ⇄ H+ + OH-
HCl Розовая. Кислотные. HCl ⟶ H+ + Cl-
H2S Светло-розовая Кислотные. H2S ⇄ H+ + HS-

 

§49

§49. Углеводороды

1. Какие углеводороды называют алканами? Напишите молекулярную формулу алкана, содержащего восемь атомов углерода.

Ответ:

Алканами называют углеводороды с общей формулой CnH2n+2.

        H   H   H    H    H   H    H   H
         |    |     |     |      |     |      |     |
H —C—C—C—C—C—C—C—C— H
         |    |     |      |     |     |     |      |
        H   H   H    H    H   H    H   H

2. В молекуле алкана неразветвлённого строения содержатся 12 атомов водорода. Изобразите его структурную формулу.

Ответ:

         H   H   H   H   H
         |    |     |     |      |   
H —C—C—C—C—C— H
         |    |     |      |     | 
        H   H   H    H    H  

3. Рассчитайте массовую долю углерода в: а) метане; б) пропане. Как изменяется массовая доля углерода в алканах с увеличением числа атомов углерода в молекуле?

Ответ:

Дано:

`"а) "CH_4`

`"б) "C_3H_8`

`ω_а(C) = ?`

`ω_б(C) = ?`

Решение

`ω_а(C) = (100*n(C)*M(C))/(M(CH_4)) = (100*1*12)/16 = 75%`

`ω_б(C) = (100*n(C)*M(C))/(M(C_3H_8)) = (100*3*12)/44 = 81.8%`

Ответ: `ω_а(C) = 75%`, `ω_б(C) = 81.8%`.

С увеличением числа атомов углерода в молекуле увеличивается массовая доля углерода.

4. Предельный углеводород массой 18 г содержит 3 г водорода. Установите молекулярную формулу этого углеводорода.

Ответ:

Дано:

`m(C_nH_(2n+2)) = 18 г`

`m(H) = 3 г`

`C_nH_(2n+2) - ?`

Решение

`(2n + 2)/(12n + 2n + 2) = (m(H))/(m(C_nH_(2n+2))`

`(2n + 2)/(14n + 2) = 3/18`

`6*(2n + 2) = 14n + 2`

`n = 5`

Формула вещества: `C_5H_12`.

Ответ: `C_5H_12`.

5. Охарактеризуйте физические свойства бутана.

Ответ:

Физические свойства бутана:

Бутан — бесцветный горючий газ, со специфическим запахом, при нормальном давлении легко сжижаем от −0,5 °C, замерзает при −138 °C; при повышенном давлении и обычной температуре — легколетучая жидкость.

Химические свойства бутана:

Для бутана характерны все реакции, которые характерны для алканов.

При поджигании сгорает в кислороде воздуха:
2C4H10 + 13O2 ⟶ 8CO2 + 10H2O

Взаимодействует с хлором:
C4H10 + Cl2 ⟶ C4H9Cl + HCl

При нагревании отщепляется водород:
C4H10  t ⟶ C4H8+ H2

6. При нагревании этана отщепляется водород и образуется этилен. Напишите уравнение этой реакции. Составьте уравнения реакций этана с хлором и кислородом.

Ответ:

С2H6   t ⟶ С2H6 + H2 

C2H6 + Cl2   t ⟶ C2H5Cl  + HCl

2C2H6 + 7O2 ⟶ 4CO2 + 6H2O

7. Составьте уравнение реакции полного сгорания пропана. Какой объём кислорода необходим для сжигания 20 л пропана? Какой объём углекислого газа при этом образуется?

Ответ:

Дано:

`V(C_3H_8) = 20 л`

`V(O_2) = ?`

Решение

`C_3H_8 + 5O_2 ⟶ 3CO_2 + 4H_2O`

`V(O_2) = 5*V(C_3H_8) = 5*20 = 100 л`

`V(CO_2) = 3*V(C_3H_8) = 3*20 = 60 л`

Ответ: `V(O_2) = 100 л`, `V(CO_2) = 60 л`.

8. Объясните, почему алканы не могут вступать в реакции присоединения, а этиленовые и ацетиленовые углеводороды могут.

Ответ:

Алканы не могут вступать в реакции присоединения, потому что они являются предельным углеводородами, т. е. не содержат кратных связей.

Этиленовые и ацетиленовые углеводороды могут вступать в реакции присоединения, потому что они являются непредельным углеводородами, т. е. содержат кратные связи. По этим связям протекают реакции присоединения.

9. На рисунке 129 изображены шаростержневые модели молекул трёх этиленовых углеводородов. Составьте их молекулярные и структурные формулы.

Ответ:

Заменяем темные шарики на С, белые на Н, палочки - на |, двойные палочки на ||.

C2H4 этилен

C3H6 пропилен

C4Hбутен-2

10. Термохимическое уравнение реакции горения ацетилена имеет вид: 2C2H2 + 5O2 = 4CO2 + 2H2O + 2520 кДж. Сколько теплоты выделяется при сгорании одного литра ацетилена (в пересчёте на нормальные условия)?

Ответ:

Дано:

`V(C_2H_2) = 1 л`

`Q = ?`

Решение

`2C_2H_2 + 5O_2 ⟶ 4CO_2 + 2H_2O + 2520" кДж"`

`n(C_2H_2) = (V(C_2H_2))/V_m = 1/22.4 = 0.0446" моль"`

`Q = (Q_("х.р.")*n(C_2H_2))/(n_("х.р.")(C_2H_2)) = (2520*0.0446)/2 = 56.2" кДж"`

Ответ: `Q = 56.2" кДж"`.

§50

§50. Кислородсодержащие органические вещества

1. Назовите важнейшие кислородсодержащие органические вещества, которые вы изучили.

Ответ:

Метанол, этанол, этиленгликоль, глицерин, уксусная кислота, глюкоза, сахароза, крахмал, целлюлоза.

2. Предположим, вы съели 50 г сыра. Какую энергию получит ваш организм?

Ответ:

Калорийность сыра ~350 ккал на 100 г, если съесть 50 г сыра, то мой организм получил 175 ккал.

3. Из перечисленных формул выберите те, которые могут относиться к углеводам: C3H8O, C5H10O5, C3H8O3, C12H22O11, C6H12O.

Ответ:

К углеводам могут относиться: C5H10O5 (C5(H2O)5), C12H22O11 (C12(H2O)11).

4. Дерево способно превращать за сутки 55 г оксида углерода (IV) в углеводы. Сколько литров кислорода (н. у.) при этом выделяется?

Ответ:

Дано:

`m(CO_2) = 55 г`

`V(O_2) = ?`

Решение

`6CO_2 + 6H_2O = C_6H_12O_6 + 6O_2`

`n(CO_2) = (m(CO_2))/(M(CO_2)) = 55/44 = 1.25" моль"`

`n(O_2) = n(CO_2) = 1.25" моль"`

`V(O_2) = n(O_2)*V_m = 1.25*22.4 = 28 л`

Ответ: `V(O_2) = 28 л`.

5. Термохимическое уравнение сгорания глюкозы имеет вид: C6H12O6(тв) + 6O2(г) = 6CO2(г) + 6H2O(ж) + 2800 кДж. Сколько энергии выделяется при сгорании 1 г твёрдой глюкозы?

Ответ:

Дано:

`m(C_6H_12O_6) = 1 г`

`Q = ?`

Решение

`C_6H_12O_"6(тв)" + 6O_"2(г)" = 6CO_"2(г)" + 6H_2O_"(ж)" + 2800" кДж"`

`n(C_6H_12O_6) = (m(C_6H_12O_6))/(m(C_6H_12O_6)) = 1/180" моль"`

`1" моль "C_6H_12O_6"   "—"   "2800" кДж"`
`1/180" моль "C_6H_12O_6"   "—"   "Q`

`Q = 2800/180 = 15.6" кДж"`

Ответ: `Q = 15.6" кДж"`.

6. Рассчитайте массовые доли углерода и воды в: а) глюкозе; б) сахарозе; в) целлюлозе. Какое из этих веществ наиболее богато углеродом, а какое – водой?

Ответ:

Дано:

`"а) "C_6(H_2O)_6`

`"б) "C_12(H_2O)_11`

`"в) "(C_6(H_2O)_5)_n`

`ω_а(C) = ?`

`ω_а(H_2O) = ?`

`ω_б(C) = ?`

`ω_б(H_2O) = ?`

`ω_в(C) = ?`

`ω_в(H_2O) = ?`

Решение

`ω_а(C) = (100*n(C)*M(C))/(M(C_6(H_2O)_6)) = (100*6*12)/180 = 40%`

`ω_а(H_2O) = 100 - ω_а(C) = 100 - 40 = 60%`

`ω_б(C) = (100*n(C)*M(C))/(M(C_12(H_2O)_11)) = (100*12*12)/342 = 42.1%`

`ω_б(H_2O) = 100 - ω_б(C) = 100 - 42.1 = 57.9%`

`ω_в(C) = (100*n(C)*M(C))/(M((C_6(H_2O)_5)_n)) = (100*6*12)/162 = 44.4%`

`ω_в(H_2O) = 100 - ω_в(C) = 100 - 44.4 = 55.6%`

Ответ:
`ω_а(C) = 40%`, `ω_а(H_2O) = 60%`;
`ω_б(C) = 42.1%`, `ω_б(H_2O) = 57.9%`;
`ω_в(C) = 44.4%`, `ω_в(H_2O) = 55.6%`.

Целлюлоза наиболее богата углеродом, глюкоза наиболее богата водой.

7. Рассчитайте объём одного моля целлюлозы (степень полимеризации 10 тысяч, плотность 1,52 г/см3).

Ответ:

Дано:

`n((C_6H_10O_5)_10000) = 1" моль"`

`ρ((C_6H_10O_5)_10000) = 1.52" г/см"^3`

`V((C_6H_10O_5)_10000) = ?`

Решение

`m((C_6H_10O_5)_10000) = n((C_6H_10O_5)_10000)*M((C_6H_10O_5)_10000) = 1*162*10000 = 1.62*10^6 г`

`V((C_6H_10O_5)_10000) = (m((C_6H_10O_5)_10000))/(ρ((C_6H_10O_5)_10000)) = (1.62*10^6)/1.52 = 1.07*10^6 = 1.066*10^6" см"^3 = 1066 м^3`

Ответ: `V((C_6H_10O_5)_10000) = 1066 м^3`.

Лабораторный опыт 14. Изучение свойств уксусной кислоты.

1. Налейте в пробирку 1 мл раствора уксусной кислоты. Ознакомьтесь с её запахом. При помощи индикатора докажите её принадлежность к классу кислот.

2. В пробирку с уксусной кислотой опустите гранулу цинка. Что происходит? Проделайте аналогичный опыт с соляной кислотой. В каком случае водород выделяется сильнее? Почему?

3. Подействуйте уксусной кислотой на карбонат натрия, гидроксид меди (II). Наблюдения и уравнения реакций запишите в тетрадь.

Ответ:

1. Уксусная кислота имеет специфический резкий запах.

Лакмус в растворе уксусной кислоты окрашивается в красный цвет, что говорит о её принадлежность к классу кислот.

2. На грануле цинка в уксусной кислоте медленно выделяется газ:
Zn + 2CH3COOH ⟶ (CH3COO)2Zn + H2

В соляной кислоте газ выделяется быстрее.
Zn + 2HCl ⟶ ZnCl2 + H2

Уксусная кислота – слабая кислота, а соляная кислота – сильная кислотой, поэтому реакция с соляной кислотой протекает быстрее, чем с уксусной.

3. При действии уксусной кислоты на карбонат натрия наблюдается выделение газа.
Na2CO3 + 2CH3COOH ⟶ CH3COONa + H2O + CO2

При действии уксусной кислоты на гидроксид меди (II) наблюдается растворение последнего и окрашивание раствора в синий цвет.
Cu(OH)2 + 2CH3COOH ⟶ (CH3COO)2Cu + 2H2O

Практическая работа 1

Практикум

Практическая работа 1. Экспериментальное решение задач по теме «Электролитическая диссоциация»

Реактивы: соляная кислота, хлорид натрия, хлорид кальция, карбонат натрия, гидроксид натрия, серная кислота, сульфат натрия, сульфат магния, карбонат калия, гидроксид калия, азотная кислота, хлорид бария, хлорид железа (III), карбонат кальция, карбонат магния, сульфат меди (II), нитрат свинца (II), оксид меди (II), оксид магния, нитрат серебра.

1. Проведите реакции между растворами следующих веществ: а) карбонатом натрия и хлоридом кальция; б) карбонатом натрия и азотной кислотой; в) сульфатом магния и гидроксидом натрия. Что наблюдаете? По каким признакам можно судить о протекании реакции обмена до конца? Напишите уравнения этих реакций в молекулярном, полном и сокращённом ионном виде.

2. Проведите реакцию между раствором гидроксида натрия и соляной кислотой. Какое вещество необходимо добавить к раствору щёлочи перед проведением опыта, чтобы можно было судить о протекании реакции? Напишите уравнения этой реакции в молекулярном, полном и сокращённом ионном виде.

3. Пользуясь выданными вам реактивами, получите:
I вариант – осадки сульфата бария и гидроксида железа (III), раствор хлорида кальция;
II вариант – осадки сульфата свинца (II) и гидроксида меди (II), раствор хлорида магния.
Напишите уравнения этих реакций в молекулярном, полном и сокращённом ионном виде.

4. Пользуясь выданными вам реактивами, проведите реакции, выражаемые следующими сокращённоионными уравнениями:
I вариант – а) H+ + OH- = H2O,
б) CuO + 2H+ = Cu2+ + H2O,
в) Ag+ + Cl- = AgCl↓;
II вариант – а) 2H+ + CO32- = CO2↑ + H2O,
б) MgO + 2H+ = Mg2+ + H2O,
в) Pb2+ + 2Cl- = PbCl2↓.

5. В пяти пронумерованных пробирках находятся растворы следующих веществ:
I вариант – соляная кислота, хлорид натрия, хлорид кальция, карбонат натрия, гидроксид натрия;
II вариант – серная кислота, сульфат натрия, сульфат магния, карбонат калия, гидроксид калия.

Опытным путём определите вещества в пробирках. Напишите уравнения реакций.

Ответ:

1.

а) Признак реакции: выпадение белого осадка.
CaCl2 + Na2CO3 ⟶ 2NaCl + CaCO3
Ca2+ + 2Cl- + 2Na+ + CO32- ⟶ 2Na+ + 2Cl- + CaCO3
Ca2+ + CO32- ⟶ CaCO3

б) Признак реакции: выделение бесцветного газа.
2HNO3 + Na2CO3 ⟶ 2NaNO3 + H2O + CO2
2H+ + 2NO3- + 2Na+ + CO32- ⟶ 2Na+ + 2NO3- + H2O + CO2
2H+ + CO32- ⟶ H2O + CO2

2.

Чтобы можно было судить о протекании реакции между раствором гидроксида натрия и соляной кислотой, необходимо добавить индикатор к раствору щёлочи, например фенолфталеин. Фенолфталеин окрашивает раствор щелочи в малиновый цвет, при протекании реакции нейтрализации происходит обесцвечивание раствора.
NaOH + HCl ⟶ NaCl + H2O
Na+ + OH- + H+ + Cl- ⟶ Na+ + Cl- + H2O
H+ + OH- ⟶ H2O

3.

I вариант

Na2SO4 + BaCl2 ⟶ 2NaCl + BaSO4
2Na+ + SO42- + Ba2+ + 2Cl- ⟶ 2Na+ + 2Cl- + BaSO4
Ba2+ + SO42- ⟶ BaSO4

FeCl3 + 3NaOH ⟶ 3NaCl + Fe(OH)3
Fe3+ + 3Cl- + 3Na+ + 3OH- ⟶ 3Na+ + 3Cl- + Fe(OH)3
Fe3+ + 3OH- ⟶ Fe(OH)3

CaCO3 + 2HCl ⟶ CaCl2 + H2O + CO2
CaCO3 + 2H+ + 2Cl- ⟶ Ca2+ + 2Cl- + H2O + CO2
CaCO3 + 2H+ ⟶ Ca2+ + H2O + CO2

II вариант

Pb(NO3)2 + CuSO4 ⟶ Cu(NO3)2 + PbSO4
Pb2+ + 2NO3- + Cu2+ + SO42- ⟶ Cu2+ + 2NO3- + PbSO4
Pb2+ + SO42- ⟶ PbSO4

CuSO4 + 2NaOH ⟶ Na2SO4 + Cu(OH)2
Cu2+ + SO42- + 2Na+ + 2OH- ⟶ 2Na+ + SO42- + Cu(OH)2
Cu2+ + 2OH- ⟶ Cu(OH)2

MgO + 2HCl ⟶ MgCl2 + H2O
MgO + 2H+ + 2Cl- ⟶ Mg2+ + 2Cl- + H2O
MgO + 2H+ ⟶ Mg2+ + H2O

4.

I вариант

а) HCl + NaOH ⟶ NaCl + H2O

б) CuO + 2HCl ⟶ CuCl2 + H2O

в) AgNO3 + NaCl ⟶ NaNO3 + AgCl↓

II вариант

а) 2HCl + Na2CO3 ⟶ 2NaCl + CO2↑ + H2O

б) MgO + 2HCl ⟶ MgCl2 + H2O

в) Pb(NO3)2 + 2NaCl ⟶ 2NaNO3 + PbCl2

5.

I вариант

1. Определим в какой пробирке находится соляная кислота с помощью оксида меди (II). В пробирке с кислотой будет наблюдаться растворение оксида меди (II) и окрашивание раствора в голубой цвет:
CuO + 2HCl ⟶ CuCl2 + H2O

2. Определим в какой пробирке находится карбонат натрия с помощью небольшого количества азотной кислоты. В пробирке с карбонатом натрия будет наблюдаться появление бесцветного газа:
CaCO3 + 2HNO3 ⟶ Ca(NO3)2 + H2O + CO2

3. Определим в какой пробирке находится хлорид кальция с помощью небольшого количества карбоната натрия. В пробирке с хлоридом кальция будет наблюдаться образование белого осадка:
CaCl2 + Na2CO3 ⟶ 2NaCl + CaCO3

4. Определим в какой пробирке находится гидроксид натрия с помощью небольшого количества сульфата меди (II). В пробирке с гидроксидом натрия будет наблюдаться образование синего осадка:
2NaOH + CuSO4 ⟶ Na2SO4 + Cu(OH)2

5. В оставшейся пробирке находится хлорид натрия, проверить можно с помощью нитрата серебра. Образуется белый творожистый осадок:
NaCl + AgNO3 ⟶ NaNO3 + AgCl↓

II вариант

1. Определим в какой пробирке находится серная кислота с помощью оксида меди (II). В пробирке с кислотой будет наблюдаться растворение оксида меди (II) и окрашивание раствора в голубой цвет:
CuO + H2SO4 ⟶ CuSO4 + H2O

2. Определим в какой пробирке находится сульфат магния с помощью гидроксида натрия. В пробирке с сульфатом магния будет наблюдаться появление белого осадка:
MgSO4 + 2NaOH ⟶ Na2SO4 + Mg(OH)2

3. Определим в какой пробирке находится сульфат натрия и карбонат кальция с помощью хлорида бария и азотной кислоты.

В пробирке с сульфатом натрия выпадет белый осадок не растворимый в азотной кислоте.
Na2SO4 + BaCl2 ⟶ 2NaCl + BaSO4

В пробирке с карбонатом калия выпадет белый осадок, который будет взаимодействовать с азотной кислотой с образованием бесцветного газа:
K2CO3 + BaCl2 ⟶ 2KCl + BaCO3
BaCO3 + 2HNO3 ⟶ Ba(NO3)2 + H2O + CO2

4. В оставшейся пробирке находится гидроксид калия, проверить можно с помощью сульфата меди (II). Образуется осадок голубого цвета:
2KOH + CuSO4 ⟶ K2SO4 + Cu(OH)2

Практическая работа 2

Практическая работа 2. Получение аммиака и изучение его свойств

Оборудование и реактивы: лабораторный штатив с лапкой, спиртовка, две сухие пробирки, газоотводная трубка с пробкой, ложечка, стеклянная палочка, вата, кристаллизатор с водой, штатив с пробирками, кусок фильтровальной бумаги, хлорид аммония, гидроксид кальция, раствор аммиака, раствор серной кислоты.

1. В небольшую фарфоровую чашку или на лист бумаги насыпьте две ложечки хлорида аммония и одну ложечку гидроксида кальция. Смесь перемешайте стеклянной палочкой (запах какого газа вы почувствовали?) и насыпьте в сухую пробирку.

2. Укрепите пробирку со смесью в лапке штатива так, чтобы её дно находилось несколько выше её отверстия. В противном случае капли воды, образующейся в результате реакции, могут попасть на горячую часть пробирки, отчего она треснет. В отверстие пробирки поместите резиновую пробку с изогнутой стеклянной трубкой, которую направьте вверх (рис. 132). На неё повесьте другую пробирку, предназначенную для сбора аммиака. Обратите внимание, что эта пробирка, а также газоотводная трубка должны обязательно быть сухими. Отверстие перевёрнутой вверх дном пробирки закройте куском ваты. Объясните, почему аммиак собирают в пробирку, расположенную отверстием вниз. Можно ли собирать аммиак методом вытеснения воды? Почему?

3. Слегка прогрейте пробирку пламенем спиртовки, а затем нагревайте в том месте, где находится смесь. Какое вещество образуется на стенках пробирки? Поднесите к отверстию пробирки для сбора газа кусочек фильтровальной бумаги, смоченной раствором фенолфталеина. Что наблюдаете? Повторяйте эту процедуру несколько раз до тех пор, пока не обнаружите аммиак. После этого прекратите нагревание. Напишите уравнение реакции.

4. К отверстию пробирки с аммиаком поднесите стеклянную палочку, смоченную раствором соляной кислоты. Что происходит? Что представляет собой образовавшийся белый дым?

5. Налейте в пробирку 1-2 мл водного раствора аммиака, добавьте несколько капель фенолфталеина, а затем серной кислоты до исчезновения окраски. Напишите уравнение реакции.

6. Сделайте выводы о физических и химических свойствах аммиака.

Ответ:

1. При смешивании хлорида аммония и гидроксида кальция чувствуется запах аммиака.

2. Аммиак собирают в пробирку, расположенную отверстием вниз, потому что аммиак легче воздуха. Аммиак нельзя собирать методом вытеснения воды, потому что аммиак хорошо растворим в воде.

3. При нагревании смеси хлорида аммония и гидроксида кальция на стенках пробирки образуется вода. Фильтровальная бумага, смоченная раствором фенолфталеина, окрашивается в малиновый цвет в присутствии аммиака.
2NH4Cl + Ca(OH)2 ⟶ CaCl2 + 2NH3↑ + 2H2O

4. При внесении стеклянной палочки, смоченной в растворе соляной кислоты, в пробирку с аммиаком, наблюдается образование белого дыма – хлорида аммония.
NH3 + HCl ⟶ NH4Cl

5. 

2NH3 + H2SO4 ⟶ (NH4)2SO4

6. Физические свойства. Аммиак – бесцветный газ с очень резким характерным запахом, хорошо растворим в воде.

Химические свойства. Аммиак взаимодействует с кислотами, фенолфталеин окрашивает раствор аммиака в малиновый цвет, что говорит о щелочной реакции среды.

Практическая работа 3

Практическая работа 3. Получение углекислого газа и изучение его свойств

Оборудование и реактивы: лабораторный штатив с лапкой, аппарат Кирюшкина (либо прибор, состоящий из колбы на 100 мл и пробки с газоотводной трубкой), стакан на 100 мл, кусок бумаги, спиртовка, держатель для пробирок, лучинка, две пробирки, известковая вода (раствор гидроксида кальция), лакмус.

1. Изучите выданный вам прибор Кирюшкина для получения газов (рис. 133). Он состоит из пробирки с трубкой для выхода газа и вставленной в неё пробки с длинной воронкой, на нижней части которой укреплена прокладка. Газоотводная трубка снабжена краном, регулирующим ток выделяющегося газа. Выньте пробку с воронкой из пробирки, но не полностью, а на 2-3 см и в образовавшийся зазор опустите несколько кусочков мрамора так, чтобы они оказались на прокладке. После этого плотно закройте пробирку пробкой. Налейте в воронку 10%-й раствор соляной кислоты в таком количестве, чтобы кислота полностью закрыла мрамор. Закройте кран на газоотводной трубке. Что происходит? Если кислота поднимается вверх по воронке, значит, прибор герметичен и готов к использованию. Если в школьной лаборатории нет приборов Кирюшкина, для получения углекислого газа можно воспользоваться колбой, закрытой пробкой с помещённой в неё газоотводной трубкой (рис. 134). В колбу кладут кусочки мрамора (или насыпают порошок мела) и заливают 10%-м раствором соляной кислоты. В чём состоит недостаток такого прибора по сравнению с прибором Кирюшкина?

2. Заполните углекислым газом химический стакан. Для этого опустите конец газоотводной трубки на дно стакана и откройте кран. Чтобы углекислый газ не смешивался с воздухом, отверстие стакана закройте листом бумаги, в котором сделайте отверстие для газоотводной трубки. Через несколько минут приподнимите бумагу и поднесите к отверстию стакана зажжённую лучинку. Что наблюдаете? Продолжайте заполнение стакана углекислым газом до тех пор, пока лучинка не станет гаснуть. После этого закройте кран на аппарате Кирюшкина. Напишите уравнение реакции получения углекислого газа. Можно ли собирать углекислый газ в сосуд, расположенный вверх дном? Почему?

3. "Перелейте" собранный углекислый газ в две пробирки. При помощи зажжённой лучинки убедитесь в том, что они заполнены углекислым газом. В одну пробирку прилейте 1 мл фиолетового раствора лакмуса. Что наблюдаете? О чём свидетельствует изменение окраски индикатора? Напишите уравнение реакции.

4. К другой пробирке с углекислым газом прилейте 1 мл известковой воды (раствор гидроксида кальция). Какое вещество выпадает в осадок? Напишите уравнение реакции.

5. В пробирку с осадком карбоната кальция поместите конец газоотводной трубки аппарата Кирюшкина и откройте кран. Пропускайте углекислый газ до тех пор, пока весь осадок не растворится. Какое вещество находится в растворе? После этого нагрейте раствор в пробирке до кипения. Напишите в тетрадь уравнения реакций.

6. Сделайте выводы о физических и химических свойствах углекислого газа.

Ответ:

1. При заливе мрамора кислотой наблюдается выделение газа. Недостаток колбы с газоотводной трубкой состоит в том, что в ней нельзя остановить реакцию, а в приборе Кирюшкина – можно.

2. При внесении тлеющей лучины в стакан с углекислым газом, лучина гаснет.

Уравнение реакции получения углекислого газа:
CaCO3 + 2HCl ⟶ CaCl2 + H2O + CO2

Углекислый газ нельзя собирать в сосуд, расположенный вверх дном, потому что углекислый газ тяжелее воздуха.

3. Раствор лакмуса в углекислом газе меняет свой цвет на красный, что говорит о кислой среде.
CO2 + H2O ⇄ H2CO3
H2CO3 ⇄ H+ + HCO3-

4. При добавлении известковой воды к углекислому газу наблюдается выпадение карбоната кальция:
Ca(OH)2 + CO2 ⟶ CaCO3↓ + H2O

5. При пропускании углекислого газа через пробирку с карбонатом кальция, наблюдается растворение осадка, т. к. в результате реакции образуется растворимый гидрокарбонат кальция.
CaCO3 + H2O + CO2 ⟶ Ca(HCO3)2

После кипячения раствора с гидрокарбонатом кальция, наблюдается появление осадка карбоната кальция.
Ca(HCO3)2   t ⟶ CaCO3↓ + H2O + CO2

6. Физические свойства. Углекислый газ – это бесцветный газ с кисловатым запахом и вкусом, он тяжелее воздуха.

Химические свойства. Углекислый газ относится к кислотным оксидам. Частично взаимодействует с водой с образованием угольной кислоты, лакмус окрашивает такой раствор в красный цвет, что говори о кислой среде. Углекислый газ взаимодействует с щелочами с образованием карбонатов, также взаимодействует с карбонатами с образованием гидрокарбонатов.

Практическая работа 4

Практическая работа 4. Экспериментальное решение задач по теме «Неметаллы»

Реактивы: гидроксид натрия, серная кислота, соляная кислота, карбонат натрия, иодная вода, бромная вода, хлорная вода, крахмальный клейстер, хлорид натрия, бромид натрия, сульфат натрия, иодид натрия, фосфат калия, нитрат серебра, сульфит натрия, хлорид бария, лакмус, твёрдый сульфат натрия с примесью карбоната натрия.

1. Получите раствор хлорида натрия тремя различными способами. Напишите уравнения реакций в молекулярном и сокращённом ионном виде.

2. Докажите, что в состав соляной кислоты входят ионы водорода и хлорид-ионы, а в состав серной – ионы водорода и сульфат-ионы.

3. Опытным путём определите, содержит ли выданный вам образец сульфата натрия примесь карбоната натрия.

4. В пронумерованных пробирках находятся растворы веществ:
I вариант – хлорид натрия, сульфат натрия, соляная кислота, карбонат натрия;
II вариант – хлорид натрия, серная кислота, силикат натрия, гидроксид натрия.
Опытным путём определите растворы в пробирках. Напишите уравнения реакций.

5. Воспользовавшись реактивами, имеющимися на столе, проведите реакции, которым соответствуют следующие сокращённые ионные уравнения:
PO43- + 3Ag+ ⟶ Ag3PO4,
SO32- + 2H+ ⟶ SO2 + H2O.

Ответ:

1. 

NaOH + HCl ⟶ NaCl + H2O
Na+ + OH- + H+ + Cl- ⟶ Na+ + Cl- + H2O
H+ + OH- ⟶ H2O

2NaI + Cl2 ⟶ 2NaCl + I2
2Na+ + 2I- + Cl2 ⟶ 2Na+ + 2Cl- + I2
2I- + Cl2 ⟶ 2Cl- + I2

BaCl2 + Na2SO4 ⟶ 2NaCl + BaSO4
Ba2+ + 2Cl- + 2Na+ + SO42- ⟶ 2Na+ + 2Cl- + BaSO4
Ba2+ + SO42- ⟶ BaSO4

2.  Доказать, что в состав соляной кислоты входят ионы водорода можно с помощью лакмуса, он окрасит раствор кислоты в красный цвет.

Доказать, что в состав соляной кислоты входят хлорид-ионы можно с помощью нитрата серебра.
HCl + AgNO3 ⟶ HNO3 + AgCl↓

Доказать, что в состав серной кислоты входят ионы водорода можно с помощью лакмуса, он окрасит раствор кислоты в красный цвет.

Доказать, что в состав серной кислоты входят сульфат-ионы можно с помощью хлорида бария.
H2SO4 + BaCl2 ⟶ 2HCl + BaSO4

3. Растворим образец сульфата натрия в воде, и прильём к нему раствор соляной кислоты, если выделится газ, то сульфат натрия содержит примесь карбоната натрия, если образование газа не наблюдается, то сульфат натрия не содержит примесь карбоната натрия.
Na2CO3 + 2HCl ⟶ 2NaCl + H2O + CO2

4. 

I вариант

1) Прилить раствор лакмуса во все пробирки. Раствор соляной кислоты индикатор окрасил в красный цвет. Подтвердить можно с помощью раствора нитрата серебра, в результате их взаимодействия выпадет белый творожистый осадок.
K3PO4 + 3AgNO3 ⟶ 3KNO3 + Ag3PO4

2) Лакмус окрасил раствор карбоната натрия в синий цвет. Подтвердить, что в пробирке раствор карбоната натрия, можно с помощью соляной кислоты, в результате их взаимодействия образуется бесцветный газ.
Na2CO3 + 2HCl ⟶ 2NaCl + H2O + CO2

3) Отличить хлорид натрия от сульфата натрия можно с помощью раствора хлорида бария. При добавлении раствора хлорида бария к раствору карбонат натрия наблюдается образование белого осадка.
BaCl2 + Na2SO4 ⟶ 2NaCl + BaSO4

4) В оставшейся пробирке находится хлорид натрия.

II вариант

1) Найти серную кислоту можно с помощью лакмуса, он окрасит раствор в красный цвет. Подтвердить можно с помощью раствора хлорида бария, в результате их взаимодействия выпадет белый осадок.
H2SO4 + BaCl2 ⟶ 2HCl + BaSO4

Лакмус окрасил растворы силиката натрия и гидроксида натрия в синий цвет. Отличить их можно с помощью соляной кислоты.

2) При добавлении соляной кислоты к раствору силиката натрия наблюдается выпадение белого студенистого осадка, и лакмус меняет окраску раствора на фиолетовую.
Na2SiO3 + 2HCl ⟶ 2NaCl + H2SiO3

3) При добавлении соляной кислоты к раствору гидроксида натрия, лакмус меняет окраску раствора на фиолетовую.
NaOH + HCl ⟶ NaCl + H2O

4) В оставшейся пробирке находится раствор хлорида натрия, подтвердить можно с помощью раствора нитрата серебра, в результате их взаимодействия выпадет белый творожистый осадок.
NaCl + AgNO3 ⟶ NaNO3 + AgCl↓

5. 

K3PO4 + 3AgNO3 ⟶ 3KNO3 + Ag3PO4

Na2SO3 + 2HCl ⟶ 2NaCl + SO2↑ + H2O

Практическая работа 5

Практическая работа 5. Экспериментальное решение задач по теме «Металлы»

Реактивы: карбонаты натрия, магния и кальция, железный купорос, хлорид кальция, гидрокарбонат натрия; растворы: роданид железа (III), хлорид натрия, хлорид кальция, хлорид алюминия, гидроксид натрия, карбонат натрия, соляная кислота, хлорная вода, известковая вода, серная кислота, лакмус.

1. В пробирках находятся карбонаты натрия, магния и кальция. Определите опытным путём, какое вещество находится в каждой из пробирок.

2. Докажите опытным путём, что образец железного купороса содержит ионы железа (III).

3. Исходя из алюминия, получите раствор алюмината натрия.

4. Определите опытным путём, в какой из выданных вам пробирок находится раствор хлорида кальция, в какой – хлорида натрия, а в какой – хлорида алюминия.

5. Исходя из железа, получите хлорид железа (III).

6. Докажите опытным путём, что выданное вам вещество – это гидрокарбонат натрия.

7. Докажите опытным путём, что выданное вам вещество – это хлорид кальция.

Ответ:

1. 

   1) Налить воду в пробирки с веществами, и перемешать содержимое. Карбонат натрия растворится в воде, а карбонаты магния и кальция – нет.

Далее прилить к нерастворимым карбонатам раствор серной кислоты.

   2) Карбонат магния растворится в серной кислоте с образованием газа.
MgCO3 + 2HCl ⟶ MgSO4 + H2O + CO2

   3) В пробирке с карбонатом кальция наблюдается быстрое затухание реакции, так как на поверхности карбоната кальция образуется пассирующий слой нерастворимого сульфата кальция.
CaCO3 + 2HCl ⟶ CaSO4 + H2O + CO2

2.  Вероятно, вместо раствора роданид железа (III) должен быть выдан раствор роданида калия.

Доказать, что образец железного купороса содержит ионы железа (III), можно с помощью роданида калия, в результате реакции раствор должен окрасится в кроваво-красный цвет.
Fe2(SO4)3 + 6KSCN ⟶ 2Fe(SCN)3 + 3K2SO4

3. Раствор алюмината натрия нельзя получить, поскольку алюминат натрия взаимодействует с водой, а не растворяется в ней:
NaAlO2 + 2H2O → NaOH + Al(OH)3
Но практически во всех современных учебниках встречается эта чушь.

4. Прилить к растворам избыток раствора гидроксида натрия.

   1) В пробирке с хлоридом кальция образуется белый осадок.
CaCl2 + 2NaOH ⟶ 2NaCl + Ca(OH)2

   2) В пробирке с хлоридом алюминия в начале образуется белый осадок, а при дальнейшем добавлении раствора гидроксида натрия осадок растворится.
AlCl3 + 3NaOH ⟶ 3NaCl + Al(OH)3
Al(OH)3 + NaOH ⟶ Na[Al(OH)4]

   3) В пробирке с хлоридом натрия изменений наблюдаться не будет.

5. 

   1) Необходимо железо растворить в соляной кислоте.
Fe + 2HCl ⟶ FeCl2 + H2

   2) В образовавшемуся раствору прилить хлорную воду.
2FeCl3 + Cl2 ⟶ 2FeCl3

6. Необходимо растворить гидрокарбонат натрия в воде в сосуде с газоотводной трубкой, свободный конец трубки поместить известковую воду.

Далее необходимо довести раствор гидрокарбоната натрия до кипения, в результате чего должен образоваться углекислый газ.
2NaHCO3   t ⟶ Na2CO3 + H2O + CO2

Углекислый газ, выходящий из газоотводной трубки, вызовет помутнение известковой воды.
Ca(OH)2 + CO2 ⟶ CaCO3↓ + H2O

Доказать наличие иона натрия, можно с помощью пламени спиртовки. Опустим стальную проволоку в раствор гидрокарбоната натрия, и внесём проволоку в пламя спиртовки, пламя окрасится в желтый цвет, что говорит о присутствии ионов натрия.

7. Хлорид кальция состоит из катионов кальция и анионов хлора. В двух пробирках растворим небольшое количество хлорида кальция.

Доказать наличие хлорид-ионов можно с помощью раствора нитрата серебра, в результате их взаимодействия образуется белый творожистый осадок.
CaCl2 + 2AgNO3 ⟶ Ca(NO3)2 + 2AgCl↓

Доказать наличие иона кальция, можно с помощью пламени спиртовки. Опустим стальную проволоку в раствор хлорида кальция, и внесём проволоку в пламя спиртовки, пламя окрасится в кирпично-красный цвет, что говорит о присутствии ионов кальция.